2016 Inservice

Réussis tes devoirs et examens dès maintenant avec Quizwiz!

149. What percentage of women present with stage IV breast cancer at diagnosis in the US and Western Europe? (A) 1 (B) 5 (C) 10 (D) 20

Key: B Solution: Approximately 5% of women present with de novo stage IV breast cancer at time of initial diagnosis. References: Siegel, et al. CA Cancer J 2013.

119. What is the format most commonly used in radiation oncology clinical trials for data submission of patient treatment information? (A) RTOG (B) XML (C) DICOM (D) SQL

Key: C Solution: Although RTOG format had been previously used, DICOM and DICOM\RT is the current prevalent format for data transfer-including submission to clinical trials. References: IROC website.

29. In children, the most common location for an ependymoma is the: (A) supratentorial brain. (B) infratentorial brain. (C) cervical spine. (D) cauda equina.

Key: B Solution: In children, 90% occur in the brain, including 60% infra-tentorial. In adults 75% of ependymomas occur in the spinal cord.NF-2 patients commonly get ependymomas in the cervical cord. Myxopapillary ependymomas occur in the cauda equina or filum terminale. References: Grill Paed Drugs 5, 2003. PMID 12891536.

18. What is the maximum dose grid resolution that should be used for SBRT dose calculations? (A) 1 mm (B) 2 mm (C) 4 mm (D) 6 mm

Key: B Solution: SBRT includes extremely high-dose gradients near the boundary of the target. Due to tissue inhomogeneity and IMRT techniques, a fine dose grid should be used. TG 101 recommends 2x2 mm dose grid or finer. The use of grid sizes greater than 3 mm is discouraged for SBRT. References: Stereotactic body radiation therapy: The report of AAPM Task Group 101, 2010.

49. What is the optimal management for a medically inoperable patient with a peripherally located 4.2 cm adenocarcinoma of the upper lobe with no evidence of metastatic disease? (A) SBRT using a single fraction of 34 Gy (B) SBRT using 3 fractions of 18 Gy (C) SBRT using 4 fractions of 12 Gy followed by adjuvant chemotherapy (D) Conventionally fractionated EBRT to 60 Gy with concurrent chemotherapy

Key: B Solution: The appropriate treatment approach for a patient with a larger (> 3 cm) peripheral tumor can still be SBRT. Timmerman's initial phase I (Chest 2003) using a 3 fraction scheme included tumors up to 7 cm, which was similar to his phase 2 study (JCO 2006). The RTOG study he led allowed tumors up to 5 cm (RTOG 0236) Timmerman NEJM 2010. Videtic reported a follow up study (RTOG 0915 - ASTRO 2014) comparing 2 less aggressive schemes (34 Gy in 1 fx OR 12 Gy x 4) although this study was tested in tumors < 5 cm. 34 Gy is not an optimal approach for this patient since only a small percentage had tumors > 3 cm to test its safety and efficacy plus the follow up for this trial is still very short. Currently the standard dose is 18Gy x3. The plan from the RTOG, NRG, is to compare 18Gy x3 with 34Gy in 1. Most experts would consider 12 Gy x 4 to be equal to 18 Gy x3 although control rates may be lower for tumors > 3 cm based retrospective data. The use of chemotherapy has not been tested in this situation although some would consider it logical based on the benefit seen in the postoperative situation especially since the patients are typically not surgically staged (ANITA, ...). Although this patient population may find harm in the addition of chemotherapy (LACE meta-analysis) References: Timmerman Chest. 2003 Nov; 124(5):1946-55. Timmerman. JAMA 2010 Mar 17; 303(11):1070-6. doi: 10.1001/JAMA.2010.261. Videtic. Int J Radiat Oncol Biol Phys. 2015 Nov 15; 93(4):757-64. doi: 10.1016/j.ijrobp.2015.07.2260. Epub 2015 Jul 17. Doulliard. Int J Radiat Oncol Biol Phys. 2008 Nov 1; 72(3):695-701. doi: 10.1016/j.ijrobp.2008.01.044. Epub 2008 Apr 24. Pignon J. Clin Oncol. 2008 Jul 20; 26(21):3552-9. doi: 10.1200/JCO.2007.13.9030. Epub 2008 May 27.

132. Which planning structure is a 4D CT used to create? (A) Clinical target volume (B) Internal target volume (C) Gross target volume (D) Planning organ at risk volume

Key: B Solution: The internal target volume is generated to account for target motion inside the patient. A 4DCT is commonly used to quantify the motion due to respiration. References: ICRU 62.

23. What percentage of oropharyngeal cancers are linked with HPV in the United States? (A) 5% (B) 35% (C) 70% (D) 95%

Key: C Solution: According to a recent meta-analysis the results show an increase in the prevalence of HPV+ OPSCC from 20.9% in the pre-1990 time period to 51.4% in 1990- 1999 and finally to 65.4% for 2000-present. In this manner, our study provides further evidence to support the hypothesis that HPV-associated OPSCCs are driving the increasing incidence of OPSCC over time in the United States. References: http://www.cdc.gov/cancer/hpv/statistics/. Chaturvedi AK, Engels EA, Pfeiffer RM, et al. Human papillomavirus and rising oropharyngeal cancer incidence in the United States. Journal of Clinical Oncology 2011; 29(32):4294-4301.

244. What is the approximate minimum total dose to the testicles to impair testosterone production? (A) 1 Gy (B) 3 Gy (C) 14 Gy (D) 25 Gy

Key: C Solution: Doses as low as 0.5 to 2 Gy can cause azoospermia. These doses do not impact testosterone production, which is impaired at 14 Gy. References: Izard MA, Radiotherapy Oncology, 1995 Petersen PM, JCO, 2002.

138. What is the effect of oxygen on cell killing produced by high LET heavy-charged particles? (A) Increase (B) Decrease (C) Little change (D) Changes cell cycle dependence

Key: C Solution: High LET radiation does not depend on free radicals/oxygen. References: Goodhead DT. Mechanisms for the biological effectiveness of high-LET radiations. J Radiat Res. 1999 Dec; 40 Suppl: 1-13. Skarsgard LD. Radiobiology with heavy charged particles: a historical review. Phys Med. 1998 Jul; 14 Suppl 1:1-19.

109. What is the half-life of iridium 192? (A) 17 days (B) 60 days (C) 74 days (D) 192 days

Key: C Solution: Kahn. Physics of Radiation Therapy fourth ed., Chapter 4: Measurement of Absorbed dose; AAPM TG 51, 1999. ICRU 38.

56. The targeted agent Vismodegib for basal cell carcinoma is designed to target which of the following signaling pathways? (A) Pl3K-AKT (B) RAF (C) Hedgehog (D) Notch

Key: C Solution: Vismodegib, which was approved by the U.S. Food and Drug Administration (FDA) to treat basal cell carcinoma in 2012, is an oral inhibitor of the Hedgehog pathway and is the first systemic treatment for patients with locally advanced or metastatic basal cell carcinoma. References: Lyseng-Williamson KA, Keating GM. Vismodegib: a guide to its use in locally advanced or metastatic basal cell carcinoma. Am J Clin Dermatol 2013; 14: 61-64.

154. What is the MOST common radiation technique for treatment of pterygium? (A) Electrons with bolus (B) IMRT (C) Orthovoltage photons (D) Beta emitter brachytherapy

Key: D Solution: Contact brachytherapy with Stronium-90 is commonly utilized in the postoperative treatment of pterygium. Other methods are less focal. References: Int J Radiat Oncol Biol Phys. 1994 Jan 1; 28(1):101-3.

51. What is the correct AJCC Staging for a maxillary sinus squamous cell carcinoma invading into the pterygoid fossa with a single 3.1 cm ipsilateral node? (A) T2N2, Stage III (B) T3N1, Stage III (C) T2N2, Stage IVA (D) T3N2, Stage IVA

Key: D Solution: Maxillary sinus cancers that invade through the posterior wall of the maxillary sinus into the pterygoid fossa are T3. A 3.1 cm lymph node classifies the patient as N2a disease. N2 disease is automatically IVA disease. References: AJCC Cancer Staging manual, 7th Edition, 2010.

87. According to NSABP R-04, the addition of oxaliplatin increased rates of: (A) OS. (B) sphincter preserving surgery. (C) pathological CR. (D) grade 3 or higher diarrhea.

Key: D Solution: Patients with clinical stage II or III rectal cancer who were undergoing preoperative RT (4S Gy in 2S fractions over 5 weeks plus a boost of S.4 Gy to 10.8 Gy in three to six daily fractions) were randomly assigned to one of the following chemotherapy regimens: continuous intravenous infusional fluorouracil (CVI FU; 22S mg/m(2), 5 days per week), with or without intravenous oxaliplatin (SO mg/m(2) once per week for Sweeks) or oral capecitabine (82S mg/m(2) twice per day, 5 days per week), with or without oxaliplatin (SO mg/m(2) once per week for 5 weeks). Before random assignment, the surgeon indicated whether the patient was eligible for sphincter-sparing surgery based on clinical staging. The surgical end points were complete pathologic response (pCR), sphincter-sparing surgery, and surgical downstaging (conversion to sphincter-sparing surgery). Capecitabine/S-FU were initially given 7 days per week and then changed to 5 days per week during radiation. RESULTS: From September 2004 to August 2010, 1,608 patients were randomly assigned. No significant differences in the rates of pCR, sphinctersparing surgery, or surgical downstaging were identified between the CVI FU and capecitabine regimens or between the two regimens with or without oxaliplatin. Patients treated with oxaliplatin experienced significantly more grade 3 or 4 diarrhea (P < .001). References: O'Connell MJ, Colangelo LH, Beart RW, Petrelli NJ, Allegra CJ, Sharif S, Pitot HC, Shields AF, Landry JC, Ryan DP, Parda OS, Mohiuddin M, Arora A, Evans LS, Bahary N, Soori GS, Eakle J, Robertson JM, Moore DF Jr, Mullane MR, Marchello BT, Ward PJ, Wozniak TF, Roh MS, Yothers G, Wolmark N. Capecitabine and oxaliplatin in the preoperative multimodality treatment of rectal cancer: surgical end points from National Surgical Adjuvant Breast and Bowel Project trial R-04. J Clin Oncol. 2014 Jun 20; 32(18):1927-34. doi: 10.1200/JC0.2013.S3.77S3. Epub2014 May S. PubMed PMID: 24799484; PubMed Central PMCID: PMC40S020S.

58. Anterior urethra in male includes: (A) penile, bulbous, and prostatic urethra. (B) membranous and bulbous urethra. (C) membranous and prostatic urethra. (D) glandular, penile, and bulbous urethra.

Key: D Solution: Urethra consists of five sections: glandular, penile, bulbous urethra, membranous and prostatic urethra. Amongst these sections, the first three is considered anterior and the last two is posterior urethra, which has surgical management implications. References: Principles and Practice of Oncology by Devita, Hellman and Rosenberg.

177. Regarding RICOVER- RT-60 trial for patients with DLBCL: (A) it was designed to look at the benefits of consolidative RT for patients between ages 60-80. (B) did NOT include bulky disease. (C) it FAILED to demonstrate improvements in event free survival, progression free survival, and overall survival. (D) was a negative trial.

Key: A Solution: RICOVER RT 60 was designed to look at the benefit of RT after R-CHOPx6+2R for elderly patients between 61-80 with initial bulky disease ( 7.5 cm) and extralymphatic disease. This arm was compared with RICOVER noRT arm. The findings were that in a perprotocol analysis, RT for bulky (7.5 cm) improved HR for EFS, PFS, and OS; This was not the case when all eligible patients were included (including the 11patients that did not get RT per trial design). Thus, answers A, B, and D are incorrect. References: Held, et al. JCO 2014. Apr 10; 32(11):1112-8.

127. During an emergency HDR source retraction, a person entered the room for 1 min while standing approximately 1 meter away from the patient and manually retracted the source. What is the estimated dose (cGy) to the person? (A) <0.5 (B) 1.0 - 5.0 (C) 6.0 - 10.0 (D) >10.0

Key: A Solution: The approximate dose rate at 1 m from a 10 Ci source is 0.08 cGy/min. References: Kubo DH, et al. High dose-rate brachytherapy treatment delivery: Report of the AAPM Radiation Therapy Committee Task Group No. 59. Med Phys. 25(4). 1998.

106. In what sequence should radiation be administered in the management of Stage III/IV endometrial cancer? (A) Preoperatively (B) Postoperatively before chemotherapy (C) Preoperatively with concurrent chemotherapy (D) Postoperatively sandwiched between chemotherapy

Key: D Solution: Patients with advanced endometrial cancer benefit from chemotherapy and radiation in the postoperative adjuvant setting. Multiple studies have examined the sequencing of adjuvant therapy. The studies have had some weaknesses limiting absolute recommendation but at this time the sandwich technique has achieved better outcome. Alvarez's group in a multicenter retrospective study compared 3 outcomes: 1) Radiation followed by chemotherapy (3 year PFS/OS =47%/54%); 2) Chemotherapy followed by radiation (3 year PFS/OS =52%/57%); 3) Chemotherapy, radiation followed by more chemotherapy (3 year PFS/OS = 69%/88%). While most guidelines suggest chemotherapy and radiation the sequence is not specified but left to the discretion of the treating physician. References: SGO Clinical Practice Endometrial Cancer Working Group, Endometrial Cancer: A review and current management strategies: Part II. Gynecologic Oncology 134(2014) 393-402. Nag, et al. The American Brachytherapy Society Recommendations for High-Dose-Rate Brachytherapy For Carcinoma of the Endometrium. IJROBP Vol 48, No. 3, pp 779-790, 2000.

68. What is the proper management for a SCLC patient with symptomatic thoracic disease but has extensive stage disease based on a solitary asymptomatic brain metastasis? (A) Chemotherapy followed by PCI (B) WBRT plus SRS followed by chemotherapy (C) Definitive chemoRT to the thorax followed by PCI (D) Chemotherapy followed by RT to the thorax and brain, based on no evidence of progression

Key: D Solution: Patients with extensive stage SCLC can be managed with induction chemotherapy with cis or carboplatin / etoposide for 4 cycles, and if there is no evidence of disease progression, to be consolidated with chest radiotherapy to 30 Gy in 10 fx (Slotman 2014) and whole brain radiation. Given the patient is asymptomatic from the disease in the brain and the fact that response rates in the brain are equal to response rates in the chest with chemotherapy, immediate radiation treatment to the brain is not necessary. References: Slotman, et al. Lancet Oncol 2014.

43. In the recurrent setting, compared to p16 negative oropharyngeal cancer, p16 positive cancers: (A) have different patterns of failure. (B) have different median time to progression. (C) can not be salvaged with surgery. (D) have doubled survival rate.

Key: D Solution: Patterns of failure and median time to progression (8.2 v 7.3 months; P = .67) were similar for patients with p16-positive and p16-negative tumors. After a median follow-up period of 4.0 years after disease progression, patients with p16-positive OPC had significantly improved survival rates compared with p16-negative patients (2-year OS, 54.6% v 27.6%; median, 2.6 v 0.8 years; P < .001). p16-positive tumor status (HR, 0.48; 95% Cl, 0.31 to 0.74) and receipt of salvage surgery (HR, 0.48; 95% Cl; 0.27 to 0.84) reduced risk of death after disease progression whereas distant versus locoregional progression (HR, 1.99; 95% Cl, 1.28 to 3.09) increased risk, after adjustment for tumor stage and cigarette pack-years at enrollment. Tumor HPV status is a strong and independent predictor of OS after disease progression and should be a stratification factor for clinical trials for patients with recurrent or metastatic OPC. References: Fakhry, et al. Human Papillomavirus and Overall Survival After Progression of Oropharyngeal Squamous Cell Carcinoma. JCO October 20, 2014 vol. 32 no. 30 3365- 3373.

199. What is an advantage of proton therapy over plaque brachytherapy for retinoblastoma treatment? (A) Eye immobilization is unnecessary during radiation treatment (B) Can treat unilateral tumors (C) Has fewer anterior segment complications (D) Can treat tumors that invade the optic nerve

Key: D Solution: Proton therapy can treat tumors close to or invading the optic nerve. The use of plaque brachytherapy to treat a tumor invading the optic nerve would deliver too much dose to the opticnerve. Both proton therapy and plaque therapy can treat unilateral tumors. Eye immobilization during proton therapy is essential for reproducibility of treatment. With plaque therapy, the radiation source moves with the eye. Plaque therapy has fewer anterior segment complications. References: Sethi, et al. The American Brachytherapy Society consensus guidelines for plaque brachytherapy of uveal melanoma and retinoblastoma, Brachytherapy. 2014 Jan- Feb; 13(1). Second nonocular tumors among survivors of retinoblastoma treated with contemporary photon and proton radiotherapy, Cancer. 2014.

222. In the RTOG 9910 randomized controlled trial of intermediate risk prostate cancer patients, extending androgen deprivation therapy (ADT) from 8 weeks to 28 weeks before radiotherapy: (A) did not improve OS but did improve DFS. (B) did improve OS but did not improve DFS. (C) did improve both OS and DFS. (D) did not improve either OS or DFS.

Key: D Solution: RTOG 9910 showed that there was no benefit on any outcome by extending 8 weeks to 28 weeks of ADT before XRT for intermediate risk patients. References: Pisansky TM, et al "Duration of androgen suppression before radiotherapy for localized prostate cancer: RTOG randomized clinical trial 9910. JCO 2015 Feb; 33(4):332-9.

290. Which methodology is used to measure the reproductive integrity of tumor cells treated with radiation? (A) Multiplex Fluorescence In Situ Hybridization (M-FISH) (B) MTT cell growth assay (C) Single cell electrophoresis (COMET) assay (D) Puck and Marcus clonogenic survival assay

Key: D Solution: Radiosensitivity is defined by the ability of an irradiated single cell to form a surviving viable colony of more than 50 cells (>6/7 cell divisions). The Puck and Marcus clonogenic survival assay measures the ability of single cells to form multicellular colonies. References: Franken, Rodermond, Stap, Haveman, and van Bree. Clonogenic assay of cells in vitro. Nat Protoc. 2006; 1(5):2315-9.

165. What is the BEST measure of plan quality for spinal SRS? (A) Conformity index (B) Gradient index (C) 12 Gy volume (D) Spinal cord dose gradient

Key: D Solution: Radiosurgery of the spinal and brain will often have different treatment planning goals. The primary goal of spinal radiosurgery is to not overdose the spinal cord and thus the plan is created to produce a very tight gradient in one direction rather than general conformity. The conformity index, gradient index and 12 Gy volume are better measures of plan quality for radiosurgery in the brain. References: Cancer. 2007 Feb 1; 109(3):628-36.RTOG 06-31.

7. The sensitivity of a screening test is calculated as the percentage of subjects: (A) without disease who are classified as having disease. (B) with disease who are classified as not having disease. (C) without disease who are classified as not having disease. (D) with disease who are classified as having disease.

Key: D Solution: Screening refers to the application of test to people who as yet have no symptoms of a particular disease. It is classified as having a positive (disease likely) or negative (disease unlikely) finding. Diagnostic tests tell whether or not a subject actually has the disease. The performance of a screen test is considered by the sensitivity and specificity. The sensitivity is the percentage of subjects with disease who are classified as having disease and the specificity is the percentage of subjects without disease who are classified as not having disease. Those subjects with the disease should all be classified as having disease, and those subjects without the disease should be classified as not having disease. Therefore a highly sensitive and specific test is preferred. Sensitivity=50/(50+50)*100%=50%.

12. Which of the following defines stage III multiple myeloma? (A) Hemoglobin > 8.5 g/dL (B) Serum calcium > 6 mg/dL (C) Bence Jones protein > 6 g/24 hrs (D) Advanced lytic lesions

Key: D Solution: Stage III MM is defined by either hemoglobin <8.5 g/dl, calcium >12 mg/dL, Bence Jones protein >12 g/24 hrs or advanced lytic lesions. References: http://www.nccn.org/professionals/physician_gls/PDF/myeloma.pdf.

198. Why would MORE late toxicity be observed after 3fx in 6 hours, compared with 3fx in 24 hours? (A) Potentially lethal damage repair (B) Cell cycle redistribution to a radiosensitive G2/M phase (C) Accelerated repopulation cannot occur (D) Insufficient time for sublethal damage repair

Key: D Solution: Sublethal damage repair refers to damage that can repair if given sufficient time, presumably due to the time to repair complex DNA lesions that require homologous recombination. Potentially lethal damage refers to modulating the post-RT environment to allows cells to live, e.g. adding conditioned media to tissue culture cells. Accelerated repopulation and cell cycle redistribution refer to phenomenon observed in tumors rather than normal tissue. EORTC 22791 showed a reduction of fibrosis less than expected from LQ-Model. References: Hall and Giacca. Radiobiology for the Radiologist, in Bentzen SM, Saunders Ml, Dische S. Repair halftimes estimated from observations of treatment-related morbidity after CHART or conventional radiotherapy in head and neck cancer. Radiother Oncol. 1999 Dec; 53(3):219-26.Edition. Chapter 3, 4, 5.

239. According to NSABP R-03, which compared preoperative versus postoperative chemoRT for locally advanced rectal cancer: (A) OS was significantly improved with preoperative chemoRT. (B) LC was significantly improved with preoperative chemoRT. (C) pathological CR was 9%. (D) no patient with a pathological CR recurred.

Key: D Solution: The 5-year DFS for preoperative patients was 64.7% v 53.4% for postoperative patients (P=.011). The5-year LC for preoperative patients was 10.7% vs. 10.7% for postoperative patients (P=0.69). The 5-year OS for preoperative patients was 74.5% v 65.6% for postoperative patients (P=.065). A complete pathologic response was achieved in 15% of preoperative patients. No preoperative patient with a complete pathologic response had a recurrence. References: Roh MS, Colangelo LH, O'Connell MJ, Yothers G, Deutsch M, Allegra CJ, Kahlenberg MS, Baez-Diaz L, Ursiny CS, Petrelli NJ, Wolmark N. Preoperative multimodality therapy improves disease-free survival in patients with carcinoma of the rectum: NSABP R-03. J Clin Oncol. 2009 Nov 1; 27(31):5124-30. doi: 10.1200/JC0.2009.22.0467. Epub 2009 Sep21. PubMed PMID: 19770376; PubMed Central PMCID: PMC2773471.

57. What is the total dose and CTV definition for APBI using 3D EBRT per NSABP B39/RTOG 0413? (A) 34 Gy; excision cavity +10 mm (B) 34 Gy; excision cavity +1 mm (C) 38.5 Gy; excision cavity +10 mm (D) 38.5 Gy; excision cavity +15 mm

Key: D Solution: The CTV uniform expansion of 15 mm on the excision site limited to 5 mm from the skin surface and by the posterior breast tissue extent (chest wall and pectoralis muscles). The EBRT dose is 38.5 Gy in 10 fractions given twice daily. The brachytherapy dose is 34 Gy in 10 fractions given twice daily. References: NSABP B39 / RTOG 0413 protocol definitions.

59. Regarding the European phase III study evaluating the benefit of thoracic RT in patients with extensive stage SCLC (Slotman et al. 2015): (A) OS at 1 yr was significantly improved with the addition of thoracic RT. (B) OS at 2 yr was NOT improved with the addition of thoracic RT. (C) Only CR to chemotherapy were eligible. (D) The primary endpoint was NOT met.

Key: D Solution: The Slotman, et al. study examined 498 patients with extensive stage SCLC. Patients with any response to initial chemotherapy (4-6 cycles of platinum/etoposide) were eligible. PCI was mandated for all patients. The thoracic radiation dose was 30Gy/10 fractions. The primary endpoint, of 1 year overall survival was not met. Median overall survival was also similar between the two arms. Two year overall survival, however favored the addition of thoracic irradiation 13% vs 3%. Rates of thoracic progression were significantly lower in the thoracic radiation group (44% vs. 80%). References: Slotman, et al. Lancet. 2015 Jan 3; 385(9962):36-42

88. Based on phase III data, the addition of trastuzumab has been shown to improve OS among which stage of patients with Her-2-overexpressing gastroesophageal cancer? (A) None (B) Stage II (C) Stage III (D) Stage IV

Key: D Solution: The ToGA trial demonstrated an improvement in survival when trastuzumab was added to standard chemotherapy in advanced gastroesophageal cancer. The vast majority of patients (97%) had metastatic disease. The RTOG 1010 trial is ongoing to evaluate the benefit of trastuzumab in earlier stages of disease, when added to standard preoperative chemoradiation. References: Bang YJ, Van Cutsem E, Feyereislova A, et al. Trastuzumab in combination with chemotherapy versus chemotherapy alone for treatment of HER2-positive advanced gastric or gastro-oesophageal junction cancer (ToGA): a phase 3, open-label, randomized controlled trial. Lancet. 2010 Aug 28; 376(9742):687-97.

169. Per the WECARE study, what is the cumulative risk of a breast cancer patient developing a contralateral breast cancer at 10 years? (A) BRCA1 carrier - 40% (B) BRCA2 carrier - 30% (C) BRCA1 or BRCA2 carrier - 35% (D) Non-carrier - 5%

Key: D Solution: The WECARE Study is a population-based, nested case-control study of contralateral breast cancer. The study included five population-based cancer registries covering the country of Denmark along with the State of Iowa, Los Angeles County and the Orange County-San Diego regions of California, and three western Washington counties in the United States (the US registries participate in the Surveillance, Epidemiology, and End Results [SEER] registry system). In this nested case-control study, patients with contra lateral breast cancer diagnosed 1year or more after a first primary breast cancer (n=705) and controls with unilateral breast cancer (n=1,398) were ascertained from an underlying population-based cohort of 52,536 women diagnosed with a first invasive breast cancer before age 55 years. Cumulative 10 year breast cancer risk is: BRCA1 carrier: 20% BRCA2 carrier: 16% BRCA1 or BRCA2 carrier: 18% Non-carrier: 5% References: Malone, KE, et al. Population-Based Study of the Risk of Second Primary Contra lateral Breast Cancer Associated with Carrying a Mutation in BRCA1 or BRCA2. J. Clinical Oncol. 2010 May 10; 28(14): 2404-2410.

265. Which concept is TRUE about radiation effects on the embryo and fetus? (A) Malformation effects are the same for each gestation stage (B) Significant congenital malformations are only seen after 20 weeks gestation (C) Mental retardation is independent of time of gestation (D) Radiation produces microcephaly and shorter stature

Key: D Solution: The effects of radiation to the embryo and fetus depend upon the stage of gestation, the dose, and the dose rate. Congenital malformations are seen with radiation in the early (2-6 weeks) phases of development. Severe mental retardation is nearly 4 times more common if the radiation is received between 8 and 15 weeks than if it is received later. Children exposed in utero are shorter, lighter, and have a smaller head diameter than those not exposed to radiation. References: Greskovich JF Jr, Macklis RM. Radiation therapy in pregnancy: risk calculation and risk minimization. Semin Oncol. 2000 Dec; 27(6):633-45. Otake M1, Schull WJ. Radiation-related small head sizes among prenatally exposed Abomb survivors. Int J Radiat Biol. 1993 Feb; 63(2):255-70. Otake M, Schull WJ, Lee S. Threshold for radiation-related severe mental retardation in prenatally exposed A-bomb survivors: a re-analysis. Int J Radiat Biol. 1996 Dec; 70(6):755-63.

243. Electron beams do NOT strictly obey a simple inverse square law like X-ray beams because: (A) the source size is large compared to the distances involved. (B) the SAD is smaller than for X-rays. (C) the penumbra is less steep. (D) electrons scatter more than X-rays.

Key: D Solution: The inverse square law depends on the implicit assumption that there is no (or minimal) scattering. See McDermott & Orton, pg. 5-12.Option A is a false statement. Choice B is true but it is not the primary physical reason. Option C is not directly relevant. References: McDermott & Orton, pg. 5-12.

44. Where is the most frequent site of LRR following mastectomy without PMRT? (A) Axillary lymph nodes (B) Internal mammary lymph nodes (C) Supraclavicular lymph nodes (D) Chest wall

Key: D Solution: The most frequent site of locoregional recurrence after mastectomy without radiation is the chest wall, followed by the axillary and supraclavicular nodal regions. References: Nielsen, et al. JCO 2006 and Recht, et al JCO 1999.

74. What does the Life Span Study (LSS) of the Japanese atomic bomb survivors indicate for the shape of the additive radiation dose response for solid tumors over the entire dose range available? (A) Linear-quadratic with a threshold (B) Linear-quadratic no threshold (C) Linear with a threshold (D) Linear no threshold

Key: D Solution: The most recent report of the Radiation Effects Research Foundation (RERF) has found that the shape of the additive radiation dose response for solid tumors is and continues to be linear for the entire dose range. However, when the dose range was limited to low doses (between 0 and 2 Gy) a significant amount of curvature was observed for the first time. References: Ozasa I, et al. Studies of the Mortality of Atomic Bomb Survivors, Report 14, 1950-2003: An Overview of Cancer and Noncancer Diseases. Radiat. Res. 177: 229-243 (2012).

205. Which detector can be used to measure absolute dose without calibration? (A) Thermoluminescent dosimeter (B) Radiochromic film (C) Diode (D) Calorimeter

Key: D Solution: The only way to measure absolute dose is by measuring the energy absorbed directly. This is done in calorimeters by relating a very precisely measured temperature change to energy deposition through the use of specific heat capacity. All other methods need to be calibrated first by delivering a known dose and relating the response to that known dose. References: Khan FM. The Physics of Radiation Therapy, Third Ed, Lippincott Williams & Wilkins, New York, 2003, pp 142-143.

72. Which of the following objectives is the goal of a phase I trial? (A) To assess efficacy (B) To estimate overall survival (C) To determine the overall response rate (D) To determine the maximum tolerated dose (MTD)

Key: D Solution: The primary objective of a phase II study is to obtain evidence of efficacy (e.g. response rate or survival) at an already-established safe dose, with common secondary objectives related to safety and tolerability. The maximum tolerated dose (MTD) is typically determined in Phase I dose-finding studies. References: Friedman LM, Furberg CD, DeMets D. (2010). Fundamentals of clinical trials (4th ed.) New York: Springer.

136. What repair process is used to fix the most lethal DNA lesions that are produced by ionizing radiation? (A) Base excision (B) Nucleotide excision (C) Mismatch (D) DNA double strand break

Key: D Solution: The principal mechanism of action for ionizing radiation is DNA double strand damage. Therefore, DNA double strand repair is the #Key mechanism for cellular recovery and survival. References: https://en.wikipedia.org/wiki/DNA_repair. Olive PL. The role of DNA single- and double-strand breaks in cell killing by ionizing radiation. Radiat Res. 1998 Nov; 150(5 Suppl):542-51.

181. Which of the following daily fractionation in Gy showed a superior local control for stage I glottis carcinoma in the randomized trial from Osaka Japan (IJROBP 2006)? (A) 1.8 (B) 2.0 (C) 2.1 (D) 2.25

Key: D Solution: The study compared 2 radiation fractionation regimens in the treatment of T1N0M0 SCC of the glottis (patients were stratified by tumor size): 60 Gy in 30 fractions/66 Gy in 33 fractions (2 Gy/frx) vs. 56.25 Gy in 25 fractions/63 Gy in 28 fractions (2.25 Gy/frx). The hypofractionation regimen showed superior local control in this study. References: Yamazaki H, et al. Radiotherapy for early glottic carcinoma (T1N0M0): results of prospective randomized study of radiation fraction size and overall treatment time. Int J Radiat Oncol Biol Phys. 2006 Jan 1; 64(1):77-82.

61. Per ABS (2000) guidelines, when using 4 fractions of HDR only in the treatment of inoperable Stage I endometrial cancer, what dose per fraction scheme should be utilized? Specify dose (Gy) at 2 cm from the midpoint of the intrauterine sources. (A) 5.7 (B) 6.4 (C) 7.3 (D) 8.5

Key: D Solution: The suggested dose using 4 fractions of HDR as a sole treatment modality is 8.5 Gy at 2 cm. The other dose regimens are delivered at higher fractions: 7.3 Gy in 5 fractions, 6.4 Gy in 6 fractions and 7 fractions of 5.7 Gy. References: Int. J. Radiation Oncology Biol. Phys., Vol 48. No. 3, pp. 779-790, 2000.

268. What is the MOST common site of metastases in patients with pediatric Ewing sarcoma? (A) Bone marrow (B) Lymph nodes (C) Central nervous system (D) Lungs

Key: D Solution: The two most common sites of metastases in pediatric Ewing sarcoma patients are lungs and focal bone locations. References: Vietti, et al. National Cancer Inst. Monogr. 1981(56):279-284.

261. What was observed regarding rates of brachial plexopathy in recent studies evaluating the role for regional nodal irradiation (RNI) in early stage breast cancer? (A) Greater incidence with inclusion of the medial supraclavicular nodes (B) Greater incidence with inclusion of the internal mammary lymph nodes (C) Greater incidence with inclusion of both the medial supraclavicular and internal mammary lymph nodes (D) No difference with regional nodal irradiation

Key: D Solution: There was no reported increase in brachial plexopathy between the regional nodal irradiation and whole breast groups on MA-20 and no reported increase in other late effects in the EORTC trial between RNI and breast/chest wall only groups (pulmonary fibrosis rate did differ). References: MA 20 and EORTC 22922 both published in NEJM 2015.

211. In addition to the surgical bed, what areas should be included in postoperative radiation for a patient with recurrent cutaneous SCC in the temple with facial nerve paralysis? (A) None (B) CN VII (C) CN VII and ipsilateral parotid (D) CN VII, base of skull and ipsilateral parotid

Key: D Solution: This immunocompromised patient is presenting with a high risk lesion with clinical perineural invasion at presentation. Retrospective data from Mendenhall and Gluck would suggest benefit to covering the course of the involved cranial nerve and consideration for elective coverage of the cavernous sinus and R parotid to provide coverage of first echelon lymph nodes and junction points with other cranial nerves such as CN VII, III, IV and VI. References: Gluck, et al. Skin cancer of the head and neck with perineural invasion: defining the clinical target volumes based on the pattern of failure. IJROBP 2009; 74(1):38-46. Mendenhall, et al. Skin cancer of the head and neck with perineural invasion. Am J Clin Oncol 2007; 30:93-6.

97. What is an appropriate RT regimen for a 6 year old with a completely resected medulloblastoma without anaplasia and with negative CSF? (A) 50.4 Gy to the resection bed (B) 59.4 Gy to the resection bed (C) 36 Gy CSI followed by a 19.8 Gy boost to the posterior fossa (D) 23.4 Gy CSI followed by a 30.6 Gy boost to resection bed

Key: D Solution: This patient has average risk medulloblastoma. Grade IV histology without diffuse anaplasia does not make the patient high risk. Packer, et al, have shown equivalent efficacy of low dose craniospinal irradiation with concurrent and adjuvant chemotherapy to patients treated with high dose chemotherapy. C represents the regimen on the ongoing Phase II study for high risk medulloblastoma patients, ACNS0332. References: Packer, et al. Journal of Clinical Oncology 17:2127-2136.

30. Which of the following statements was TRUE in the combined analysis of the surgery vs. SBRT trials for early stage NSCLC (STARS and ROSEL; Chang, et al. Lancet Oncology 2015)? (A) LC was higher in the surgery arm (B) OS was higher in the surgery arm (C) LC was higher in the SBRT arm (D) OS was higher in the SBRT arm

Key: D Solution: This study combined patients from two incompletely accruing SBRT vs. surgery trials for early stage NSCLC (cT1-T2a < 4 cm) in medically operable patients. It included only 58 patients, however contains much important data. There was no difference in local control, regional control, or distant control between surgery and SBRT. There were no grade 4-5 toxicities in the SBRT arm. One patient died of surgical complications in the surgery arm. There was no difference in recurrence free survival (86% in the SBRT group vs. 80% in the surgery group, p = ns). There was, however, a difference in 3 year overall survival (95% in the SBRT group vs. 79% in the surgery group). References: Chang, et al. Lancet Oncol. 2015 Jun; 16(6):630-7.

293. In the RTOG 92-07 trial of brachytherapy boost with concurrent chemotherapy for esophageal cancer, which toxicity was the major reason that this treatment strategy was abandoned? (A) Bleeding (B) Pneumonitis (C) Hematologic (D) Fistula

Key: D Solution: This trial demonstrated a 12% incidence of fistulas, some of which were fatal. Primarily for this reason, the authors did not recommend this approach to be extended further into a Phase Ill trial. References: Gaspar LE, Qian C, Kocha WI, Coia LR, Herskovic A, Graham M. A phase I/II study of external beam radiation, brachytherapy and concurrent chemotherapy in localized cancer of the esophagus (RTOG 92-07): preliminary toxicity report. IntJ Radiat Oncol Biol Phys. 1997 Feb 1; 37(3):593-9.

64. What is the expected CR (%) rate for invasive male urethral cancer treated with chemoRT (2 cycles of 5-FU and mitomycin-C)? (A) 20 (B) 40 (C) 60 (D) 80

Key: D Solution: Though surgical resection is the most common treatment in this rare tumor site, combined chemoradiation has been shown to be effective with a complete response rate of 79%, and disease specific and disease free survival rates of 68.4% and 43.2% respectively. Cheomradiation may be effective for male urethral cancer, and should be considered for patients who decline surgery or are not surgical candidates. References: Kent M, et al. Combined Chemoradiation as Primary Treatment for Invasive Male Urethral Cancer. J. Ural 2015 Feb; 193(2): pp 532-537.

31. Per the recent Trans-Tasman Radiation Oncology Group (TROG) study, short course radiotherapy for rectal cancer: (A) is administered with current 5-FU chemotherapy. (B) is followed by surgical resection after 10-14 days. (C) is administered over 2 weeks. (D) includes elective LN coverage up to the sacral promontory.

Key: D Solution: Three hundred twenty-six patients were randomly assigned; 163 patients to SC and 163 to LC. Median potential follow-up time was 5.9 years (range, 3.0 to 7.8 years). Three-year LR rates (cumulative incidence) were 7.5% for SC and 4.4% for LC (difference, 3.1%; 95% CI, -2.1 to 8.3; P = .24). For distal tumors (< 5 cm), six of 48 SC patients and one of 31 LC patients experienced local recurrence (P = .21). Five-year distant recurrence rates were 27% for SC and 30% for LC (log-rank P = 0.92; hazard ratio [HR] for LC:SC, 1.04; 95% CI, 0.69 to 1.56). Overall survival rates at 5 years were 74% for SC and 70% for LC (log-rank P = 0.62; HR, 1.12; 95% CI, 0.76 to 1.67). Late toxicity rates were not substantially different (Radiation Therapy Oncology Group/European Organisation for Research and Treatment of Cancer G3-4: SC, 5.8%; LC, 8.2%; P = .53). References: Ngan SY, et al. Randomized Trial of Short-Course Radiotherapy Versus Long- Course Chemoradiation Comparing Rates of Local Recurrence in Patients with T3 Rectal Cancer: Trans-Tasman Radiation Oncology Group Trial 01.04. JCO 2012 30(31): 3827-33.

156. What is the minimum incident photon energy required for triplet production? (A) 0.511 MeV (B) 1.02 MeV (C) 1.53 MeV (D) 2.04 MeV

Key: D Solution: Triplet production occurs in the electric field of an atomic electron and the atomic electron will recoil with sufficient energy to be ejected from the atomic shell. The energy threshold is four times the rest mass energy of an electron. References: Mayles P. Handbook of Radiotherapy Physics.

170. The volume doubling time (in days) for a tumor with a cell loss factor of 90% and a potential doubling time (Tpot) of 10 days would be estimated as: (A) 5 (B) 10 (C) 50 (D) 100

Key: D Solution: Tumor volume doubling time is estimated: =1-(Tpot / TD) where is the cell loss factor, Tpot is the potential doubling time and TD is the measured tumor volume doubling time. 0.9=1-(10 days/Tv01) or Tvol = 100 days. References: Radiobiological Modelling in Radiation Oncology; Chapter 5 repopulation effects. Edited by Roger G. Dale, Bleddyn Jones.

3. For a female patient with metastatic squamous cell carcinoma of the lung and a minimal smoking history, to which novel therapeutic is she MOST likely to respond? (A) Crizotinib (B) Ipilimumab (C) Erlotinib (D) Nivolumab

Key: D Solution: lpilimumab has not been shown to be effective in lung cancer. Erlotinib, Gefitinib, Afatinib and Crizotinib have all been shown to be effective in select Adenocarcinomas who clinically have the typical presentation of a young Asian female nonsmoker and of course now has an EGFR mutation or in crizotnib's case has the gene rearrangement of ALK. Nivolumab is the first novel agent approved for squamous cell carcinoma which is a fully human lgG4 programmed death 1(PD-1) immune- checkpointinhibitor antibody which doubled the response rate and doubled survival in comparison to docetaxel which is FDA approved in the second line setting. References: Brahmer J, et al. N Engl J Med 2015; 373:123-135July 9, 2015001: 10.1056/NEJMoa1504627.

75. What is a

Key advantage of MVCT over kV CBCT? (A) Better soft tissue contrast (B) Better spatial resolution (C) Fewer artifacts from metal objects (D) Lower dose per image #Key: C Solution: MVCT has a higher average beam energy than kVCT, which means fewer of interactions between the beams and the target will be governed by the photoelectric effect, which is very sensitive to differences in atomic number (Z). This means that MVCT will have fewer artifacts from metal objects such as fiducials, dental fillings, and metal prosthetics. References: Khan FM. "Physics of Radiation Therapy, 4th Edition", Chapter 25.

186. What is a

Key advantage of magnetic resonance imaging (MRI) scans over computer tomography (CT) scans? (A) High geometric accuracy (B) Lower susceptibility to motion artifacts (C) Improved bone imaging (D) Improved soft tissue contrast #Key: D Solution: MRI and CT are complementary imaging modalities in radiotherapy. CT scans essentially measure electron density relative to water, which, as it is roughly related to mass density for low-Z materials, provides limited contrast between different tissue types, as they tend to have very similar material compositions and densities. MRI images tend to have greater contrast, especially in soft tissues, where small differences in proton densities can lead to large differences in signal strength, creating high contrast images. References: Khan FM. "Physics of Radiation Therapy," 4th Edition, Chapter 12.

53. Which cytokine is elevated after lung irradiation and is associated with pneumonitis injury? (A) (B) IL-8 (C) INF- (D) GM-CSF

Key: A References: Anscher MS, Kong FM, Andrews K, Clough R, Marks LB, Bentel G, Jirtle RL. Plasma transforming growth factor beta1 as a predictor of radiation pneumonitis. Int J Radiat Oncol Biol Phys. 1998 Jul 15; 41(5):1029-35.

117. In tumors, how can HIF- (A) binds to the regulatory region of the VEGF gene (B) activates phosphatidylinositol 3-kinase (PI3K) signaling (C) binds to ATM to promote signaling (D) produces ATP via oxidative phosphorylation

Key: A Solution: References: Fokas E, McKenna WG, Muschel RJ. Cancer Metastasis Rev., (2012). The impact of tumor microenvironment on cancer treatment and its modulation by direct and indirect antivascular strategies. 2012 Dec; 31(3-4):823-42. Dewhirst MW, Cao Y, Moeller B., Nat Rev Cancer, (2008). Cycling Dewhirst MW, Cao Y, Moeller B. hypoxia and free radicals regulate angiogenesis. 2008 Jun; 8(6):425-37.

19. Which lymphoma has a high risk of CNS recurrence? (A) Ocular lymphoma involving the retina (B) MALT lymphoma of the conjunctiva (C) Diffuse large cell lymphoma of the extra-ocular orbital tissues (D) T-cell lymphoma of the skin with involvement of the eyelid

Key: A Solution: 50-80% of ocular lymphomas patients have CNS progression. The remaining choices less commonly progress in the CNS. References: Blood. 2013; 122(14):2318-2330.

1. RTOG 9804 (Phase III Trial of Observation +/- Tamoxifen vs. RT +/- Tamoxifen for Good Risk DCIS) found which of the following 7-year LR rates? (A) 1% with RT (B) 5% with RT (C) 10% without RT (D) 15% without RT

Key: A Solution: 7 year recurrence rate is 0.9% with RT and 6.7% without RT. References: RTOG 9804: prospective randomized trial for good-risk ductal carcinoma in situ comparing radiotherapy with observation. McCormick, B. et al. J Clin Oncol. 2015 Mar 1:33(7):709-715.

116. In a clinical trial with a PFS endpoint, which of the following status at last followup is counted as a censored observation? (A) Grade 4 treatment-related toxicity (B) Grade 5 adverse event (C) Alive with oligometastatic progression (D) Death, cause unknown

Key: A Solution: A Grade 4 toxicity is a life-threatening event, with censoring of the PFS time as the patient is still alive. PFS time is measured until the first occurrence of any disease failure or death due to any cause.

128. According to AAPM Task Group 51 recommendations for MV linac output calibration, the ion chamber calibration should be based on: (A) absorbed dose to water. (B) exposure. (C) dose to air. (D) air kerma.

Key: A Solution: AAPM TG 51 is an absorbed dose to water calibration. Its predecessor, TG 21 (this is reason the stem specifies TG51) was based on air kerma. TG 51 is not based on exposure, air kerma or dose to air. References: Kahn, Physics of Radiation Therapy fourth ed., Chapter 4: Measurement of Absorbed dose; AAPM TG 51, 1999.

289. What is the clinical stage of a 4 cm squamous cell carcinoma of the anal canal with FDG avid unilateral pelvic side wall and inguinal lymph nodes? (A) T2N2M0 (B) T2N3M0 (C) T3N2M0 (D) T3N3M0

Key: A Solution: AJCC staging: For anal canal cancer, tumors that are 2-5 cm are T2. For N stage, involvement of the inguinal nodes and internal iliac nodes on the same side is N2. References: AJCC Cancer Staging Manual 7ed. Chicago, IL: Springer; 2010.

113. Which of the following has NOT been shown to increase the risk of oral cancer? (A) Coca tea (B) Betel nut (C) Tobacco (D) Alcohol

Key: A Solution: Alcohol and tobacco are risk factors that increase the incidence of oral cancer. Betal nut chewing has also been shown to cause oral cancer. Coca tea is not suggested to increase oral cancers. The association of some substances, such as marijuana, khat, and mate tea, with oral cancer is controversial. References: Perez and Brady's Principles and Practice of Radiation Oncology, 5th Edition, Chapter 41.

228. What is the estimated risk of LRR (%) at 5 years without adjuvant radiation for a 75 year old with G2 endometrial cancer, < 50% myometrial invasion, and no LVSI or nodes? (A) 10 (B) 15 (C) 20 (D) 25

Key: A Solution: Although favorable pathology the patient still has a risk of local recurrence of 10% at 5 years. The above citation provides a nomogram to assist in estimating the risks based on patient age and pathologic features. VB is estimated to cut the risk to < 5% in these circumstances with minimal risk of toxicity. References: Nomograms for Prediction of Outcome With or Without Adjuvant Radiation Therapy for Patients With Endometrial Cancer: A Pooled Analysis of PORTEC-1 and PORTEC-2 Trials Creutzberg, et al, IJROBP Vol. 91, No. 3, pp. 530e539, 2015.

107. A patient with N3 vulvar cancer has: (A) an ulcerated LN. (B) 1 LN with ECE. (C) 2 LNs with <5 mm of disease. (D) 3 LNs with <5 mm of disease.

Key: A Solution: An ulcerated LN is N3; 1LN with extranodal spread is N2c; 2 LNs with < Smm of disease is N1a; 3 LNs with < Smm of disease is N2a. References: Edge S, Byrd D, Compton C, Fritz A, Greene F, Trotti A. AJCC Cancer Staging Manual. New York: Springer Verlag; 2010.

168. The long-term follow up of the RTOG 9501 demonstrated that the addition of cisplatin to postoperative RT for patients with either positive margin and/or ECE improved 10-year: (A) LC and DFS. (B) LC, but not DFS. (C) DFS and OS. (D) DFS, but only for patients with >5 nodes.

Key: A Solution: At 10 years, the local-regional failure rates were 28.8% vs 22.3% (P=.10), disease-free survival was 19.1% vs 20.1% (P=.25), and overall survival was 27.0% vs 29.1% (P=.31) for patients treated by RT vs RT + CT, respectively. In the unplanned subset analysis limited to patients who had microscopically involved resection margins and/or extracapsular spread of disease, local-regional failure occurred in 33.1% vs 21.0% (P=.02), disease-free survival was 12.3% vs 18.4% (P=.05), and overall survival was 19.6% vs 27.1% (P=.07), respectively. With 46 months follow-up, previous analysis of this prospective randomized trial demonstrated improved local control and disease-free-survival following concurrent cisplatin and radiation therapy in high-risk resected head-and-neck carcinomas. Now with 10 years of follow-up potentially available, only in the unplanned subgroup with involved margins and/or extracapsular extension does the improvement in local control and disease-free-survival remain. Concurrent postoperative therapy remains the standard of care only for the subgroup of patients having involved margins and/or extracapsular spread. References: Cooper et al. Long-term Follow-up of the RTOG 9501/lntergroup Phase III Trial: Postoperative Concurrent Radiation Therapy and Chemotherapy in High-Risk Squamous Cell Carcinoma of the Head and Neck. Int J. Radiat Oncol Physics Biol. 84:1198- 1205, 2012.

86. The EBCTCG overview demonstrated that breast conserving surgery with adjuvant whole breast RT resulted in an absolute 15 year OS improvement of: (A) 5 % (B) 8 % (C) 10 % (D) 15 %

Key: A Solution: BCS and WBI result in a 5% absolute improvement in 15 year overall survival and a relative risk of ipsilateral breast tumor recurrence by approximately 70% at 5 years. References: Clark M, et al. Effects of Radiotherapy and of Differences in The Extent of Surgery for Early Breast Cancer on Local Recurrence and 15-year survival: An overview of the Randomised Trials. Lancet 2005; 366:2087-2106.

38. Which of the following has an increased incidence with the use of bortezomib-based therapy? (A) Herpes zoster (B) Cytomegalovirus (C) Renal failure (D) Hemorrhage

Key: A Solution: Bortezomib-based therapy has an increased incidence of Herpes zoster and prophylactic acyclovir should be administered. References: http://www.nccn.org/professionals/physician_gls/PDF/myeloma.pdf.

253. After brachytherapy for prostate cancer, a high initial International Prostate Symptom Score (IPSS) and enlarged prostate volume are associated with: (A) increased urinary symptoms. (B) increased rectal bleeding. (C) decreased erectile function. (D) decreased biochemical control.

Key: A Solution: Both high pre-implant IPSS and enlarged prostate volume are associated with increased post-implant urinary bother. Patients with these factors are not good candidates for brachytherapy and sometimes they receive few months of androgen ablation before brachytherapy.

94. What is the BEST predictor of toxicity associated with CNS radiosurgery for brain metastases? (A) Tumor diameter over 3 cm (B) Conformity of the high dose radiation (C) Homogeneity index (D) Gradient index

Key: A Solution: CNS radiosurgery planning goals include conformity of the high dose levels to the target. Plans are generally not homogeneous. The best correlate of toxicity is tumor diameter or treatment volume. In RTOG 9005 tumor diameter over 3 cm had a much higher risk of toxicity on multivariate analysis compared to smaller tumors. References: RTOG 90-05. Int J Radiat Oncol Biol Phys. 2000 May 1; 47(2):291-8.

212. Which of the following subtypes of renal cell cancer is associated with a deletion in chromosome 3p and Von Hippel Lindau Disease? (A) Clear cell (B) Papillary (C) Chromophobe (D) Collecting duct

Key: A Solution: Clear cell carcinoma makes up 75-85 percent of renal cell cancers. They typically have a deletion of chromosome 3p and are specifically associated with von Hippel- Lindau disease. References: Presti JC Jr, Rao PH, Chen Q, Reuter VE, Li FP, Fair WR, Jhanwar SC. Histopathological, cytogenetic, and molecular characterization of renal cortical tumors. Cancer Res. 1991 Mar 1;51(5):1544-52.

217. Following concurrent chemoRT and brachytherapy for a cT2bN0M0 cervical cancer, a patient should undergo: (A) observation. (B) hysterectomy. (C) laparoscopic LN sampling. (D) maintenance chemotherapy.

Key: A Solution: Currently, no therapeutic or diagnostic interventions are known to improve outcomes of patients with locally advanced cervical cancer after standard chemoradiation. In addition, total hysterectomy after "full dose" brachytherapy is technically challenging. Yield of node sampling for initially radiographically node-negative patient is low after radiotherapy. Use of consolidative chemotherapy is currently being studied in a randomized North American trial. References: NCCN Clinical Practice Guidelines in Oncology.

242. Which protein is DIRECTLY involved in rejoining double-strand breaks by nonhomologous end joining (NHEJ)? (A) DNA Ligase IV (B) MRE11 (C) FEN1 (D) PARP1

Key: A Solution: DNA Ligase IV complexes with XRCC4 to join the two "broken" ends of a double-strand break. References: Wilson et al. Yeast DNA ligase IV mediates non- homologous DNA end joining. Nature 388(6641): 495-8.

133. A Tata Memorial Hospital trial randomizing patients with clinical T1/2N0 oral cavity squamous cell carcinoma to elective neck dissection versus observation after primary resection concluded that elective neck dissection resulted in: (A) higher rates of DFS and OS. (B) higher rates of DFS, but not OS. (C) higher rates of DFS, but with poorer QoL. (D) decreased OS due to complications.

Key: A Solution: Elective neck dissection resulted in significantly higher rates of disease-free (69.5% vs. 45.9%; P<0.001) and overall survival (80.0% vs. 67.5%; P=0.01). References: D'Cruz AK, Vaish R, Kapre N, et al. Elective versus therapeutic neck dissection in node-negative oral cavity cancer. NEJM 2015. DOI: 10.1056/NEJMoa1506007.

80. Cushing disease with a secretory pituitary adenoma is associated with an elevated level of: (A) cortisol. (B) thyroid stimulating hormone. (C) insulin-like growth factor 1. (D) prolactin.

Key: A Solution: Elevated ACTH and Cortisol are associated with Cushing's Disease.IGF-1 is elevated in Acromegaly. References: Newell-Price Lancet 367, 2006. PMID 16698415.

28. What is the MOST likely explanation for the accelerated repopulation seen in skin after irradiation? (A) Shortened cell cycle time (Tc) (B) Decreased growth fraction (GF) (C) Increased cell loss (D) Increase in the potential doubling time (Tpot)

Key: A Solution: Factors relevant for repopulation, include shortening of Tc, increase of the GF and decreased cell loss. References: Rezvani M, Hopewell JW, Morris GM, Wilding D, Whitehouse E, Robbins ME, Cortina-Borja MJ. Repair, repopulation and cell cycle redistribution in rat foot skin. Radiother Oncol. 1998 Feb; 46(2):193-9. Hopewell JW, Nyman J, Turesson I. Time factor for acute tissue reactions following fractionated irradiation: a balance between repopulation and enhanced radiosensitivity. Int J Radiat Biol. 2003 Jul; 79(7):513-24.

83. What is recommended following gross total resection of a non-functioning pituitary adenoma? (A) Surveillance imaging (B) EBRT to 45 Gy (C) SRS to the entire sella to 15 Gy (D) Somatostatin

Key: A Solution: Following a gross total resection of a non-secretory adenoma, there is no defined role for adjuvant radiotherapy, although some tumors, such as those with cavernous sinus invasion, are at higher risk of recurrence. References: Loeffler J. Clin Endocrinol Metab 96, 2011PMID 21525155.

274. According to QUANTEC data, what is the threshold mean dose in Gy of conventionally fractionated RT for sensorineural hearing loss? (A) No threshold (B) 35 (C) 45 (D) 50

Key: A Solution: For conventionally fractionated RT, to minimize the risk for SNHL, the mean dose to the cochlea should be limited to <45 Gy (or more conservatively <35Gy). Because a threshold for SNHL cannot be determined from the present data, to prevent SNHL the dose to the cochlea should be kept as low as possible. References: Radiation Therapy and Hearing Loss. Int. J. Radiation Oncology Biol. Phys., Vol. 76, No. 3, Supplement, pp. 550-557, 2010

190. Regarding The National Lung Cancer Screening Trial, the rate of: (A) lung cancer-specific mortality was decreased 20% in the low-dose CT group. (B) death from any cause was the same in both groups. (C) adherence to screening was 50%. (D) false positives were lower in the low-dose CT group.

Key: A Solution: From August 2002 through April 2004, The National Lung Screening Trial Research Team enrolled 53,454 persons at high risk for lung cancer, at 33 U.S. medical centers. Participants were randomly assigned to undergo three annual screenings with either low-dose CT (26,722 participants) or single-view posteroanterior chest radiography (26,732). The rate of adherence to screening was more than 90%. The rate of positive screening tests was 24.2% with low-dose CT and 6.9% with radiography over all three rounds. A total of 96.4% of the positive screening results in the low-dose CT group and 94.5% in the radiography group were false positive results. There were 247 deaths from lung cancer per 100,000 person-years in the low-dose CT group and 309 deaths per 100,000 person-years in the radiography group, representing a relative reduction in mortality from lung cancer with low-dose CT screening of 20.0% (95% Cl, 6.8 to 26.7; P = 0.004). The rate of death from any cause was reduced in the low-dose CT group, as compared with the radiography group, by 6.7% (95% Cl, 1.2 to 13.6; P = .02). References: The National Lung Screening Trial Research Team. Reduced lung-cancer mortality with low-dose computed tomographic screening. N Engl J Med 2011; 365:395- 409.

27. What is the most common site of involvement for MALT lymphomas? (A) GI Tract (B) Orbit (C) Lung (D) Skin

Key: A Solution: GI tract is the most common site of involvement (50% of MALT) and within the GI tract, stomach is the most common (80%); Orbit is 7-12%, lung (8-14%), skin (9- 12%). References: Isaacson PG, et al. Lyon: IARC; 2008:214-217. NCCN 2015 guidelines. Nathwani BN, et al. JCO 1999; 17:2486-92. Thieblemont C, et al. JCO 1997; 15:1624-30.

114. Grave's opthalmopathy is an inflammatory condition of which structures? (A) Extra-ocular muscles (B) Retina (C) Optic nerves (D) Cornea

Key: A Solution: Grave's disease is an autoimmune inflammatory disease with involvement of the extra-ocular eye muscles although damage to other structures and vision loss can occur due to mass effect. References: Practical Radiation Oncology (2014) 4, 233-239.

171. What is the most common endocrine dysfunction after cranial irradiation? (A) Growth hormone deficiency (B) Gonadotropin deficiency (C) Thyrotropin deficiency (D) Hyperprolactinemia

Key: A Solution: Growth hormone deficiency can occur after hypothalamic-pituitary doses as low as 18 Gy. Deficiency of gonadotropins and TSH typically occur after > 40 Gy. References: IJROBP 31(5), p.1113-1121, 1995.

71. Why is dose in stereotactic RT prescribed to lower isodoses compared with conventional RT? (A) To improve the dose fall-off outside of the target (B) To reduce the hotspot within the target (C) To allow for re-oxygenation during treatment (D) To improve the accuracy of treatment delivery

Key: A Solution: In an SBRT plan, prescribing to lower isodoses improves the dose fall-off and helps spare neighboring normal tissues. Prescribing to lower isodoses often increases the hotspot within the target. Re-oxygenation during treatment and treatment delivery accuracy are not impacted by the prescription isodose. References: "Stereotactic body radiation therapy: The report of AAPM Task Group 101," S Benedict et al, Medical Physics 37:8 2010.

125. What is the 10-year bladder-intact DFS rate (%) after the treatment of muscle invasive bladder cancer with definitive chemoRT? (A) 40 (B) 55 (C) 70 (D) 85

Key: A Solution: In reported studies the rates vary from 44% or less. RTOG 88-02 reported 4- year rate of bladder preservation to be 44% and the MGH retrospective review reported 5- year rate to be 43%. References: Mak RH, et al. J Clin Oncol. 2014 32(34):3801-9. RTOG 88-02 (Tester W, J Clin Oncol. 1996 Jan; 14(1):119-26). MGH experience (Kachnic LA et al. J Clin Oncol. 1997 Mar; 15(3):1022-9).

77. Assuming no previous pelvic or scrotal surgery, what is the total dose at 2 Gy/fx and lymph node field design for pathologic stage I seminoma (NCCN Guidelines)? (A) 20 Gy; para-aortic (B) 20 Gy; para-aortic and ipsilateral ilioinguinal (C) 30 Gy; para-aortic (D) 30 Gy; para-aortic and ipsilateral ilioinguinal

Key: A Solution: In the absence of prior pelvic surgery (inguinal herniorrhaphy or orchioplexy) the recommended dose for a Stage I Seminoma is 20 Gy in 10 fractions delivered to the paraaortic lymph nodes. Only when there has been prior pelvic surgery, which can alter the normal lymphnode drainage, is coverage of the ipsilateral iliac and inguinal lymphnodes considered. 30 Gy in 2 Gy fractions is no longer recommended by NCCN guidelines, given the results of MRC TE18, demonstrating the equivalence of 20 Gy to 30 Gy in Stage References: National Comprehensive Cancer Network Guidelines: Testicular Cancer, version 2.2015.

238. What rectal wall volume (cc) receiving > 50 Gy was correlated with grade 3 or higher late rectal toxicity in a Phase I-II clinical trial of SBRT for prostate cancer? (A) 3 (B) 8 (C) 13 (D) 18

Key: A Solution: In this Phase 1-11 dose escalation SBRT trial for prostate cancer, more than 35% of the rectal wall circumsference receiving 39 Gy and more than 3cc of rectal wall volume receiving 50 Gy were correlated with Grade 3 or higher late rectal toxicity. References: Kim DW et al, Predictors of rectal tolerance observed in a dose-escalated phase 1-2 trial of stereotactic body radiation therapy for prostate cancer. Int J Radiat Oncol Biol Phys. 2014 Jul 1; 89(3):509-17.

42. A survival analysis estimated that the PFS hazard ratio of males versus females is 1.3. How should this be interpreted? (A) Males have a 30% higher hazard of progression or death compared to females (B) Median PFS time of males is 1.3 months longer than that of females (C) 30% more males died than females (D) 30% of males progressed throughout the study duration

Key: A Solution: In this scenario, the hazard ratio is calculated by dividing the hazard of progression or death among males by that among females. The hazard of progression or death among males was 1.3 times that of females, or 30% higher (option A). The hazard ratio does not lead to inferences about median survival time (option B) or number of progression or death events (options C and D).

180. Which cells are likely to exhibit the earliest reduction (within 24-48 hrs.) in total number of cells after TBI? (A) Lymphocytes (B) Granulocytes (C) Platelets (D) Red blood cells

Key: A Solution: Lymphocytes are one of the most sensitive cell lines to radiation. Doses as little as 0.3 Gy leads to reduction in the number of cells and the changes in these cells take place earlier than granulocytes, platelets, and red blood cells. It takes higher doses to see changes in the number of red blood cells. The other cells are intermediate in terms of the irrelative radiosensitivity. Thus, you see a precipitous drop in lymphocytes immediately after TBI. References: Hall E, Giaccia A. "Radiobiology for the Radiologist". See figure 20.3, page 337.

164. According to NCCN guidelines, what is the preferred induction therapy for CNS lymphoma? (A) Methotrexate (B) Rituximab (C) 18 Gy WBRT (D) 45 Gy WBRT

Key: A Solution: Methotrexate is the most effective agent against primary CNS lymphoma. After a CR to chemotherapy, low dose WBRT to 23.4 Gy may be considered. For patients who are not candidates for chemotherapy, WBRT to 24 to 36 Gy is followed by a boost to 45 Gy. References: www.NCCN.org.

175. Which of the following tumor characteristics is ideal for bladder preservation treatment with chemoRT? (A) Unifocal tumor at the dome (B) Unifocal tumor at the trigone (C) Multifocal tumor at the dome (D) Multifocal tumor at the trigone

Key: A Solution: Multifocal tumors would require boosting the entire bladder which decreases the chances of a functional bladder. Boost dose to the dome of the bladder is tolerated better than that at the trigone. References: RTOG 95-06 (Kaufman DS, Oncologist. 2000; 5(6):471-6).

105. What was observed in the Russell's Mega Mouse project examining the heritable effects of radiation? (A) Reduction in genetic effects with greater time interval between exposure and conception (B) Significant differences for specific- locus mutation frequencies with age subgroups (C) Temporary sterility in males at doses above 6 Gy (D) No correlation between radiation dose rate and the number of mutations per locus

Key: A Solution: Mutations were found to increase linearly with dose and dose rate. Males were more radiosensitive than females. Temporary sterility in males was seen with doses significantly less than 1Gy. The longer the period between radiation exposure and conception, the less the genetic effects on the offspring. References: Russell WL and Kelley EM. Mutation frequencies in male mice and the estimation of genetic hazards of radiation in men. PNAS. 1982:79(2):542-544. Russell WL and Kelley EM. Specific-locus mutation frequencies in mouse stem-cell spermatogonia at very low radiation dose rates. PNAS 1982. 79(2) 539-41. Russell, W. L. (1977) Proc. Nati Acad. Sci. USA 74, 3523-3527.

247. What mechanism of cell death is characterized by a gain in cell volume, swelling of organelles, membrane rupture and the resulting loss of cellular contents? (A) Necrosis (B) Apoptosis (C) Autophagy (D) Senescence

Key: A Solution: Necrosis is thought to come about through the inability of a cell to maintain ionic homeostasis. This results in organelle and membrane swelling and subsequent rupture. The release of cellular contents can result in an immune response characterized by inflammation; a state which is avoided by apoptosis and more regulated cell death mechanisms. Autophagy typically involves the isolation of unwanted or damaged cellular constituents within a double-membraned "auto phagosome". These are eventually fused to lysosomes and their contents are degraded making them available for other uses. A form of programed cell death is characterized by the appearance of auto phagosomes, but it is unclear whether these bodies are engaged in the death process or whether they result from some last ditch attempt of the cell to survive. Apoptosis is a well-studied programed cell death process that digests important cellular constituents such as genomic DNA while the cellular membrane fragments into membrane bound vesicles referred to as "apoptotic bodies" containing most cellular constituents. These vesicles can by phagocytized by surrounding cells thus preventing an immune response. Senescent cells are permanently blocked in a resting state of the cell cycle. They are reproductively "dead" while remaining metabolically active. References: Kroemer G, and Levine B. Autophagic cell death: the story of a misnomer. Nature Rev. Mol Cell Biol. doi: 10.1038/nrm2529 (2008). Ohtani, N. et al. Cellular senescence: Its role in tumor suppression and aging. Cancer Sci. 100:792-797 (2009).

10. A randomized trial testing the efficacy of palifermin for patients receiving conventional radiation and concurrent chemotherapy for stage III/IV head and neck cancer showed palifermin significantly decreased the: (A) severity of mucositis. (B) treatment breaks. (C) use of opioids. (D) need of supplemental nutrition.

Key: A Solution: Palifermin, a recombinant human keratinocyte growth factor, was tested to decrease oral mucositis for patients receiving concurrent chemoradiation for stage III/IV head and neck cancer. While palifermin was effective in decreasing severity of oral mucositis, that did not translate into fewer treatment breaks or less use of opioid analgesic or supplemental nutrition. We await long-term efficacy and safety data to confirm that this growth factor does not result in worse tumor control. References: Le, et al. Palifermin Reduces Severe Mucositis in Definitive Chemoradiotherapy of Locally Advanced Head and Neck Cancer: A Randomized, Placebo- Controlled Study JCO 2011, 29:2808-2814.

129. What is the appropriate management for a patient with a history of metastatic NSCLC who presents with a 2 week history of bilateral lower extremity weakness and incontinence of bowel with imaging demonstrating spinal cord compression at T8? (A) RT alone to 30 Gy in 2 weeks (B) Transversectomy and postoperative RT (C) Second line chemotherapy (D) SBRT to 20 Gy in 1 fraction

Key: A Solution: Palliative radiation for pain management is more appropriate in this case since the duration of the symptoms does not justify using a surgical decompression approach as per the Patchell trial. Patients who presents with symptoms within 48 hours may be helped with surgical decompression and postoperative radiotherapy. SBRT will not be appropriate for tumor mass compressing directly on the spinal cord. Chemotherapy will not address the cord compression or the pain. References: Patchell et al. Lancet 2005.

144. Per ASTRO Evidence-Based Guidelines (2014), patients with endometrioid endometrial cancer may be observed after hysterectomy with which pathologic features? (A) G1 or G2 and < 50% myometrial invasion (B) G2 and < 50% myometrial invasion and LVSI (C) G2 and > 50% myometrial invasion and age > 60 (D) G3 and > 50% myometrial invasion and extension to cervical stroma

Key: A Solution: Patients with grade 1 or 2 cancers with either no invasion or <50% myometrial invasion (Ml), especially when no other high risk features are present, can be safely observed after hysterectomy. Vaginal cuff brachytherapy is as effective as pelvic radiation therapy at preventing vaginal recurrence for patients with grade 1or 2 cancers with 50% Ml or grade 3 tumors with <50% Ml. Patients with grade 3 cancer with 50% Ml or cervical stroma invasion may benefit from pelvic radiation to reduce the risk of pelvic recurrence. References: Klopp, et al. Practical Radiation Oncology (2014): The Role of Postoperative Radiation Therapy for Endometrial Cancer: An ASTRO Evidence-Based Guideline.

179. What is your recommendation for a pT1b2 N0 adenocarcinoma of the cervix with negative margins and + LVSI? (A) EBRT alone (B) Brachytherapy alone (C) Chemotherapy alone (D) ChemoRT

Key: A Solution: Patients with intermediate risk for recurrence after radical hysterectomy (at least two of the following risk factors: >1/3 stromal invasion, LVSI, and palpable tumor diameter >4cm) benefit from postoperative external beam radiotherapy. The GOG 92 study randomized 237 such patients to observation vs. pelvic radiotherapy to 46 or 50.4 Gy. Radiotherapy was associated with statistically significant reduction in any recurrence and with a trend towards improved overall survival. While concomitant radio sensitizing chemotherapy can be also considered with this regimen, a randomized study of chemotherapy is still ongoing. References: Rotman, et al. A phase III randomized trial of postoperative pelvic irradiation in Stage IB cervical carcinoma with poor prognostic features: follow-up of a Gynecologic Oncology Group Study (2006).

145. Which CSF tumor marker levels are consistent with non-metastatic CNS pure germinoma? (A) (B) AFP 5, (C) (D)

Key: A Solution: Patients with pure germinomas do not have elevated alpha-feto protein levels, whereas non-germinomatous germ cell tumor can (but are not required to) have elevated alpha-feta protein. Beta-HCG levels can be elevated in pure germinomas, but beta-HCG levels within normal limits are sometimes found in pure gerinomas. Markedly elevated beta-HCG levels (cutoff levels vary between 50 and 100) are considered consistent with components of choriocarcinoma and/or mixed histology germ cell tumors. The distinction between pure germinomas and other germinomas is important for determining the intensity of chemotherapy and radiotherapy, with pure germinomas generally treated with less intense regimens. References: Halperin, et al, Pediatric Radiation Oncology, 5th edition, Chapter 3.

279. For a resectable cancer of the gastric antrum, which of the following treatment strategies is NOT currently supported by published results from a randomized clinical trial? (A) Preoperative chemoRT (B) Postoperative chemoRT (C) Peri-operative chemotherapy (D) Postoperative chemotherapy

Key: A Solution: Preoperative chemoradiation for resectable gastric cancer has not yet been validated by a Phase III randomized clinical trial. The TOPGEAR trial is ongoing and comparing preoperative chemoradiotherapy vs. preoperative chemotherapy alone. The other adjuvant treatment strategies are supported by published randomized clinical trials as listed above. Note that the CROSS trial, among other esophageal cancer trials of preoperative chemoradiation, did include some gastroesophageal junction cancers. References: Smalley SR, Benedetti JK, Haller DG, et al. Updated analysis of SWOGdirected intergroup study 0116: a phase 111 trial of adjuvant radiochemotherapy versus observation after curative gastric cancer resection. J Clin Oncol. 2012 Jul 1;30(19):2327- 33. Cunningham D, Allum WH, Stenning SP, et al. Perioperative chemotherapy versus surgery alone for resectable gastroesophageal cancer. N Engl J Med. 2006 Jul 6; 355(1):11-20. Noh SH, Park SR, Yang HK, et al. Adjuvant capecitabine plus oxaliplatin for gastric cancer after 02 gastrectomy (CLASSIC}: 5-year follow-up of an open-label, randomised phase 3 trial. Lancet Oneal. 2014 Nov; 15(12):1389-96.curative gastric cancer resection. J Clin Oncol. 2012 Jul 1; 30(19):2327-33.

298. Which characteristic of a pancreatic cancer would make it unresectable? (A) Encasing the superior mesenteric artery (B) Gross peri-pancreatic lymph node involvement (C) Invasion of the duodenum (D) Abutment of the portal vein without vein contour irregularity

Key: A Solution: Primary tumor encasing the SMA is not considered resectable based on consensus guidelines. Lymph node involvement and invasion of the duodenum are not contraindications to resection. Abutment of the portal vein (<180 degrees) without vein contour irregularity is not considered a contraindication to resection. References: http://www.nccn.org/professionaIs/physician_gls/pdfIpancreatic.pdf

46. How is ATM activated as a response to radiation-induced DNA double strands? (A) Auto-phosphorylation converting inactive dimer to an active monomer (B) By association of Ku70 and Ku80 at site of DNA damage (C) By dissociation of the DNA-dependent protein kinase (DNA-PKcs) (D) Following the binding of the LIG4-XRCC4 complex

Key: A Solution: Radiation induced DNA double strand breaks (DSBs) are regarded as a critical lesion in radiation mediated cell killing; failure to repair DNA DSBs leads to produce of chromosome aberrations and death by mitotic catastrophe. ATM activation is a #Key step in responding to radiation-induced DNA DSBs. References: Bakkenist CJ, Kastan MB. DNA damage activates ATM through intermolecular autophosphorylation and dimer dissociation. Nature 2003 Jan 30; 421(6922):499-506. Paull TT. Mechanisms of ATM Activation. Annu Rev Biochem. 2015; 84:711-38.

139. A front-pointer is used in linac quality assurance to: (A) measure SSD. (B) test the linac gantry rotation axis. (C) indicate the toe of a wedge. (D) point toward the source of the x-rays.

Key: A Solution: Rationale is not given for this question. References: The Physics and Technology of Radiation Therapy" McDermott & Orton, pg. 18-8.

194. What is the sensitivity of a new test with 500 patients, 100 of whom are known to have the disease, with results positive in 50 of the 100 patients with the disease, as well as in 20 patients who do not have the disease? (A) 50% (B) 95% (C) 71.4% (D) 88.4%

Key: A Solution: Screening refers to the application of test to people who as yet have no symptoms of a particular disease. It is classified as having a positive (disease likely) or negative (disease unlikely) finding. Diagnostic tests tell whether or not a subject actually has the disease. The performance of a screen test is considered by the sensitivity and specificity. The sensitivity is the percentage of subjects with disease who are classified as having disease and the specificity is the percentage of subjects without disease who are classified as not having disease. Those subjects with the disease should all be classified as having disease, and those subjects without the disease should be classified as not having disease. Therefore a highly sensitive and specific test is preferred. References: Chapter 11 in Mausner /Bahn, Epidemiology: An Introductory Text, 1974.

189. Why do senescent cells survive better than actively proliferating cells after irradiation? (A) Repair DNA damage more readily (B) Have less DNA damage from radiation (C) Have more DNA damage from radiation (D) Use less energy for repair

Key: A Solution: Senescent cells can resolve more DNA damage which allow them to survive. References: Fumagalli, et al. Nature Cell Biology14, 355-365 (2012).

112. What is the appropriate stage for a 41-year old female who underwent thyroidectomy and lymph node dissection for a papillary carcinoma invading the larynx with 5 positive pretracheal nodes? (A) Stage I, T4aN1aM0 (B) Stage II, T2N1bM0 (C) Stage III, T3N1bM0 (D) Stage IV, T4aN1aM0

Key: A Solution: Staging for thyroid papillary carcinoma also depends on patient's age. Stage I in 41-year-old includes any T, any N, M0. Larynx involvement makes T4a, and pretracheal nodes are N1a disease. References: AJCC Cancer Staging Manual, 7th edition.

197. Which multi-targeted receptor tyrosine kinase (RTK) inhibitor is a first line agent in the management of unresectable clear cell carcinoma of the kidney? (A) Sunitinib (B) Bevacizumab (C) lpilimumab (D) Denosumab

Key: A Solution: Sunitinib is a multi-targeted receptor tyrosine kinase used in the treatment of metastaticunresectable metastatic Clear Cell Carcinoma of the Kidney. Bevacizumab is a monoclonal antibody that blocks angiogenesis by inhibiting vascular endothelial growth factor A also used in the treatment of metastatic kidney cancer. lpilimumab is a monoclonal antibody that works to activate the immune system by targeting CTLA-4, a protein receptor that downregulates the immune system, and is approved for the use in metastatic melanoma. Denosumab is a monoclonal antibody to RANKL used to help retard bone loss in patients with bone metastasis in renal, as well as other malignancies.

148. How do the surface dose, depth of penetration and X-ray contamination change, respectively, as the electron energy increases from 6 MeV to 20 MeV? (A) All three parameters increase (B) Decreases, increases, and increases (C) Remains the same, increases, and increases (D) Increases, increases, and remains the same

Key: A Solution: The PDD of an electron beam changes according to option A when the beam energy increases. These changes are important to take into account when considering with which energy to treat. References: F. Khan. Chapter 14 of The Physics of Radiation Therapy, 2009.

266. What would reduce the incidence of myelopathy for the treatment of a pediatric patient with an ependymoma arising in the low posterior fossa? (A) Limiting the dose per fraction (B) The inclusion of concurrent chemotherapy (C) Limiting dose exposure to the surface of the spinal cord while allowing higher doses to the center of the spinal cord (D) Use of hypo-fractionated approaches such as stereotactic radiosurgery

Key: A Solution: The correct answer is A. For pediatric patients with ependymoma the proximity of the target volume frequently leads to spinal cord doses in excess of 5O Gy. There is evidence that the cervical spinal cord is more radiation resistant than the thoracic cord and as such can tolerate higher doses. Likewise, the surface of the spinal cord, which is well vascularized, is likely more radiation resistant than the center of the cord. Chemotherapy has no proven role in the treatment of such patients and concurrent chemotherapy would potentially increase toxicity rates. Stereotactic radiosurgery is not routinely used in the primary treatment of these patients and hypo-fractionated techniques could increase the dose per fraction to the spinal cord. On the current Children's Oncology Group trial, where most patients will receive 59.4Gy, the allowable fraction sizes to the cervical spinal cord are limited during the boost in order to reduce the risk of myelopathy.

101. Patients with an occult primary head and neck cancer treated with neck dissection WITHOUT adjuvant radiation are most likely to recur: (A) in the dissected neck. (B) in the undissected neck. (C) in the lungs. (D) at a previously unidentified primary site.

Key: A Solution: The dissected neck is the most common site of recurrence without adjuvant radiation. In the cited study about 25% of the patients recurred in the dissected neck. Distant metastatic recurrence, undissected neck recurrence, and mucosal recurrence occurred in less than 10% of patients each. References: Coster JR, et al. Cervical nodal metastasis of squamous cell carcinoma of unknown origin: indications for withholding radiation therapy. Int J Radiat Oncol Biol Phys. 1992; 23(4):743-9.

55. What minimal cumulative dose (Gy) of radiation would cause premature ovarian failure in a 30 year old? (A) 14 (B) 16 (C) 18 (D) 20

Key: A Solution: The effective sterilizing dose (ESD: dose of fractionated radiotherapy [Gy]at which premature ovarian failure occurs immediately after treatment in 97.5% of patients) decreases with increasing age at treatment. ESD at birth is 20.3 Gy; at 10 years 18.4 Gy, at 20 years 16.5 Gy, and at 30 years 14.3 Gy. Clinical application of this model will enable physicians to counsel women on their reproductive potential following successful treatment. References: Wallace WH, Thomson AB, Saran F, Kelsey TW. Predicting age of ovarian failure after radiation to a field that includes the ovaries. IJROBP 2005; 62(3):738.

151. The international prognostic index (IPI) for non-Hodgkin's lymphoma includes which of the following patient criteria? (A) Age (B) Fever (C) Weight (D) Pruritus

Key: A Solution: The international prognostic index (IPI) for non-Hodgkin's lymphoma includes lactic dehydrogenase (LDH) level, involvement of extranodal sites, age, performance status (ECOG), and cancer stage.

208. What additional treatment should be provided to a patient with hepatitis B related cirrhosis and HCC treated with SBRT? (A) Antiviral therapy for hepatitis B (B) Topical lidocaine (C) Prednisone (D) Sorafenib

Key: A Solution: The risk of liver toxicity is minimal with such a low liver mean dose. Similarly, the risk of esophageal toxicity is also very low. There is no role currently for adjuvant sorafenib following liver directed therapy for non-metastatic solitary hepatocellular carcinoma. Data suggests that reactivation of hepatitis B occurs after liver SBRT. Therefore HBV carriers should start on antiviral therapy prior to beginning treatment. References: Kim JH, Park JW, Kim TH, Koh DW, Lee WJ, Kim CM. Hepatitis B Virus Reactivation After Three-Dimensional Conformal Radiotherapy in Patients With Hepatitis B Virus-Related Hepatocellular Carcinoma. Int J Radiat Oncol Biol Phys. 2007 Nov 1; 69(3):813-9.

153. What is the expected nodal recurrence rate after a negative sentinel node biopsy in women with early stage breast cancer? (A) <1% (B) 3 - 5% (C) 7 - 10% (D) 12 - 15%

Key: A Solution: The risk of nodal recurrence is less than 1% in patient with negative clinical nodes and no nodes involved on sentinel node biopsy. References: Van der Ploerg IM, Nieweg OE, van Rijk MC, Valdes Olmos RA, Kroon BB. Axillary Recurrence After a Tumour Negative Sentinel Node Biopsy in Breast Cancer Patients: A Systemic Review and Meta-analysis of the Literature. Eur J Surg Oncol 2008; 34:1277-1284. Burstein H, Morrow M. Nodal Irradiation after breast cancer surgery in the era of effective adjuvant therapy. N Engl J Med 2015; 373:379-381 July 23, 2015.

202. What are the biological rationales for TBI when used as part of the preparative regimen for an allogeneic hematopoietic stem cell transplantation? (A) Immunosuppression, malignant cell killing, treatment of sanctuary sites (B) Graft versus host effect, immunosuppression, malignant cell killing (C) Graft versus host effect, malignant cell killing, treatment of sanctuary sites (D) Graft versus tumor effect, immunosuppression, malignant cell killing, treatment of sanctuary sites

Key: A Solution: The term graft versus host effect refers to an undesired complication of an allogeneic transplant, not a rational e for TBI. Graft versus tumor effect is a desired consequence of the allogeneic stem celltransplantation, not a consequence of preparative TBI. TBI, alongside conditioning chemotherapy, immunosuppresses the host for acceptance of the graft. TBI can have a direct malignant cell killing effect. TBI provides treatment of sanctuary sites, such as brain and testes, that may not be optimally addressed with systemic agents. References: Hill-Kayser CE, et al. Bone Marrow Tranplant. 2001 Apr; 46(4):475-84. 2001

287. What is the approximate initial dose rate (Gy/hr) required to deliver 160 Gy to the prostate using an 125I permanent seed implant? (A) 0.08 (B) 0.8 (C) 8 (D) 80

Key: A Solution: The total dose of a permanent seed implant is estimated by Dtotal=Do, average x 1.44t 1/2. The half-life of 1251 is 59.4 days. Do, average = 160 Gy/(1.44 X 59.4 dx 24 hr/d) = 0.0779 Gy/hr. This (reasonably) requires knowledge of average prescription dose (Do) and the half-life of I-125. References: Khan. Physics of Radiation Therapy second ed., Chapter 15: Brachtherapy.

47. What is the estimated risk (%) of LRR at 10 years following radiotherapy for stage I testicular seminoma? (A) 1 - 5 (B) 6 - 10 (C) 11 - 15 (D) 16 - 20

Key: A Solution: Though declining in use, radiotherapy is considered highly effective in reducing relapse risk for early stage disease. The decline in use is attributable to increased observation as well as the documented late risk for complications including secondary malignancy which this same group has published may be as high at 16% at 30 years for GI malignancies. Ural Oncol 2014 May 32(4): pp 496-500. References: Hallemeier CL, et al. Excellent Long-term Disease Control with Modern Radiotherapy Techniques for Stage I Testicular Seminoma-The Mayo Clinic Experience. Urol. Oncol. 2014 Jan; 32(1):24.1-6.

240. In subset analysis of the CROSS trial for esophageal cancer, which histology derived benefit from the addition of preoperative chemoradiation to surgery? (A) Both squamous cell carcinoma and adenocarcinoma (B) Squamous cell carcinoma only (C) Adenocarcinoma only (D) Neither squamous cell nor adenocarcinoma

Key: A Solution: Though the rate of pathologic complete response was higher among patients with squamous cell carcinoma, both adenocarcinoma and squamous cell patients benefitted from neoadjuvant chemoradiation. References: van Hagen P, Huishof MC, van Lanschot JJ, et al. Preoperative chemoradiotherapy for esophageal or junction a I cancer. N Engl J Med. 2012 May31; 366(22):2074-84.

92. In Acute Lymphocytic Leukemia, which of the following is an indication to irradiate the CNS? (A) Cranial nerve involvement (B) Loss of consciousness (C) Vomiting (D) Proptosis

Key: A Solution: While all patients with ALL should have CNS prophylaxis, cranial nerve involvement is an indication for immediate irradiation. References: NCCN ALL Guidelines, 1.2015, p. 18.

214. Which of the following is NOT a recommendation for treating a patient with an implanted cardiac defibrillator? (A) Use an 18 MV beam to take advantage of the larger buildup region (B) Avoid treatment beam directions that either enter or exit through the device (C) Avoid the device when imaging for IGRT (D) Estimate the dose to the device and verify using in-vivo dosimetry

Key: A Solution: While the 18MV beam does have a deeper depth of maximum dose, that beam more importantly also creates neutrons that have been shown to cause possible interactions with a pacemaker and causing a malfunction. Therefore, no beams with energy more than 10 MV beams should be used. References: Gelblum DY, and Amols H. "Implanted cardiac defibrillator care in radiation oncology patient population." International Journal of Radiation Oncology, Biology, Physics 73.5 (2009): 1525-1531.

124. Which feature is a FAVORABLE prognostic factor in neuroblastoma? (A) Age of 8 months (B) Diploid tumor (C) MYCN amplification (D) INSS stage 4

Key: A Solution: Younger patients have a better prognosis than older patients. All of the other features are associated with a worse prognosis. References: Halperin, et al. Pediatric Radiation Oncology 2011.

248. What is the statistical test for comparing two variances commonly used with ANOVA? (A) F-test (B) t-test (C) Chi-square test (D) Wilcoxon rank sum test

Key: A Solution: t-test: compares a mean with a norm or compares two means with small sample sizes. Also tests whether a correlation coefficient or a regression coefficient is zero; Chi-square test: used to test the null hypothesis and proportions are equal or, equivalently, that factors or characteristics are independent or not associated; Wilcoxon rank sum test: A nonparametric test for comparing two independent samples with ordinal data or with numerical observations that are not normally distributed. References: Dawson B, and Trapp RG. Basic and Clinical Biostatistics. 4th edition, 2004; pp. 406.

173. What molecular feature occurs in anaplastic large cell lymphoma of childhood? (A) 100% are CD30 positive (B) 50% are CD30 positive (C) 50% have an ALK gene rearrangement (D) 5% have an ALK gene rearrangement

Key: A Solution:Ana plastic large cell lymphoma accounts for approximately 10% of childhood NHL. Most (>90%) pediatric ALCLs have a chromosomal rearrangement involving the ALK gene. All cases are CD30 positive. References: http://dx.doi.org/10.1016/j.pcl.2014.09.010. Pediatr Clin N Am 62 (2015) 139-165.

299. If the same current used to generate a photon beam was used in error to generate an electron beam, the dose to the patient would be approximately: (A) 100 times higher than expected. (B) 100 times lower than expected. (C) 10 times higher than expected. (D) 10 times lower than expected.

Key: A Solution:The current used to generate an electron beam is on the order of 1mA. Due to bremsstrahlung photon production inefficiency, however, the several hundred mA of current is needed to generate a photon beam. A photon current used to generate an electron beam would result into approximately 100 times higher dose. References: Greene D. Chapter 2 of Linear Accelerators for Radiation Therapy, 1997.

103. What would you recommend for a patient with uterine papillary serous carcinoma pathologic stage IA with no myometrial invasion? (A) Vaginal brachytherapy (B) Extended field IMRT (C) Chemotherapy with pelvic IMRT (D) Chemotherapy with extended field IMRT

Key: A & C NOTE: This item was multi-#Keyed for scoring purposes upon post-exam statistical item analysis (Both A and C were given credit as correct responses). Solution: The NCCN guidelines list a wide spectrum of management options for this patient population including observation, chemotherapy with or without vaginal brachytherapy, or tumor directed radiation. The role of adjuvant therapy in this population in evolution. Historic studies lumped the findings of Stage IA no myometrial invasion with those with myometrial invasion and some even included those with Stage II. In the multiinstitutional study chemotherapy was not associated with significant improvement in PFS References: Mahdi, et al. Adjuvant vaginal brachytherapy decreases the risk of vaginal recurrence in patients with stage I non-invasive uterine papillary serous carcinoma. A multi-institutional study. Gynecologic Oncology 136(2015) 529-533. NCCN guidelines Version 2.2015.

166. According to QUANTEC, what is the risk of optic neuropathy if the optic nerve Dmax is 12 Gy in SRS? (A) <1% (B) 5% (C) 10% (D) 20%

Key: A and B accepted as correct responses (less than 10%). Solution: QUANTEC table References: www.ncbi.nlm.nih.gov/pubmed/20171502?dopt=Abstract.

13. What is the approximate rate (%) of organ preservation for early stage invasive squamous cell carcinoma of the penis treated with brachytherapy? (A) 65 (B) 75 (C) 85 (D) 95

Key: B NOTE: This item was 0-weighted for scoring purposes upon post-exam statistical item analysis (did not count for or against candidate in calculation of test scores). Solution: In properly selected case brachytherapy is very effective at organ preservation. For Stage 1-11 patient's surgery vs brachytherapy produced overall survival rates and local control rates of 80% and 86% for penectomy vs. 79% and 84% for brachytherapy. Though brachytherapy training and use appears to be declining, it is a highly effective means for maintaining organ preservation in multiple sites, including penile cancer. References: Hasan S, et al. The Role of Brachytherapy in Organ Preservation for Penile Cancer: A Meta-analysis and Review of the Literature. Brachytherapy 2015 Jul-Aug 14(4) pp 517-524.

269. By which process are X-ray photons absorbed at high energies, characteristic of a linac? (A) Photoelectric process (B) Compton process (C) Fluorescence radiation (D) Pair production

Key: B Solution: References: Hall and Giacca. Radiobiology for the Radiologist, 7th Edition. Chapter 1.

39. Which of the following is one of the criteria used to define Active (symptomatic) Myeloma? (A) Normal calcium level (B) Renal insufficiency (Cr > 2 mg/dL) (C) Hyponatremia (D) Hypovolemia

Key: B Solution: "CRAB" criteria use to determine symptomatic myeloma: 7C-Elevated Calcium, R-Renal Insufficiency, A- Anemia, Bone-lytic/osteopenic lesions. References: Durie BG, et al. Leukemia. 2006; 20(9)1467-73; International uniform response criteria for multiple myeloma.

36. What is the correct radiation dose (Gy) for early stage (stage IA/IIA) follicular lymphoma per the International Lymphoma Radiation Oncology Group (ILROG) NHL consensus guidelines? (A) 4 (B) 24 (C) 36 (D) 45

Key: B Solution: "For potentially curative treatment of localized early stage (I and II) disease, RT is used as the primary treatment approach .... For potentially curative RT to stage IA/IIA, a dose of 24-30 Gy in 12-15 fractions is recommended". References: lllidge T, et al. IJROBP. 2014. Vol 89(1)49-58.

206. Which of the following is a purpose of a meta-analysis? (A) Increase the statistical power by decreasing sample size (B) Resolve uncertainty when reports do not agree (C) Identify risk factors for individual patients (D) Predict outcomes for individual patients

Key: B Solution: #Key purposes of meta analyses, include: 1) to increase statistical power by increasing sample size,2) resolve uncertainty when reports do not agree,3) to improve estimates of effect size, 4) to answer questions posted at the beginning of a study. References: Dawson-Saunders and Trapp. Basic & Clinical Biostatistics. 2nd Ed. P224-6.

213. What is the clinical stage for a patient with a 5 cm primary breast mass, palpable ipsilateral axillary and supraclavicular nodes, and no evidence of distant metastatic disease? (A) Stage IIIA cT2N2M0 (B) Stage IIIC cT2N3M0 (C) Stage IIIA cT3N2M0 (D) Stage IIIC cT3N3M

Key: B Solution: 5 cm breast mass is T2, supraclavicular nodal involvement is N3, stage IIIC. References: AJCC staging manual, 7th edition.

221. Assuming platelet values follow a normal distribution, in what range would 99% of normal values fall if the mean platelet count is 270,000 cells/mm3 and with a SD of 40,000 cell/mm3? (A) 110,000 - 440,000 (B) 150,000 - 390,000 (C) 190,000 - 350,000 (D) 239,000 - 310,000

Key: B Solution: 99% of subjects fall within ± 3 standard deviations of the mean.

192. Which of the following strategies is NOT used to control for confounding in a retrospective study? (A) Matching (B) Case-control design (C) Stratified analysis (D) Multivariable regression

Key: B Solution: A case-control design does not prevent or minimize confounding of the outcome and treatment variables by one or more of the other variables. Matching at the patient or stratum level, strata-adjusted analysis and regression modeling are standard methods to control for confounding.

235. What technique is used to match the inferior border of the cranial field with the divergent superior border of the posterior spinal field when performing craniospinal irradiation? (A) Couch rotation (B) Collimator rotation (C) Spinal gap calculation (D) Half beam block

Key: B Solution: A collimator rotation is needed for these divergent fields. A couch rotation is used to match the divergent inferior border of the cranial fields into the spinal fields (note: the Parker supine technique does not require a couch kick, as the half beam block creates a non-divergent inferior border of the cranial field). A spinal gap calculation is to prevent overlap of 2 matched spinal fields. A half beam block can be used to eliminate divergence along the central axis, but a collimator rotation is always needed to match the cranial field to the superior spinal field. References: Radiother Oncol. 2006 Feb; 78(2):217-22.

276. Which of the following patients would be considered low risk for LRR and not require PMRT after neoadjuvant chemotherapy followed by mastectomy? (A) Age 33, IIB cT3N0, ypN1 (B) Age 40, IIA cT2N0, ypCR (C) Age 52, IIIA cT2N2, ypCR (D) Age 60, IIB cT2N1, ypT1N2

Key: B Solution: A panel of breast cancer specialists through the Univ of California system sought to define a group of patient treated with neoadjuvant chemotherapy (NAC) followed by mastectomy at "low risk" for loco regional failure (LRF <10%) in whom post mastectomy radiation could be safely omitted. Variables for LRF included clinical stage and age at diagnosis, extent of residual disease after NAC, and adverse risk factors such as LVI, ECE, and triple- negative disease. This panel defined patients with cT1-2N0-1 tumors who achieved a pCR, were ypNO, or who had ypN1 disease and ER positive disease, aged >40 years with no LVI or ECE as low risk. References: Fowble, et al. IJROBP 83(2) 2012.

24. Which of the following side effects of treatment is MOST commonly associated with TBI for hematopoietic stem cell transplantation? (A) Skin desquamation (B) Acute parotitis (C) Radiation pneumonitis (D) Radiation-induced second malignancies

Key: B Solution: Acute parotitis can occur in up to 74% of patients. The dose of radiation is insufficient to result in significant skin desquamation. Radiation pneumonitis and radiationinduced second malignancies are less uncommon. References: Chaillet MP, et al. Health Phys. 1993 Apr; 64(4):370. Ringden O, et al. Blood 1999 Apr 1; 93(7):2196-201.

50. On the RTOG 9704 study of adjuvant treatment for resected pancreatic cancer, among patients with relapses, what percentage was local? (A) 18 (B) 28 (C) 38 (D) 48

Key: B Solution: Although this was a phase III study comparing adjuvant chemotherapy with gemcitabine vs 5-fluorouracil, distribution of relapse was similar among all patients. Local relapse occurred in 30% of patients on the 5-FU arm and 25% on the gemcitabine arm (overall 28%). Regional relapse was similar on both arms at 8% vs. 7%. Distant relapse was >70% in both arms. References: Regine WF, Winter KA, Abrams R, et al. "Fluorouracil-based Chemoradiation with Either Gemcitabine or Fluorouracil Chemotherapy after Resection of Pancreatic Adenocarcinoma: 5-year Analysis of the U.S. lntergroup/RTOG 9704 Phase Ill Trial." Ann Surg Oncol (2011) 18:1319-26.

237. How do the anti-apoptotic BCL-2 subfamily proteins block apoptosis? (A) They activate caspases (B) They prevent activation of BAX-like BCL-2 proteins (C) They inhibit the actions of IAP proteins (D) They prevent MDM2 activation of p53

Key: B Solution: Apoptosis is tightly regulated by the BCL-2 family of proteins. The BAX-like members (including BAX, BAK and BOK) of the family form pores in the mitochondrial outer membrane when activated. This facilitates the release of Cytochrome c from the organelle which can then participate in Caspase activation. The anti-apoptotic BCL-2 subfamily (including BCL-2, BCL-XL and BCL-W) block the activation of the BAX-like members by BCL-2 proteins from the BH3 Only Subfamily (i.e. they lack the BH1, BH2 and BH4 domains; including PUMA, BID and BAD). IAP (Inhibitor of Apoptosis Proteins) directly prevent the activation of caspases. Caspases are the workhorses of apoptosis and cleave a host of cellular substrates including DNA and other caspases. These need to be activated for apoptosis to proceed. MDM2 blocks the activation of p53. References: Belka, C, and Budach W. Anti-apoptotic proteins: structure, function and relevance for radiation biology. Int. J. Radiat. Biol. 78: 643-658 (2002). Taylor RC, et al. Apoptosis: controlled demolition at the cellular level. Nature Rev. Mal. Cell Biol. 9: 231- 241 (2008). Fulda S, and Vucic D. Targeting IAP proteins for therapeutic intervention in cancer. Nature Rev Drug Disc. 11: 109-125 (2012).

102. RTOG 9003, a randomized trial to compare hyperfractionation (HFX) and two variants of accelerated fractionation to standard fractionation RT for head and neck cancer, demonstrated HFX increased: (A) 5-year locoregional control, but not OS. (B) both 5-year locoregional control and OS. (C) grade 3 or higher toxicity. (D) need for prolonged feeding tube use.

Key: B Solution: At 5 years, only HFX improved LRC and overall survival for patients with locally advanced sec without increasing late toxicity. With patients censored for LRC at 5 years, only the comparison of HFX with SFX was significantly different: HFX, hazard ratio (HR) 0.79 (95% confidence interval 0.62-1.00), P=.05; AFX-C, 0.82 (95% confidence interval0.65-1.05), P=.11. With patients censored at 5 years, HFX improved overall survival (HR 0.81, P=.05). References: Beitler J, et al. Final Results of Local-Regional Control and Late Toxicity of RTOG 9003: A Randomized Trial of Altered Fractionation Radiation for Locally Advanced Head and Neck Cancer. Int. J of Radial Oncol Physic and Biol. 89, 12-20, 2014.

200. Which of the following beam parameters are explicitly included in TMR ratios, but not PDDs? (A) Heterogeneities (B) Distance from source (C) Field size (D) Energy

Key: B Solution: Because the PDD takes account of both inverse square effect and attenuation by matter, the use of PDD for the given radiological depth is incorrect. Choice A, C, and D are false statements. References: The Physics and Technology of Radiation Therapy'' McDermott & Orton, pg. 14-54.

286. What is the rationale for inclusion of concurrent bevacizumab for the re-irradiation of recurrent glioblastoma? (A) Proven benefit to OS in newly diagnosed glioblastoma (B) Has been shown effective in the treatment of radiation necrosis (C) Alters MGMT methylation status (D) Induces a transition from a mesenchymal to pro-neuronal phenotype

Key: B Solution: Bevacizumab has not been shown to improve overall survival outcomes in the treatment of newly diagnosed glioblastoma. However, both animal and clinical studies have shown that bevacizumab is effective in the treatment of radiation necrosis. For reirradiation, the incidence of radiation necrosis is higher than with primary radiation and thus it may be expected that bevacizumab will reduce the incidence of radiation necrosis. Bevacizumab, an angiogenesis inhibitor, does not alter MGMT methylation status or glioblastoma phenotype. References: Gutin, et al. Safety and efficacy of bevacizumab with hypofractionated stereotactic irradiation for recurrent malignant gliomas. Int J Radiat Oncol Biol Phys. 2009 Sep 1; 75(1):156-63. Farid, et al. Restriction-Spectrum Imaging of Bevacizumab-Related Necrosis in a Patient with GBM. Front Oncol. 2013 Sep 30; 3:258.

182. To which class of targeted therapies inhibitors does Bortezomib (Velcade) belong? (A) Tyrosine kinase (B) Proteasome (C) mTor (D) K-ras

Key: B Solution: Bortezomib is a proteasome inhibitor. References: https://www.velcade.comLUnderstanding-velcadeLAbout-velcade.

300. Which experimental technique can be used to measure the metastatic potential of a cancer cell? (A) Clonogenic survival assay (B) Tail vein injection assay (C) TCD50 flank xenograft assay (D) Wound healing assay

Key: B Solution: Clonogenic survival assays are in vitro assays which assess the ability of a cell to form a colony. Tail vein injection assays are designed to assess the ability of a cell to extravasate from the bloodstream and develop metastatic deposits. TCDSO assays investigate the radiosensitivity of a cell line, but do not study metastasis. Wound healing assays are in vitro assays that investigate the ability of a cell to migrate across a "wound". While this may correlate with metastatic potential, it is not a direct test of metastases. References: Hall and Giaccia Radiobiology for the Radiologist.

296. Which of these agents damages DNA and is regarded as a radiosensitizer? (A) Avastin (B) Cisplatin (C) Tirapazamine (D) Taxol

Key: B Solution: Concurrent administration of radiation and chemotherapy has emerged as a dominant form of cancer treatment. Nevertheless, our understanding of the specific mechanisms of interaction between radiation and chemotherapy is still evolving. Biological evidence gained in experimental cell lines and tumors suggests that cisplatin lesions cause repair inhibition of radiation-induced DNA damage affecting both homologous recombination and nonhomologous end joining and therefore act as a radiosensitizer. References: Wilson GD, Bentzen SM, Harari PM. Biologic basis for combining drugs with radiation. Semin Radiat Oneal. 2006 Jan; 16(1):2-9.

184. Average CBCT doses used for IGRT are on the order of: (A) <1 mGy (B) 0.1-10 cGy (C) 25-50 cGy (D) 1-2 Gy

Key: B Solution: Dose from CBCT varies with imaging parameters and CBCT system. For the two most common CBCT systems in use for IGRT the ranges are: Elekta XVI: 0.1- 3.5 cGy and Varian OBI: 1.1-8.3 cGy. This is from a 2008 study, but the values still represent a reasonable range. References: Song, et al. Med Phys 35 (2008).

161. Elevated EGFR expression levels in the primary head and neck squamous cell carcinoma tumor have consistently been correlated with decreased survival. How is the EGFR expression level elevated? (A) Transcriptional deregulation (B) Gene amplification (C) EGFR receptor dimerization (D) Transactivation by G-protein-coupled receptors (GPCR)

Key: B Solution: EGFR status was analyzed in 86 tumor samples from 82 HNSCC patients by fluorescent in situ hybridization (FISH). High EGFR gene copy number by FISH is frequent in HNSCC and is a poor prognostic indicator. References: Chung CH, Ely K, McGavran L. Increased epidermal growth factor receptor gene copy number is associated with poor prognosis in head and neck squamous cell carcinomas. J Clin Oncol. 2006; 24:4170-4176. Grandis JR, Melhem MF, Gooding WE. Levels of TGF-alpha and EGFR protein in head and neck squamous cell carcinoma and patient survival. J Natl Cancer Inst. 1998; 90:824-832. Harari PM, Wheeler DL, Grandis JR. Molecular target approaches in head and neck cancer: epidermal growth factor receptor and beyond. Semin Radiat Oncol. 2009 Jan; 9(1):63-8.

35. Regarding the National Cancer Data Base analysis (Gynecologic Oncology, 2015), what is the reduction in risk of death with the addition of chemotherapy to postoperative radiation for pathologic node positive vulvar cancer patients? (A) 5-15% (B) 25-40% (C) 50-65% (D) 70-85%

Key: B Solution: For pathologic node-positive vulvar cancer, adjuvant radiotherapy has an established benefit, whereas the impact of chemotherapy in the adjuvant setting was previously not known. A National Cancer Data Base (NCDB) analysis was conducted to determine patterns of care and evaluate the survival impact of adjuvant chemotherapy. The NCDB was queried for vulvar cancer patients diagnosed from 1998-2011 who underwent extirpative surgery with confirmed inguinal nodal involvement treated with adjuvant radiotherapy. References: Gill, et al: Impact of adjuvant chemotherapy with radiation for node-positive vulvar cancer: A National Cancer Data Base (NCDB) analysis. Gynecologic Oncology 137 (2015) 365-372.

20. Which is the best treatment option for a medically inoperable 61 year old with clinical stage T1b1 squamous cell carcinoma of the cervix? (A) Conization (B) EBRT + brachytherapy (C) ChemoRT + brachytherapy (D) Chemotherapy followed by brachytherapy

Key: B Solution: For patients with cervical cancer treated with definitive radiotherapy, brachytherapy is central to cure. In addition, for stage IB1, prophylactic treatment of pelvic lymph nodes is required. There is yet no evidence to support use of radiosensitizing chemotherapy with definitive radiotherapy for stage IB1disease. Excisional biopsy is not adequate for invasive cancer meeting the definition of stage IB1. References: Delgado, et al. A prospective surgical pathological study of stage I squamous carcinoma of the cervix: a Gynecologic Oncology Group Study (1990).

2. What is the recommended preoperative dose (Gy) of radiation (LDR equivalent) for gross Stage IIB adenocarcinoma of the endometrium? (A) 60-65 (B) 70-75 (C) 80-85 (D) 90-95

Key: B Solution: For preoperative therapy for gross stage llB disease a total dose of 75 to 80 Gy LDR equivalent to the tumor volume is recommended. References: NCCN guidelines Version 2.2015

63. A thin layer of wax is used to cover metal eye shields in order to: (A) increase the enhanced surface dose from backscattered electrons. (B) decrease the enhanced surface dose from backscattered electrons. (C) increase the dose at depth from penetrating photons. (D) decrease the dose at depth from penetrating photons.

Key: B Solution: High Z material increases dose to the surface by increasing the electron backscatter. A thin layer of low Z material will absorb these electrons and therefore reduce the surface dose. References: Chapter 8 of Radiation Oncology Physics: A Handbook for Teachers and Students, E. Podgorsak 2005.

270. High risk HPV-types are associated with approximately what percentage of cancers worldwide? (A) <1% (B) 5% (C) 10% (D) 20%

Key: B Solution: High risk HPV is associated with approximately 5% of cancers worldwide. In the United States, it is associated with about 3% of female cancers and 2% of male cancers. References: http://www.cancer.gov/about-cancer/causes-prevention/risk/infectiousagents/ hpv-fact-sheet#r12. de Martel C, Ferlay J, Franceschi S, et al. Global burden of cancers attributable to infections in 2008: A review and synthetic analysis. Lancet Oncology 2012; 13(6):607-615.

70. Compared to primary glioblastomas, secondary glioblastomas that arise from low grade gliomas tend to have which molecular or pathologic characteristic? (A) Pseudo-pallisading necrosis (B) Mutation of IDH1 (C) Vascular proliferation (D) EGFR amplification

Key: B Solution: IDH1 mutations occur in up to 80% of grade II gliomas, but in less than 5% of de novo GBMs. A GBM with an IDH1 mutation usually denotes a secondary GBM, transformed from a low grade glioma. EGFR amplification, pseudo-pallisading necrosis and vascular proliferation are associated with GBM References: Olar Sem Rad One 25, 2015 26050585.

34. Which of the following treatment modalities is NOT recommended outside of a clinical trial or registry by the ASTRO "Choosing Wisely" Campaign for patients with prostate cancer? (A) SBRT (B) Proton beam therapy (C) HDR Brachytherapy (D) Electromagnetic tracking

Key: B Solution: In 2013, ASTRO released a list of five radiation oncology specific treatments that are commonly ordered but may not always be appropriate as part of the national "Choosing Wisely'" campaign. The list included two dealing with prostate cancer, 1. Don't initiate management of low-risk prostate cancer without discussing active surveillance, and 2. Don't routinely recommend proton beam therapy for prostate cancer outside of a prospective clinical trial or registry. It states that "there is no clear evidence that proton beam therapy for prostate cancer offers any clinical advantage over other forms of definitive radiation therapy and that clinical trials are necessary to establish a possible advantage of this expensive therapy." References: Hahn C, Kavanagh B, Bhatnagar A, Jacobson G, Lutz S, Patton C, Potters L, Steinberg M. Choosing wisely: the American Society for Radiation Oncology's top 5 list. Pract Radiat Oncol. 2014; 4(6):349-55.

258. Per PORTEC-1 (Postoperative Radiation Therapy Endometrial Cancer) what is the 3 year OS (%) after salvage radiation for patients who relapsed at the vaginal cuff following observation? (A) 60 (B) 70 (C) 80 (D) 90

Key: B Solution: In PORTEC-1, the 3-year overall survival was 51% for the 46 control patients who relapsed and were treated with salvage radiation and 73% for patients with recurrence at the vaginal cuff. While reserving radiation in the adjuvant setting potentially prevents overtreatment of patients, salvage radiation is not curative for many patients. References: Creutzberg CL, van Putten WL, Koper PC, et al. Survival after relapse in patients with endometrial cancer: Results from a randomized trial. Gynecol Oncol 2003; 89:201-209.

174. In a molybdenum-based mammography unit, the X-ray energy spectrum consists of a broad distribution of photon energies and two sharp peaks at 17.5 and 19.6 keV. The process responsible for these peaks is: (A) Bremsstrahlung. (B) characteristic radiation. (C) Compton scattering. (D) electron emission.

Key: B Solution: In a mammography unit, or any x-ray unit, a broad spectrum of photons is produced by the bremsstrahlung process, in which a free electron is deflected by the electric field of a nucleus. If one of the electrons ionizes an atom by knocking out an innershell electron, then that opening in the electron orbital will be filled by an electron from a higher energy level. In dropping from one atomic level to another, a photon having energy exactly equal to the energy difference between the two levels will be emitted. This characteristic radiation is specific to the atomic structure of the target material in question. For molybdenum, the emission of 17.5 and 19.6 keV photons provide a supply of lowenergy photons that are very useful for discriminating the small differences seen in healthy and cancerous breast tissue. References: Khan FM. "Physics of Radiation Therapy, 4th Edition."

245. Which measurement device would NOT be sufficient for patient specific quality assurance on an IMRT plan? (A) Ion chamber along with radiochromic film (B) Thermoluminescent detector (TLD) (C) 2D diode detector array (D) Electronic portal imager

Key: B Solution: Intensity-modulated radiotherapy utilizes computer optimization to create precisely crafted beams that overlap a target area in such a way that dose to surrounding critical structures is minimized. To achieve this, the photon fluence distribution within any single beam can be complex and highly inhomogeneous. Performing quality assurance on such a beam requires a measurement technique that can assess the planar distribution of dose across the beam. Techniques measuring dose to a single point, such as those using a single detector or TLD, do not provide information about in homogeneities elsewhere in the beam, and are therefore inadequate for determining whether the delivered dose matches the intended dose. References: Khan FM."Physics of Radiation Therapy, 4th Edition", Chapter 20.

147. Which of the following is the most common route of intracranial invasion in a nasopharyngeal carcinoma? (A) Direct invasion through clivus (B) Foramen lacerum (C) Foramen ovale (D) Jugular foramen

Key: B Solution: It is important to know the routes of spread for nasopharyngeal cancer. NPC patients can present with cranial nerve neuropathy due to local extension of the disease. The foramen lacerum, located directly above Rosenmuller fossa, is a weak spot in the base of skull. Through which the tumor can gain access into the cavernous sinus and the middle cranial fossa and invade cranial nerves II to VI. Tumor may also invade through the foramen ovale into middle cranial fossa, the petrous part of the temporal bone and the cavernous sinus. The most common cranial nerves involvements by NPS are V1 and V2 by tumor extension through foramen lacerum into cavernous sinus. References: Chong VF, Fan YF, Khoo JB. According to Perez and this reference: J Comput Assist Tomogr. 1996 Jul-Aug; 20(4):563-9. Nasopharyngeal carcinoma with intracranial spread: CT and MR characteristics...) the foramen ovale is the most common route of spread.

263. According to Kalapurakal, et al in IJROBP 2013, what is the PRINCIPAL organ spared by IMRT instead of AP-PA fields for pediatric whole lung irradiation? (A) Esophagus (B) Heart (C) Liver (D) Thyroid

Key: B Solution: Kalapurakal, et al, showed that the dose to the heart could be significantly reduced while improving PTV whole lung coverage. However, dose to the liver and thyroid were not significantly different. Esophagus was not assessed in the manuscript but is relatively small in cross sectional area and effectively included in whole lung PTV. References: Int J Radiation Oncol Biol Phys. 85(3):761-767.

278. In elderly patients with newly diagnosed anaplastic astrocytoma or glioblastoma, per NEJM 2007 what is the estimated median survival (months) with best supportive care alone? (A) 1 (B) 4 (C) 8 (D) 10

Key: B Solution: Keime-Guibert, et al randomized patients older than 70, with KPS > 70 with newly diagnosed anaplastic astrocytoma or glioblastoma multiform to supportive care only vs supportive care plus radiotherapy to 50 Gy. The median survival was 29.l weeks (95% Cl, 25.4 to 34.9) with radiotherapy plus supportive care and 16.9 weeks (95% Cl, 13.4 to 21.4) with supportive care alone. Radiotherapy improved median survival by 12.2 weeks. References: http://www.nejm.org/doi/full/10.1056/NEJMoa065901.

140. Which of the following compounds is approved by FDA for use in the event of a radiation emergency? (A) Keratinocyte growth factor (B) Potassium iodide (C) MESNA (D) Amifostine

Key: B Solution: Keratinocyte growth factor has been studied for the prevention of radiation mucositis; MESNA is used in reduce the incidence of hemorrhagic cystitis following ifosfamide or cyclophosphamide treatment; amifostine is used for the prevention of xerostomia; none of these compound is approved for use in a radiation emergency. Potassium iodide was approved by the FDA for use in the prevention of thyroid complications following an exposure to radioactive iodine. References: http://www.fda.gov/Drugs/EmergencyPreparedness/BioterrorismandDrugPreparedness/uc m072265.htm.

209. What is the biomarker profile for Luminal B breast cancer? (A) ER+ and/or PR+, HER2- (B) ER+ and/or PR+, HER2+ (C) ER-, PR-, HER2- (D) ER-, PR-, HER2+

Key: B Solution: Luminal A: ER positive and/or PR positive, HER2 negative, low Ki67 Luminal B: ER positive and/or PR positive, HER2 positive or HER2 negative with high Ki67 Triple negative/basal-like: ER negative, PR negative, HER2 negative HER2 type: ER negative, PR negative, HER2 positive. References: www.komen.org/BreastCancer/SubtypesofBreastCancer.html Molecular Subtypes of Breast Cancer/ Susan G. Komen, http://ww5.komen.org/BreastCancer/SubtypesofBreastCancer.html

275. What is the most common location of desmoid tumors in patients with Gardner syndrome? (A) Lower extremity (B) Intraabdominal cavity (C) Shoulder (D) Chest wall

Key: B Solution: Most of these patients develop abdominal tumors, one-half of which are intraabdominal. References: Uptodate.com.

232. What percent of prostate cancer patients are in the NCCN high-risk category at diagnosis? (A) 15 (B) 25 (C) 35 (D) 45

Key: B Solution: Nearly three-fourths of prostate cancer patients at diagnosis are lowintermediate risk. References: Cooperberg MR, Lubeck DP, Mehta SS, Carroll PR. CaPsure. Time trends in clinical risk stratification for prostate cancer: implications for outcomes (data from CaPSURE). J Uro/ 2003; 170(6 Pt 2): 521-5; discussion 56-7.

137. Which is the correct stage for a NSCLC patient with a 4 cm tumor and a separate 2 cm tumor in the same lobe, an ipsilateral paratracheal lymph nodes, and no distant metastases? (A) T2bN1M0 stage IIB (B) T3N2M0 stage IIIA (C) T4N2M0 stage IIIB (D) T2bN2M0 stage IIIA

Key: B Solution: Not given References: AJCC Cancer staging manual, seventh edition.

204. What percentage of the individual populations is represented in a normally distributed population having a 1.5 SD about the mean? (A) 68% (B) 90% (C) 95% (D) 100%

Key: B Solution: Of the normal distribution, 1 standard deviation of the mean represents approximately 68%, 1.5 standard deviations of the mean represent approximately 90%, 2 standard deviations of the mean represents approximately 95%, and 3 standard deviations of the mean represent approximately 99% of the population. References: Dawson B, and Trapp RG. Basic and Clinical Biostatistics. 4th edition, 2004; pp 31 and 80.

99. For SRS of a 3 cm brain metastasis, what is an acceptable ratio of the prescription isodose volume to the PTV volume? (A) 0.9 (B) 1.2 (C) 1.8 (D) 2.2

Key: B Solution: Option A is unrealistically conformal and will lead to a 10% underdosing of the target volume, whereas options C and D conformal will not be sufficiently conformal and will lead to over-irradiation of normal tissue. References: RTOG 9508 Curr. Probl. Cancer, May/June 2005.

195. What is the MOST common type of plasmacytoma? (A) Extraosseous (B) Osseous (C) Porous (D) Extraporous

Key: B Solution: Osseous plasmacytomas are 40% more common than extraosseous. Porous and extraporous do not exist. References: http://www.nccn.org/professionals/physician_gls/PDF/myeloma.pdf.

66. What is the preferred treatment approach for a patient diagnosed epithelioid mesothelioma with involvement of a mediastinal lymph node without evidence of metastatic disease? (A) Observation (B) Combination chemotherapy (C) Extrapleural pneumonectomy followed by adjuvant chemotherapy and RT (D) Decortication and RT

Key: B Solution: Patients with N+ nodal disease with malignant mesothelioma should not be offered surgical options. Palliative chemotherapy using combined chemotherapy with cisplatin and pemetrexed should be considered the standard of care. References: NCCN guidelines.

257. What is the risk of IBTR associated with a positive surgical margin (tumor on ink) in a stage I or II patient receiving breast conservation therapy? (A) 1-fold increase (B) 2-fold increase (C) 4-fold increase (D) 5-fold increase

Key: B Solution: Positive margins are associated with a 2-fold increase in risk of IBTR compared with negative margins. The use of no ink on tumor serves as the standard for an adequate margin in invasive cancer is associated with low rates of IBTR. References: Moran, Meena, et al. Society of Surgical Oncology - American Society for Radiation Oncology Consensus Guidelines on Margins for Breast-Conserving Surgery with Whole-Breast Irradiation in Stages I and II Invasive Breast Cancer. Ann Surg Oncol. February 2014. Published online DOI 10.1245/s101434-014-3481-4/.

135. Which histologic variant of rhabdomyosarcoma has the WORST prognosis? (A) PAX-FOXO1 fusion-negative alveolar (B) PAX-FOXO1 fusion-positive alveolar (C) Spindle cell (D) Botryoid

Key: B Solution: Prognosis effusion-negative alveolar rhabdomyosarcoma is similar to that of embryonal rhabdomyosarcoma. Spindle cell and botryoid are variants of embryonal histologies and usually have a favorable prognosis. Rhabdomyosarcoma is the most common childhood soft tissue sarcoma. References: F1000Prime Reports 2015, 7:59 (doi: 10.12703/P7-59); uptodate.com.

142. Regarding RTOG 9802: (A) only low grade astrocytoma was included in the study. (B) radiation dose was 54 Gy. (C) initial publication in 2012 showed a modest OS benefit with the addition of chemotherapy to radiation. (D) chemotherapy included procarbazine, cytoxan, and vincristine.

Key: B Solution: RTOG 9802 included both low grade astrocytoma and oligodendroglioma patients. Radiation dose was 54. Chemotherapy used were procabazine, lomustine, vincristine. In 2012, Shaw et al reported no OS benefit with the addition of PCV chemotherapy to radiation. However, and update at ASTRO 2014 showed that with longer median follow-up of 11.9 months, the addition of PCV chemotherapy to radiation improved overall survival from 7.8 years to 13.3 years. References: http://jco.ascopubs.org/content/30/25/3065.full ; http://www.redjournal.org/articleS0360-3016{14)00804-9/abstract .

295. Why is isotope Radium-223 used for treatment of bone metastases? (A) Radium-223 emits a low LET electron (B) Preferentially absorbed by bone (C) 95% of the decay energy is in the form beta radiation (D) 10-minute half-life reduces normal tissue damage

Key: B Solution: Radium 223 is an alpha emitter. Its short range reduced the bone marrow dose which in turn reduced bone marrow toxicity. Its half-life of 11 days is also long enough for commercialization and wide use. Lack of clearance is an issue for all heavy metals and is not an advantage. References: https://en.wikipedia.org/wiki/Radium-223.

82. What is the appropriate stage and group for a child with a 6 cm orbital embryonal rhabdomyosarcoma that is not amenable to complete surgical resection? (A) Stage 1; Group I (B) Stage 1; Group III (C) Stage 3; Group I (D) Stage 3; Group III

Key: B Solution: Rationale is not given for this question. References: F1000Prime Reports 2015, 7:59 (doi: 10.12703/P7-59).

249. In which linac component are Bremsstrahlung photons created for treatment delivery? (A) Reflection target (B) Transmission target (C) Scattering foil (D) Flattening filter

Key: B Solution: Reflection targets are not used in linacs to generate treatment beams. Scatter foils and flattening filters may be sources for some undesired bremsstrahlung photons but creating them for the treatment beam is not their purpose. References: "The Physics and Technology of Radiation Therapy" McDermott & Orton, pg. 9-18.

100. What is the histopathologic finding MOST characteristic of pediatric atypical teratoid/rhabdoid tumors (AT/RT) of the brain? (A) Rosenthal fibers (B) Negative INI-1staining (C) Pseudo-palisading necrosis (D) Chromosome 19q deletion

Key: B Solution: Rosenthal fibers are associated with low grade gliomas including pilocytic astrocytomas. AT/RT are characterized by loss of SMARCB1/INI-1 tumor suppressor gene and an absence of INI-1 by FISH analysis is diagnostic. Pseudopallisading necrosis is a feature of high grade gliomas. Chromosome 19q deletion is associated with oligodendrogliomas. References: Halperin, et al. Pediatric Radiation Oncology, Chapters 3 and 4.

158. A 45 year old woman with a new diagnosis of breast cancer and a family history positive of first degree relatives with leukemia and primary brain tumors could be associated with which gene mutations or syndromes? (A) Ataxia Telangiectasia (B) Li-Fraumeni Syndrome (C) Lynch Syndrome (D) Cowden Syndrome

Key: B Solution: The ATM gene normally helps repair damaged DNA. Inheriting 2 abnormal copies of this gene causes the disease ataxia-telangiectasia. Inheriting 1 mutated copy of this gene has been linked to a high rate of breast cancer in some families. The TP53 gene gives instructions for making a protein called p53 that helps stop the growth of abnormal cells. Inherited mutations of this gene cause Li-Fraumeni syndrome People with this syndrome have an increased risk of developing breast cancer, and several other cancers such as leukemia, brain tumors, and sarcomas (cancer of bones or connective tissue). This is a rare cause of breast cancer. Lynch syndrome, also known as hereditary non- polyposis colorectal cancer (HNPCC), is a type of inherited cancer of the digestive tract. People who have Lynch syndrome have a significantly increased risk of developing colorectal cancer and an increased risk of developing other types of cancers, such as endometrial (uterine), stomach, breast, ovarian, small bowel pancreatic, prostate, urinary tract, liver, kidney, and bile duct cancers. Lynch syndrome is a possible diagnosis when multiple people on the same side of the family are diagnosed with colorectal cancer. In addition, cancer is more likely to be diagnosed at a young age. The average age for colorectal cancer to be diagnosed in someone with Lynch syndrome is 45, as compared with the average age of 72 for a new diagnosis of colorectal cancer in the general population. In Lynch syndrome, colorectal cancer is somewhat more likely to develop on the right side of the colon. Cowden syndrome (CS) is part of the PTEN hamartoma tumor syndrome. CS is characterized by a high risk of both benign and cancerous tumors of the breast, thyroid, endometrium (uterus), colorectal, kidney, and skin (melanoma). Other #Key features of CS are skin changes, such as trichilemmomas (skin tags) and papillomatous papules and macrocephaly. References: NCCN Guidelines-Genetic/Familial High Risk Assessment: Breast and Ovarian Cancer American Cancer Society web site: Causes and Prevention for Breast Cancer: www.cancer.org. Lynch Syndrome and Cowden Syndrome: www.cancer.net.

272. For a 15 MeV electron beam with a 10x10 cm cone, which statement describes the behavior of the 20% and 80% isodose lines at the field edges? (A) Both will constrict with increasing depth. (B) The 20% will bulge out and 80% will constrict with increasing depth. (C) The 80% will bulge out and 20% will constrict with increasing depth. (D) The isodose line pattern is similar to that of a 12 MV photon field.

Key: B Solution: The IDLs for a standard electron field such as 15 MeV, 10x10 cm bulge out for lower IDL levels and constrict for high IDL. This is an important concept for creating block edges large enough for adequate PTV coverage and also for matching adjacent treatment fields. This is a difference between photon and electron fields. References: Podgorsak, Radiation Oncology Physics Handbook, Ch. 8.

150. The Mayo Clinic has prospectively evaluated liver transplantation after external beam chemoradiation and brachytherapy for: (A) intrahepatic cholangiocarcinoma. (B) hilar cholangiocarcinoma. (C) extrahepatic cholangiocarcinoma. (D) gallbladder cancer.

Key: B Solution: The Mayo Clinic protocol includes careful selection of patients with early stage CCA which is either unresectable or arising in the setting of underlying PSC. The upper limit of tumor size is 3 cm, and there must be no evidence of intra- or extrahepatic metastases. The protocol specifically excludes patients with intrahepatic CCA or gallbladder involvement. Neoadjuvant therapy includes 4000 to 4500 cGy administered by external beam, followed by 2000 to 3000 cGy transcatheter irradiation with iridium. 5-FU is given during the radiation treatment and capecitabine is then administered until transplantation. Prior to transplantation, patients undergo a staging abdominal exploration. Regional lymph node metastases, peritoneal metastases, or locally extensive disease preclude transplantation. References: Heimbach JK, Haddock MG, Alberts SR, Nyberg SL, Ishitani MB, Rosen CB, Gores GJ. Liver Transpl. 2004 Oct; 10(10 Suppl 2):S65-8. Transplantation for hilar cholangiocarcinoma.

273. According to NCCN guidelines, adjuvant therapy for a pT3N0 urothelial carcinoma of the ureter is: (A) RT. (B) chemotherapy. (C) concurrent chemotherapy and RT. (D) sequential chemotherapy and RT.

Key: B Solution: The NCCN guidelines recommend that patients with a resected pT2, pT3, pT4, or pN+ urothelial cancer be considered for adjuvant chemotherapy. References: NCCN Guidelines Version 2.2015 Upper GU Tract Tumors.

193. Based on the results of RTOG 0617 (Bradley 2015), which of the following cardiac dose constraints were found to be predictive of OS on multivariate analysis? (A) V20 (B) V30 (C) V45 (D) V60

Key: B Solution: The RTOG 0617 was a double randomized study looking at 60 vs 74Gy as well as cisplatin +/- cetuximab for the treatment of locally advanced Stage IIIA-B NSCLC. One of the important findings of the study was that cardiac V5 and V30 were both predictive of mortality on multivariate analysis. This has been incorporated into RTOG 1308 where the cardiac constraints were now lowered to V30<50%. References: Bradley et al. Lancet Oncol. 2015 Feb; 16(2):187-99.

229. Which mouse strain has a Prkdc mutation and severe deficiency in functional B and T lymphocytes and is used for tumor xenografts experiments? (A) Nude mouse (B) SCID mouse (C) BALB/c mouse (D) C57Black6 mouse

Key: B Solution: The SCID mouse carries the Prkdc mutation with the phenotype of significantly impaired humoral and cellular immune systems. Nude mice have a disruption of the FOXNl gene mainly resulting in a deteriorated or absent thymus. The BALB/c and the C57 black 6 mice are commonly used inbred animals used as models for human disease. References: http://jaxmice.jax.org/strain/001303.html. Ito M, Hiramatsu H, Kobayashi K, Suzue K, Kawahata M, Hioki K, Ueyama Y, Koyanagi Y, Sugamura K, Tsuji K, Heike T, Nakahata T. NOD/SCID/gamma(c)(null) mouse: an excellent recipient mouse model for engraftment of human cells. Blood. 2002 Nov 1;100(9):3175- 82.

233. What is the T stage and 3 year OS for a lymph node negative 10 cm renal cell carcinoma confined to the kidney (A) T2a; 50% (B) T2a; 80% (C) T2b; 50 % (D) T2b; 80%

Key: B Solution: The T stage of a 10 cm renal cell tumor is T2a. This corresponds to a stage II prognostic group with an associated 3 year survival of 80%. References: Kidney. AJCC Cancer Staging Manual, Springer, New York 2009.

282. Based on QUANTEC, NTCP estimates to achieve a <1% probability of cardiac mortality are based upon which of the following dose constraints? (A) Whole heart < 30 Gy (B) V25 < 10% (C) Mean heart < 10Gy (D) Mean LAD < 8 Gy

Key: B Solution: The V25 for the heart should be < 10% to achieve a <1% risk of cardiac mortality. Every effort should be made to keep the heart dose and LAD to as low as reasonably achievable. References: Marks L, et al. Use of Normal tissue complication probability models in the clinic. Int. J Radiation Oncology Biol. Phys, Vol 76, No 3, supplement, pp. S10-19, 2010. QUANTEC Data Tables.

5. What does the Winston Lutz test verify? (A) Accuracy of collimator rotation (B) Coincidence of laser and radiation isocenter (C) MLC leaf leakage (D) Accuracy of linac output

Key: B Solution: The Winston-Lutz test uses a small target that is positioned at isocenter using lasers and then imaged using the treatment beam from different gantry/collimator and couch combinations to verify the coincidence of the laser isocenter with the radiation isocenter. References: Lutz W, Winston KR, Maleki N, A system for stereotactic radiosurgery with a linear accelerator, Int J Radiat Oncol Biol Phys 1988; 14:373.

224. What assumption does the absolute risk model for radiation-induced carcinogenesis make? (A) Reduces cancer risk with age (B) Produces an increase in cancer risk over that occurring naturally (C) Increases the risk of cancer as a function of dose (D) Increases the risk of cancer as a function of the square of the dose

Key: B Solution: The absolute risk model assumes any increase in carcinogenic risk will be simply added upon the risk of cancer induction in the absence of radiation exposure. The relative risk model takes age into account as the risk for many cancers increases with age. The time dependent relative risk model treats cancer risk as a function of dose, the square of the dose, age at exposure, and time since exposure. References: Hall and Giaccia, Radiobiology for the Radiologist, Thompson DE, Mabuchi K, Ron E, Soda M, Tokunaga M, Ochikubo S, Sugimoto S, Ikeda T, Terasaki M, Izumi S, et al. Cancer incidence in atomic bomb survivors. Part II: Solid tumors, 1958-1987. Radiat Res. 1994 Feb; 137(2 Suppl):517-67.h Edition, 2012 pp. 138-139.

157. When comparing proton to photon therapy, which might account for the increase of the therapeutic ratio for a pediatric patient treated for medulloblastoma? (A) Reduced RBE of proton beams enhances tumor control (B) Reduced dose to anterior structures including the heart (C) Sharper penumbra of proton beams reduces exposure to the kidneys (D) Reduced neurotoxicity with proton therapy

Key: B Solution: The correct answer is B. For medulloblastoma the entire brain and spine are treated. For treatment of the spine, in contrast to photon therapy, proton therapy eliminates exit dose to anterior structures such as the heart. This reduction in dose is expected to reduce the incidence of radiation induced cardiac disease. For proton therapy, the RBE is 1.1 in comparison to photons. This is accounted for during treatment planning and hence the effective doses for either modality are expected to be similar. References: Kris S. Armoogum and Nicola Thorp. Dosimetric Comparison and Potential for Improved Clinical Outcomes of Paediatric CNS Patients Treated with Protons or IMRT. Cancers (Basel). 2015 Jun; 7(2): 706-722. Yock, et al. Long-term toxic effects of proton radiotherapy for paediatric medulloblastoma: a phase 2 single-arm study. Lancet Oncol. 2016 Mar; 17(3):287-98. Levin, et al. Proton beam therapy. Br J Cancer. 2005 Oct 17; 93(8):849-54.

251. What is the risk of neck recurrence without adjuvant radiation therapy for a pathologic T1-2N0 low risk oral tongue cancer? (A) 10% (B) 20% (C) 30% (D) 40% NOTE: This item was 0-weighted for scoring purposes upon post-exam statistical item analysis (did not count for or against candidate in calculation of test scores). (i.e. due to the question not indicating if a neck dissection was performed).

Key: B Solution: The decision for adjuvant PORT in early stage oral tongue cancer is determined by adverse pathologic features of the primary tumor and cervical lymph nodes. These include close/positive margins, lymphovascular invasion, perineural invasion, and positive lymph nodes. Patients without these adverse features are considered low risk (pT1-T2NO disease) and, thus, typically do not receive PORT. The largest series of long-term follow up on these low risk oral tongue patients demonstrated unexpected high recurrence rate; 10% local, and 18% regional. Among the regional recurrence, 61% recurred in ipsilateral, and 39% in contralateral neck. Failure occurs predominantly in patients who have primary tumors that are 4 mm thick. Only approximately 33% of patients who develop a neck recurrence are successfully salvaged. References: Ganly, et al. Long-term regional control and survival in patients with "lowrisk," early stage oral tongue cancer managed by partial glossectomy and neck dissection without postoperative radiation: the importance of tumor thickness. Cancer 2013 119(6) 1168-76.

121. The magnetic field from MRI machine in the same room as a linac can impact the dose distribution primarily by: (A) changing the photon beam into an electron beam. (B) changing the secondary electron trajectories. (C) changing the temperature of the patient's tissue. (D) increasing neutron contamination at the surface.

Key: B Solution: The magnetic field from an in-room MRI unit will impact the secondary electrons. The magnetic field cannot change a photon beam into an electron beam. The dose distribution is not impacted by tissue temperature. The magnetic field will impact the electron trajectories, which has also been shown. This question tests the fundamental physics of both magnetic fields and dosimetric parameters. References: Raaymakers B, et al. "Integrating a MRI scanner with a 6 MV radiotherapy accelerator: Dose deposition in a transverse magnetic field" Physics in Medicine and Biology 2004.

15. What is the most common presentation of patients with osteosarcoma? (A) Fever (B) Pain (C) Weight loss (D) Pathologic fracture

Key: B Solution: The majority of patients with osteosarcoma present with localized pain, typically of several months' duration. Pain frequently begins after an injury, and may wax and wane over time. Systemic symptoms such as fever, weight loss, and malaise are generally absent. References: Uptodate.com

98. What is the MOST common pattern of relapse in patients presenting with superior sulcus tumors (T3-4 N0-1) treated with induction chemoRT followed by surgical resection? (A) Local recurrence (B) Distant metastases (C) Nodal recurrence within the mediastinum (D) Nodal recurrence within the supraclavicular fossa

Key: B Solution: The pattern of recurrence in the Intergroup study was distant (non-brain) only (33%), brain only (33%), local only (17%), local + distant (12%). Despite the extensive local disease, 76% of patients underwent complete resection with pathological CR or minimal microscopic disease seen in 56% of the resection specimens, resulting in low local recurrence rates. In appropriately staged patients with mediastinoscopy (or EBUS + PETCT), recurrence in nodal regions outside of CTV remains low. References: Rusch, et al. Induction chemoradiation and surgical resection for superior sulcus non-small-cell lung carcinomas: long-term results of Southwest Oncology Group Trial 9416 (Intergroup Trial 0160). J Clin Oncol. 2007 Jan 20; 25(3):313-8.

91. Regarding the French multicenter Phase II trial (2014), what is the anticipated grade 2 acute GI toxicity (%) with postoperative 45 Gy pelvic IMRT for Stage I-II endometrial cancer? (A) 10 (B) 25 (C) 33 (D) 50

Key: B Solution: The phase II RTCMIENDOMETRE trial was designed to test the hypothesis that IMRT could reduce the incidence of grade 2 or more acute GI toxicity toless than 30% in patients irradiated post-operatively for an endometrial cancer. In accordance with the hypothesis, post-operative IMRT resulted in a low rate (27%) of acute GI grade 2 toxicity, in patients with endometrial carcinomas. Whereas "conventional" pelvic irradiation (up to 45-50Gy) following hysterectomy is associated with a high rate of adverse gastro-i References: Barillot I, Tavernier E, Peignaux I, Williaume D, Nickers P, Leblanc-Onfroy M, Lerouge D. Impact of postoperative intensity modulated radiotherapy on acute gastrointestinal toxicity for patients with endometrial cancer: results of the phase II RTCMIENDOMETRE French multicentre trial. Radiother Oncol. 2014 Apr; 111(1):138-43. Epub 2014 Mar 11.

123. When calibrating a 6 MV beam in flattening filter free (FFF) mode, the position of the ion chamber is important because: (A) the neutron contamination is minimized. (B) the shape of the radiation fluence profile is non-uniform. (C) the dose rate is low. (D) the beam steering is unstable.

Key: B Solution: The radiation fluence profiles are intentionally non-uniform in FFF mode, with the maximum output located on the central axis. If the beam is calibrated when the ion chamber was off-axis, the calibration point would not be at the location of maximum dose. The dose rate is very high in FFF beams. Beam steering is stable in modern linacs. Neutron contamination is not an issue in this situation. References: Georg D, et al. "Current status and future perspective of flattening filter free photon beams," Medical Physics 38:1280 (2011).

134. Regarding adjuvant therapy for a NSCLC patient with a 6 cm adenocarcinoma with 3 involved hilar lymph nodes with tumor 5 mm from the bronchial margin: (A) no further therapy recommended. (B) post-operative RT is recommended if adjuvant chemotherapy refused. (C) post-operative concurrent chemoRT. (D) a completion pneumonectomy is recommended because of close margins.

Key: B Solution: The recommended adjuvant treatment for this patient is chemotherapy alone. While PORT is currently recommended only for patients with positive mediastinal disease (pN2), analysis of PORT delivered in the ANITA trial demonstrated benefit of PORT in pN1 patients who did not receive adjuvant chemotherapy. References: Douillard et al. Int J Radiat Oncol Biol Phys. 2008 Nov 1; 72(3):695-701. Impact of postoperative radiation therapy on survival in patients with complete resection and stage I, II, or IIIA non-small- cell lung cancer treated with adjuvant chemotherapy: the adjuvant Navelbine International Trialist Association (ANITA) Randomized Trial.

207. Which of the following linac treatment head components is unique to electron beam production? (A) Bending magnet assembly (B) Scattering foil (C) Waveguide (D) Electron gun

Key: B Solution: The scattering foil is used to spread a very thin electron beam into a clinically usable uniform field and is not used when a photon beam is required. All of the other components are used as part of the acceleration and energy selection of an original electron stream that can then be turned into a clinical photon or electron beam. References: Khan FM. The Physics of Radiation Therapy, Third Ed, Lippincott Williams & Wilkins, New York, 2003, pp 45-46.

283. Regarding ECOG 5194 (Local Excision Alone Without Irradiation for DCIS of the Breast): (A) the median lesion size was 12 mm. (B) detection was by mammogram screening only. (C) the population consisted of post-menopausal women only. (D) the study excluded patients with a history of bloody nipple discharge.

Key: B Solution: The study included both pre and post-menopausal women. Median lesion size was 6 mm. All lesions were detected by mammography screening only. Patients with bloody nipple discharge were included but represented only 2.5% of the patient population. This study is important because it was used to validate the Oncotype DX DCIS score TM. References: Hughes L, et al. Local Excision Alone Without Irradiation for Ductal Carcinoma In Situ of the Breast: A Trial of the Eastern Cooperative Oncology Group. J. Clin. Oncol 27:5319-5324, 2009.

93. To qualify as an NRC "Medical Event" the total dose must differ from the prescribed dose by___________ % or more and the fractionated dose by _________% or more. (A) 20, 20 (B) 20, 50 (C) 50, 20 (D) 50, 50

Key: B Solution: The term "differ from" is important because an under-dose could qualify as a "medical event." References: Nuclear Regulatory Commission, Code of Federal Regulations, 10 CFR Part 35, Section 35.3045(a).

65. Which of the following is a sub-site of the hypopharynx? (A) Vallecula (B) Postcricoid (C) Ventricle (D) Morgagni's sinus

Key: B Solution: The vallecula is part of the boundary between the larynx and the oropharynx. The ventricle is part of the larynx between the true vocal cords and the false cords. Morgangni's sinus is the same thing as the ventricle. The postcricoid is a subsite of the hypopharynx. The postcricoid region is comprised of the mucosa overlying the cricoid cartilage, with the arytenoid and esophageal mucosa forming the superior and inferior borders, respectively. References: Perez and Brady's Principles and Practice of Radiation Oncology, 5th Edition, Chapter 43.

250. What is the TN stage for a patient with a 5.1 cm right upper lobe adenocarcinoma of the lung, and biopsy-proven levels 4R, 5, and 7 lymph nodes? (A) T2bN2 (B) T2bN3 (C) T3N2 (D) T3N3

Key: B Solution: This patient presents with a T2 (3-7 cm) tumor with bilateral mediastinal lymph nodes (N3). Station 5 for a right sided tumor is considered a contralateral node, therefore N3. References: AJCC manual 7th edition.

25. As per the Hayflick limit, in how many successive divisions would a normal human diploid cell undergo before it becomes senescent? (A) 5-10 (B) 40-60 (C) 150-200 (D) 400-500

Key: B Solution: This phenomenon was first described as the Hayflick limit, and was important in aging research since as it disproved the prevailing theory of immortality of normal cells. Empirical evidence shows that the telomeres associated with each cell's DNA will get slightly shorter with each new cell division until they shorten to a critical length preventing division. References: Hayflick and Moorhead. "The serial cultivation of human diploid cell strains." Exp Cell Res 25 (3): 585-621. Hayflick. The limited in vitro lifetime of human diploid cell strains. Exp Cell Res. 1965 Mar; 37: 614-36.

262. The BASIC biological rational and aim of hyperfractionation is to: (A) reduce early effects. (B) allow full tumor reoxygenation. (C) distinguish between early and late effects. (D) prevent tumor proliferation during treatment. NOTE: This item was 0-weighted for scoring purposes upon post-exam statistical item analysis (did not count for or against candidate in calculation of test scores).

Key: C References: Hall, EJ. Radiobiology for the Radiologist, (2012) 7th Edition. Chapter 22. Anscher MS, Kong FM, Andrews K, Clough R, Marks LB, Bentel G, Jirtle RL., Int J Radiat Oncol Biol Phys, (1998). The rationale for continuous, hyperfractionated, accelerated radiotherapy (CHART). 1990 Nov; 19(5):1317-20.

220. Which assay can be used to MOST easily measure DNA double strand breaks from clinical tumor samples? (A) Pulse field gel electrophoresis (B) Graded field gel electrophoresis (C) COMET assay (D) Alkaline elution assay

Key: C References: Olive PL., Mutation Research, (2009) -Impact of the comet assay in radiobiology. 2009 Jan-Feb; 681(1):13-23.

225. Why is homologous recombination repair (HRR) more effective than nonhomologous end-joining (NHEJ) for DNA double strand breaks? (A) Operates in all cell cycle phases (B) Error prone (C) Higher repair fidelity (D) Short half-time of repair

Key: C References:Olive PL. Mutation Research, (2009) Induction and repair of DNA double strand breaks: the increasing spectrum of non-homologous end joining pathways. 2011 Jun 3; 711(1-2):61-72. Valerie K, Povirk LF, Oncogene, (2003) Regulation and mechanisms of mammalian doublestrand break repair. 2003 Sep 122(37):5792-812.

291. What is the appropriate testes dose (Gy) for ALL patients who have persistent testicular disease after induction chemotherapy? (A) 4 (B) 14 (C) 24 (D) 34

Key: C Solution: 24 Gy is the recommended dose for persistent testicular ALL. References: NCCN Guidelines 1.2015, p.5 Updates.

37. A 4 year-old boy with non-metastatic 5.5 cm non-parameningeal neck alveolar rhabdomyosarcoma has an incomplete excision followed by chemotherapy. What is the MOST appropriate total dose and timing of adjuvant RT relative to the start of chemotherapy? (A) 30.6 Gy starting at week 4 (B) 30.6 Gy starting at week 20 (C) 50.4 Gy starting at week 4 (D) 50.4 Gy starting at week 20

Key: C Solution: 30.6 Gy would be given after a gross total resection. A 50.4 Gy is the dose for gross residual disease. References: F1000Prime Reports 2015, 7:59 (doi: 10.12703/P7-59).

21. What patient selection criterion is considered UNSUITABLE for APBI according to the ASTRO consensus statement for APBI? (A) Age > 60 (B) Margins are negative (C) BRCA mutation is present (D) Tumor size is < 3 cm

Key: C Solution: A patient is unsuitable for APBI if any of the following criteria is met: Age <50, BRCA 1 or 2 mutation is present, tumor size is > 3 cm, stage is T3-4, margins are positive, extensive LVSI is present, multicentricity of disease, or node positive. References: Smith BD, et al. Accelerated Partial Breast Irradiation Consensus Statement from the American Society for Radiation Oncology (ASTRO). Int. J Radiation Oncology Biol. Phys, Vol 74, Issue 4, pp. 987-1001, July 15, 2009.

111. Regarding the role of elective nodal radiation tested in a randomized trial for stage III NSCLC treated with concurrent chemoRT (Yuan et al. 2007): (A) overall response rate was the same. (B) the RT pneumonitis rate was the same. (C) the rate of LC was better in the involved field arm. (D) OS was better in the involved field arm.

Key: C Solution: A total of 200 eligible patients with inoperable stage III non-small cell lung cancer (NSCLC) were treated with concurrent chemoradiotherapy and randomized into either an involved-field irradiation (IFI) or elective nodal irradiation (ENI) arm. A total of 4 to 6 cycles of cisplatin-based chemotherapy were delivered, and concurrent radiotherapy was started after the second cycle of chemotherapy. Three-dimensional conformal radiotherapy was delivered in once-daily fractions of 1.8 to 2 Gy to 68 to 74 Gy for IFI or 60 to 64 Gy for ENI. Patients in the IFI arm achieved better overall response rate (90% vs. 79%, P = 0.032) and better 5-years local control rate (51% vs.36%, P = 0.032) than those in the ENI arm. The radiation pneumonitis rate in patients with IFl was lower than in patients with ENI (17% vs. 29%, P = 0.044), and similar trends appeared in the radiation esophagitis, myelosuppression, and radiation pericarditis between 2 study arms, although not significantly. The 1-, 2-, and 5-year survival rates were 60.4%, 25.6%, and 18.3% for the ENI arm and 69.9%, 39.4%, and 25.1% for the IFI arm, respectively. Only the 2-year survival rates were statistically significant (P = 0.048). IFI arm achieved better overall response and local control than ENI arm, and it allowed a dose of 68 to 74 Gy to be safely administered to patients with inoperable stage III NSCLC. References: Yuan S, et al. A randomized study of involved-field irradiation versus elective nodal irradiation in combination with concurrent chemotherapy for inoperable stage III non-small cell lung cancer. Am J Clin Oncol 2007; 30(3):239-244.

210. Which of the following correctly identifies the superior and inferior borders of the Level III cervical lymph node chain? (A) Skull base and cranial aspect of hyoid bone (B) Mandible and anterior belly of the digastric muscle (C) Caudal aspect of hyoid bone and caudal aspect of cricoid cartilage (D) Cricoid cartilage and caudal aspect of clavicle

Key: C Solution: According to consensus guidelines Level III is bounded laterally by the deep surface of the SCM, medially by the scalenes and carotid artery, anteriorly by the anterior surface of the SCM or the thyro-hyoid muscle, and posteriorly by the posterior border of the SCM. The superior border is the caudal aspect of the hyoid bone and the inferior aspect is the caudal aspect of the cricoid cartilage. Choice A represents the superior and inferior borders of Level II. Choice B represents the superior and inferior borders of Level IB. Choice D represents level IV nodes. References: Gregoire, et al. Delineation of the neck node levels for head and neck tumors: A 2013 update. DAHANCA, EORTC, HKNPCSG, NCIC CTG, NCRI, RTOG, TROG consensus guidelines. Radiotherapy and Oncology. January 2014.

256. The MOST likely late toxicity of rectal irradiation (i.e. at 3-4 years after radiation) include: (A) rectal spasm and cramping. (B) intractable rectal pain. (C) decreased storage capacity. (D) rectal distention.

Key: C Solution: Acute rectal effects occur during or soon after RT and typically include softer or diarrhea-like stools, pain, a sense of rectal distention with cramping, and frequency. Occasionally, superficial ulceration causes bleeding that may require endoscopic cauterization, treatment for anemia, or transfusion. Late injuries are usually clinically manifest within 3 to 4 years after RT and may include stricture, diminished rectal compliance, and decreasing storage capacity with resultant small/frequent bowel movements. Injury to the anal musculature can lead to fecal incontinence or stricture. These morbidities can be severe and markedly affect quality of life (QOL). References: Michalski JM, Gay H, Jackson A, Tucker SL, Deasy JO. Radiation dosevolume effects in radiation-induced rectal injury. Int J Radiat Oncol Biol Phys. 2010 Mar1; 76(3 Suppl):S123-9. doi:10.1016/j.ijrobp.2009.03.078. Review. PubMed PMID: 20171506; PubMed Central PMCID: PMC3319467.

271. The most appropriate treatment after total parotidectomy for a T2N0M0 adenoid cystic carcinoma in a patient with facial droop is: (A) close observation. (B) radiation to the parotid bed and ipsilateral neck. (C) radiation to the parotid bed, ipsilateral neck and skull base. (D) concurrent chemotherapy and radiation.

Key: C Solution: Adenoid cystic carcinoma has a predilection for perineural invasion. Despite no pathologic evidence of perineural invasion on microscopy, the patient presented with a facial droop. Adenoid cystic carcinoma may have skip metastases along the nerve, and the facial droop suggests perineural invasion of the facial nerve. Perineural involvement of a named nerve should be treated with radiation therapy to the skull base. The role of chemotherapy in adenoid cystic carcinoma is unclear. Observation is not appropriate as this patient has perineural invasion. References: Gunderson and Tepper, Clinical Radiation Oncology, Chapter 35.

131. The MRC ASTEC trial for Stage I endometrial cancer demonstrated that pelvic lymphadenectomy: (A) improved OS and RFS. (B) improved OS but NOT RFS. (C) did NOT improve OS or RFS. (D) did NOT improve OS but improved RFS.

Key: C Solution: After a median follow-up of 37 months (IQR 24-58), 191 women (88 standard surgery group, 103 lymphadenectomy group) had died, with a hazard ratio (HR) of 1.16 (95% Cl 0.87-1.54;p=0.31) in favor of standard surgery and an absolute difference in 5- year overall survival of 1% (95% Cl - 4 to 6). 251 women died or had recurrent disease (107 standard surgery group, 144 lymphadenectomy group), with an HR of 1.35 (1.06- 1.73; p=0.017) in favor of standard surgery and an absolute difference in 5-year recurrence- free survival of 6% (1-12). With adjustment for baseline characteristics and pathology details, the HR for overall survival was 1.04 (0.74-1.45; p=0.83) and for recurrence-free survival was 1.25 (0.93-1.66; p=0.14). Interpretation - the authors reported that the results show no evidence of benefit in terms of overall or recurrence-free survival for pelvic lymphadenectomy in women with early endometrial cancer. They concluded that pelvic lymphadenectomy cannot be recommended as a routine procedure for therapeutic purposes outside of clinical trials. References: Lancet 2009: 373:125-36.

115. The Johns Hopkins rapid autopsy series (Iacobuzio-Donahue, JCO, 2009) examined patients who had died of pancreatic cancer. In this study, what was the rate of death with locally destructive pancreatic cancer? (A) 5% (B) 15% (C) 30% (D) 50%

Key: C Solution: At autopsy, 30% of patients died with locally destructive pancreatic cancer, and 70% died with widespread metastatic disease. These divergent patterns of failure found at autopsy (locally destructive v metastatic) were unrelated to clinical stage at initial presentation, treatment history, or histopathologic features. References: Iacobuzio-Donahue CA, Fu B, Vachida S, et al. "DPC4 Gene Status of the Primary Carcinoma Correlates With Patterns of Failure in Patients with Pancreatic Cancer." J Clin Oncol (2009) 27:1806-13.

118. What distinguishes Burkitt lymphoma from other non-Hodgkin lymphomas in children? (A) It is relatively indolent (B) The most common primary site of disease is the bone marrow (C) Tumor cells express the t(8;14) chromosomal translocation (D) It is four times more common among girls than boys

Key: C Solution: B-cell non-Hodgkin lymphoma-Burkitt lymphoma in children is a highly aggressive tumor. The most common primary sites of disease are the lymph nodes and abdomen. It is more common among boys than girls (approximately 4:1). References: http://dx.doi.org/10.1016/j.pcl.2014.09 .010. Pediatr Clin N Am 62 (2015) 139-165; Pediatr Clin N Am 62 (2015) 139-165.

223. Regarding thymoma, how would one classify a patient following surgery if the tumor was completely resected en bloc with microscopic capsular invasion with the neoplastic epithelia component scattered among a rich population of lymphocytes? (A) Modified Masaoka stage IIA thymoma, WHO type B1 histologic classification (B) Modified Masaoka IIIA thymoma, WHO type B2 histologic classification (C) Modified Masaoka stage IIA thymoma, WHO type B2 histologic classification (D) Modified Masaoka stage IIIA thymoma, WHO type B1 histologic classification

Key: C Solution: Based on Masaoka staging the patient was diagnosed with stage fibthymoma based on the microscopicinvasion into the capsule and WHO type A histologic type based on the tumor being ad mixed with foci rich in lymphocytes. Masaoka stage I is represented by tumors macroscopically completely encapsulated, with no microscopic capsular invasion. WHO type Bl histologic type is represented by the tumor resembling normal functional thymus; combines large expanses having an appearance practically in distinguishable from that of normal thymic cortex with areas resembling thymic medulla. Of anterior mediastinal masses, thymomas are the most common tumor, com prising 30% of the tumors in this location. Patients with Masaoka stage I thymoma have a 5-year survival rate of more than 90%. References: Perez and Brady's Principles and Practice of Radiation Oncology, Fifth edition, pages, 1109-1130.

183. The use of a flattening-filter free (FFF) photon beam, as compared to a photon beam with a flattening filter, will cause the PDD at 10cm to: (A) increase by 30%. (B) increase by 3%. (C) decrease by 3%. (D) decrease by 30%.

Key: C Solution: Because of the lack of the flattening filter and associated beam hardening, the x-ray photon beam energy fluence will be "softer" and lead to a decrease in the penetrability of the beam. References: Glide-Hurst C, et al. Med Phys 40(3). 2013.

48. A bone marrow biopsy would be most appropriate for which of the following clinical stages in patients with Hodgkin's lymphoma? (A) IA (B) IIA (C) IIB (D) IIIC

Key: C Solution: Bone marrow biopsy is recommended in patients who have clinical stages IB-IIB and III and IV Hodgkin's lymphoma.

191. A palliative, AP/PA radiation plan is created for a patient with large mediastinal mass and 30 Gy in 10 fractions. What acute complication is most likely if the fields incorporate the mediastinum, the whole heart, the lower trachea, carina and mainstem bronchi, 15% of total lung volume, and the lower 1/3 of the esophagus? (A) Bronchial stenosis (B) Radiation fibrosis (C) Radiation pericarditis (D) Esophageal stricture

Key: C Solution: Bronchial stenosis is a complication more commonly seen with SBRT courses to central lesions and is a late complication. Radiation pneumonitis is unlikely as the V20 would only be 15%. Per QUANTEC reports, the symptomatic pneumonitis rate when V20 is <30% is<20%. Esophageal stricture is a late complication, unlikely to be seen at this dose. Radiation to the whole heart is associated with high rate of radiation pericarditis, up to 46% in Hodgkin's Lymphoma data before the LV and subcarinal blocks were introduced in mantle fields. References: Gagliardi G, et al. IJROBP 2010 Mar; 76(3): 577-585. Carmel RJ and Kaplan HS. Cancer 1976; 37: 2813-2825.

285. Which of the following lymph node stations are removed during a D2 gastric cancer resection? (A) None (B) Perigastric (C) Perigastric and celiac (D) Perigastric, celiac, and para-aortic

Key: C Solution: D2 dissection is an extended lymphadenectomy, typically corresponding to perigastric nodes and nodes along the celiac artery and its branches. D1 dissection includes perigastric nodes only, and D3 dissection also includes the peri-aortic nodes. References: Songun I, Putter H, Kranenbarg EM, Sasako M, van de Velde Cl. Surgical treatment of gastric cancer: 15-year follow-up results of the randomised nationwide Dutch D1D2 trial. Lancet Oncol. 2010 May;11(5):439-49.

90. What is the MOST frequent cause of death among patients with bilateral retinoblastoma? (A) Locally recurrent retinoblastoma (B) Metastatic retinoblastoma (C) Second malignancy (D) Cardiovascular disease

Key: C Solution: Due to the presence of a germline RB1 mutation, patients with bilateral retinoblastoma are at elevated risk for second malignancies compared to patients with unilateral retinoblastoma, who have only a 10% chance of having a germline mutation. References: Shinohara, et al. PBC 2014.

69. What is the best treatment for a 28 year old woman with a 2 cm visible squamous cell carcinoma of the cervix extending onto the posterior vaginal fornix and no other evidence of disease on PETCT and MRI? (A) Brachytherapy alone (B) EBRT + brachytherapy (C) Modified radical hysterectomy and pelvic lymph node dissection (D) Loop electrosurgical excision procedure (LEEP) with negative margins

Key: C Solution: For young, healthy patients with stage FIGO IIA disease, radical surgery is preferred to radiotherapy. Main disadvantages of radiotherapy for this situation include risk of secondary malignancy, premature ovarian failure and suboptimal outcomes for sexual function. Excisional biopsy is not appropriate for stage II, as the tumor is demonstrating extension beyond the cervix. References: NCCN Clinical Practice Guidelines in Oncology.

81. For tomotherapy treatments, periodic QA tests of the couch need to include accuracy of: (A) position. (B) motion. (C) position and motion. (D) position, motion and weight limits.

Key: C Solution: Helical tomotherapy involves the delivery of overlapping fan-beams of radiation, which is achieved by rotating the gantry while the couch translates through the bore. Due to the helical delivery, the couch position and motion accuracy are therefore critical for accurate delivery. Weight limits are not tested as part of periodic QA. References: "QA for helical tomotherapy: reports of the AAPM Task Group 148" K Langen et al, Medical Physics 37:9; 2010.

141. What is the optimal time to deliver hyperthermia relative to radiation in order to achieve the greatest tumor radiosensitization? (A) Four hours before RT (B) One hour before RT (C) Simultaneously with RT (D) One hour after RT

Key: C Solution: Hyperthermia is most effective radiosensitizer if given at the time of radiation. However, the best therapeutic effective is achieved using sequential heat treatment rather than simultaneous treatment. Sequential treatment spares normal tissue injury because of differences in blood flow between tumor and normal tissues. References: Moyer HR, Delman KA. The role of hyperthermia in optimizing tumor response to regional therapy, Int J Hyperthermia, 24:251-261, 2008.; Horsman MR, Overgaard J. Hyperthermia: a potent enhancer of radiotherapy. Clin Oncol (R Coll Radiol). 2007 Aug; 19(6): 418-26

159. Which of the following signaling pathways is directly activated by hypoxia? (A) Ras signaling (B) NFB (C) HIF- (D) EGFR

Key: C Solution: Hypoxia activates HIF1a. The other pathways are either not activated or indirectly activated. References: https://en.wikipedia.org/wiki/HIF1A. Semenza GL. Targeting HIF-1 for cancer therapy. Nature Reviews. Cancer 3 (10): 721-32. Span PN, Bussink J. Biology of hypoxia. Semin Nucl Med. 2015 Mar; 45(2):101-9.

40. A nearly spherical lung lesion treated with EBRT has a prescription isodose volume (PIV) to target volume ratio (PITV) of 1.3. If the PIV is 148 cc, what is the estimated diameter of the target? (A) 2cm (B) 4cm (C) 6cm (D) 8cm

Key: C Solution: If the PIV is 148cc, the target volume is 114cc. From that volume (4/3 r^3) you can determine the radius (and then diameter) of the sphere. The estimated diameter for a sphere is D = ((3* ) / (32*V))^(1/3). References: PITV is originally defined in RTOG BR-0023.

215. Which of the following is influenced by the prevalence of disease in the population that is being tested? (A) Sensitivity (B) Specificity (C) Positive and negative predictive values (D) True positive and negative test

Key: C Solution: If we test in a high prevalence setting, it is more likely that subjects who test positive truly have disease than if the test is performed in a population with low prevalence. It is illustrated in below two examples, the total population is 100 subjects and there are same sensitivity and specificity but different prevalence of disease. Example 1: Prevalence of disease = (10+5)/100=15%. Disease Status Present Absent Test Results Positive 10 40 Negative 5 45 Sensitivity=l0/(10+5)=66.7%; Specificity=45/(40+45)=52.9%; Positive predictive values=l0/(10+40)=20%; Negative predictive values=45/(5+45)=90%. Example 2: Prevalence of disease = (20+10)/100=30%. Disease Status Present Absent Test Results Positive 20 33 Negative 10 37 Sensitivity=20/(20+ 10)=66.7%; Specificity=37 /(33+37)=52.9%; Positive predictive values=20/(20+33)=38%; Negative predictive values=37 /(10+37)=79%. Using the same test in a population with higher prevalence increases positive predictive value. Conversely, increased prevalence results in decreased negative predictive value. References: https://onlinecourses.science.psu.edu/stat507/node/71.

155. The use of smaller, <5 mm, MLC leaf widths for the delivery of IMRT treatments has been shown to be most beneficial for which type of target volume geometry? (A) Small, regularly-shaped targets (B) Large, regularly-shaped targets (C) Small, irregularly-shaped targets (D) Large, irregularly-shaped targets

Key: C Solution: Improvements in target coverage and gradient index were shown to exist when using a 2.5mm over a 5.0mm leaf width for treatment of small, irregularly-shaped targets. No improvements were noted for large, regularly or irregularly shaped targets between 2.Smm and 5.0mm MLC leaf widths. References: Chae SM, et al. Radiat Oncol. 9(72). 2014.

234. According to RTOG 91-11 (locally advanced laryngeal cancer), concurrent chemoRT was superior to sequential chemotherapy and radiation for which endpoint? (A) Laryngectomy free survival (B) DFS (C) LC (D) OS

Key: C Solution: In RTOG 91-11, concurrent chemoradiation had better outcomes than sequential treatment for laryngeal preservation, local control, and loco regional control. There was no statistically significant difference for laryngectomy-free survival, distant control, disease-free survival, and overall survival. References: Forastiere AA, et al. Long-term results of RTOG 91-11: a comparison of three nonsurgical treatment strategies to preserve the larynx in patients with locally advanced larynx cancer. J Clin Oncol. 2013 Mar 1; 31(7):845-52.

78. Which of the following bone sarcomas is considered to be the most radiosensitive? (A) Osteosarcoma (B) Chordoma (C) Ewing sarcoma (D) Chondrosarcoma

Key: C Solution: In contrast to osteosarcoma and other primary bone sarcomas, Ewing sarcoma is more radiosensitive, and radiation may be considered an effective option for local control. References: Uptodate.com.

95. What was the LRR at 10 years with and without PMRT in the Danish 82c (postmenopausal) breast trial? (A) 4% with RT versus 16% no RT (B) 6% with RT versus 23% no RT (C) 8% with RT versus 35% no RT (D) 12% with RT versus 46% no RT

Key: C Solution: In the Danish 82c study of high-risk postmenopausal breast cancer patients treated with mastectomy and one year of adjuvant tamoxifen, locoregional recurrence occurred in 8% of those receiving radiation and in 35% without radiation (p<0.001). References: Overgaard, et al. Lancet 1999.

67. What percentage of patients with inflammatory breast cancer present with metastatic disease? (A) 1-5% (B) 10-15% (C) 25-30% (D) 45-50%

Key: C Solution: In the NCCN outcomes database, 29% of patients with inflammatory breast cancer had metastatic disease at presentation, as did 25% of patients in the CDC population bases statistics review. References: Matro, et al. Clin Breast Cancer 2015 and Wingo, et al. Cancer Causes Control 2004.

143. What is the median survival (months) in patients with MGMT methylation who received combined temozolomide and RT in the EORTC-NCIC trial by Stupp et al.? (A) 12 (B) 15 (C) 23 (D) 30

Key: C Solution: In the update of the EORTC-NCIC study published in Lancet in 2009,the median survival was 23.4 months in those with MGMT hypermethylation who received combined chemoRT. References: http://www.ncbi.nlm.nih.gov/pubmed/19269895.

185. "Incident learning" is defined as learning from: (A) clinical trial results in order to improve treatment practices. (B) published research in order to improve treatment practices. (C) errors and near misses in order to improve patient safety. (D) hospital based quality improvement initiatives in order to improve clinical outcome.

Key: C Solution: Incident learning is a recommended safety strategy by ASTRO and AAPM and is more commonly being adopted in radiation oncology. References: Thomadsen B, et al. Chapter 6 of Quality and safety in radiotherapy: Learning the new approaches in Task Group 100 and Beyond, proceedings of the 2013 AAPM Summer School. https://www.astro.org/uploadedFiles/Main_Site/Clinical_Practice/Patient_Safety/Blue_Book /SafetyisnoAccident.pdf.

259. While high LET radiation is more effective against radioresistant tumors, what is the major disadvantage of high LET radiation? (A) Elevated exit dose (B) Lack of skin sparing (C) More damaging to normal tissue (D) High scatter dose

Key: C Solution: Less tissue repair after High LET radiation due to higher proportion of lessrepairable complex DNA breaks, hence higher toxicity. References: Goodhead DT. Mechanisms for the biological effectiveness of high-LET radiations. J Radiat Res. 1999 Dec; 40 Suppl: 1-13. Skarsgard LD. Radiobiology with heavy charged particles: a historical review. Phys Med. 1998 Jul;14 Suppl: 1:1-19.

4. Which cell types are most likely to die by apoptosis following 2 Gy of external beam radiation? (A) Post-synaptic motor neurons (B) Invasive ductal carcinoma cells (C) CD8+ T-lymphocytes (D) Type I pneumocytes

Key: C Solution: Lymphoid cells are particularly predisposed to apoptotic death following radiation. Epithelial tumors, such as breast cancer, primarily respond via mitotic cell death. Other somatic cell types are not as likely to undergo apoptosis as are lymphoid cells. References: Hall and Giacca. Radiobiology for the Radiologist, 7th Edition. Chapter 3.

110. What is the most common site of heterotopic ossification formation? (A) Elbow (B) Knee (C) Hip (D) Shoulder

Key: C Solution: Most commonly, HO is seen around the hip after fracture and open reductioninternal fixation (ORIF) procedures or total hip arthroplasties (THA). References: Balboni TA1, Gobezie R, Mamon HJ. Int J Radiat Oncol Biol Phys. 2006 Aug 1; 65(5):1289-99. Heterotopic ossification: Pathophysiology, clinical features, and the role of radiotherapy for prophylaxis.

17. What are the corresponding nodal stations for a patient with a left upper lobe NSCLC and pathologic involvement of the ipsilateral hilar and lower paratracheal lymph nodes? (A) 10,7 (B) 12,5 (C) 10,4 (D) 12,2

Key: C Solution: Nodal stations 10-14 are classified as Nl, nodal stations 1-9 are classified as N2. References: Rusch, et al. J Thorac Oncol. 2009 May; 4(5):568-77. The IASLC lung cancer staging project: a proposal for a new international lymph node map in the forthcoming seventh edition of the TNM classification for lung cancer.

54. What is the measure of association between an exposure and an outcome in a case controlled study called? (A) Relative risk (B) Risk factor (C) Odds ratio (D) Sensitivity

Key: C Solution: Odds ratio = (exposed cases/unexposed cases) / (exposed noncases/ unexposed non-cases). References: Szumilas, M. (2010). Explaining Odds Ratios. Journal of the Canadian Academy of Child and Adolescent Psychiatry, 19(3), 227-229.

96. What elements in the history and physical examination may suggest a thymoma for a patient with a large anterior mediastinal mass? (A) 10% Weight loss (B) Dehydration (C) Early fatigability (D) Family history of cancer

Key: C Solution: Paraneoplastic syndrome related to thymoma is commonly, which includes Myasthenia gravis (MG), and hallmarks are early fatigability due to proximal muscle weakness that worsens with movement, as well as diplopia. Weakness in the eye lid muscles and ocular muscles are one of the first signs of MG. Other selection, such as weight loss, is nonspecific, but may point to possibly lymphoma or lung cancer. Dehydration may point to SIADH, associated with neuroendocrine cancers. Family history may elicit familial cancers, such as testicular germ cell tumors. References: Devita, 6th edition

14. Per the RTOG Breast Contouring Atlas, what are the anatomical boundaries of the internal mammary lymph nodes when contouring breast regional nodal treatment volumes? (A) Superior aspect of the medial first rib to the cranial aspect of the third rib (B) Inferior aspect of the medial first rib to the cranial aspect of the third rib (C) Superior aspect of the medial first rib to the cranial aspect of the fourth rib (D) Inferior aspect of the medial first rib to the cranial aspect of the fourth rib

Key: C Solution: Per the RTOG Breast Contouring Atlas, the anatomical boundaries for the IMNs extend from the superior aspect of the medial first rib to the cranial aspect of the fourth rib. References: White, et al. RTOG Breast Contouring Atlas.

230. What is the biological rationale for using SBRT for prostate cancer? (A) Hypoxia (B) Rapid repopulation (C) (D) High rate apoptosis

Key: C Solution: Prostate cancer has low a/ratio, thus more sensitive to high dose per fraction of SBRT.

187. Which of the following agents improves OS for patients with castrate resistant metastatic prostate? (A) Iodine (B) Strontium (C) Radium (D) Samarium

Key: C Solution: Radium (Ra 223) dichloride is an alpha emitter with tropism for bone. Unlike the beta-emitters, Strontium and Samarium, Radium dichloride has demonstrated an increase in survival by more than 30% (14.9 months in Radium dicloride-treated men vs 11.3 months in placebo-treated men). The primary indication is castrate resistant prostate cancer, those that have had disease progression after medical or pharmacologic castration and are either refractory or intolerant of Docetaxel. It is not indicated for use in patients with visceral disease. It is ideally used for patients with multiple symptomatic metastases and can be used concurrently with focal external beam radiotherapy. References: N Engl J Med 2013; 369:213-223.

22. Which structure runs through the cavernous sinus? (A) Cranial nerve VII (B) Cranial nerve VIII (C) Internal carotid artery (D) Pineal gland

Key: C Solution: Rationale is not given for this question. References: Gunderson LL, Tepper JE, Clinical Radiation Oncology 3.0 ed.

254. NCCN Guidelines recommend genetic screening for any breast cancer patient with no family history of breast or ovarian cancer when they are under the age (years) of: (A) 35 (B) 40 (C) 45 (D) 50

Key: C Solution: Recommendation for genetic screening varies with the patient's personal breast cancer history and family history. Screening is recommended when: No family history of breast or ovarian cancer AND patient's breast cancer is diagnosed at 45 years or younger. One close blood relative (first, second or third degree relative) with breast cancer under the age of 50 and/or with one close blood relative with epithelial ovarian cancer at any age AND patient's breast cancer is diagnosed at age 50 or younger. Limited family history AND patient's breast cancer is diagnosed at age 50 or younger. Triple negative breast cancer AND the patient's breast cancer is diagnosed at age 60 or younger. References: NCCN Guidelines - Genetic/Familial High Risk Assessment: Breast and Ovarian Cancer.

260. A FIGO Stage III of the vagina involves the: (A) rectum. (B) bladder. (C) pelvic LNs. (D) paravaginal tissues but not the pelvic side wall.

Key: C Solution: Rectum and Bladder are FIGO Stage IVA, regional pelvic or inguinal nodes are FIGO Stage Ill, Paravaginal but not pelvic side wall disease is FIGO Stage II. References: Vaginal Cancer Staging: TNM and FIGO Staging for Vaginal Cancer.

26. Which of the following tests should be used in the analysis of a continuous independent variable and a continuous dependent variable? (A) t-test (B) Chi Square (C) Regression (D) ANOVA

Key: C Solution: Regression is used for the above analysis. The T-test and ANOVA involve the analysis of a nominal independent variable and a continuous dependent variable. The Chi Square analysis is involved in the analysis of the nominal independent variable and the nominal dependent variable.

8. What is a major disadvantage of using historical controls as a comparison group for evaluating a new treatment? (A) Limited power (B) Unequal sample sizes (C) Selection bias (D) Large treatment effect

Key: C Solution: Selection bias in using historical controls often arises due to differences in disease heterogeneity, known and unknown prognostic factors. Power is a function of sample sizes in the historical control and treatment comparator groups. Sample sizes in the historical control and treatment comparator groups do not have to be the same to achieve high power for detecting a clinically significant difference. A large effect will actually strengthen a treatment comparison using historical controls.

45. What is the correct T stage for a patient with Mycosis Fungoides that has one or more tumors (1 cm in diameter)? (A) T1 (B) T2 (C) T3 (D) T4

Key: C Solution: T1-limited patches, plaques < 10% of skin surface T2-patches, plaques covering 10% of skin surface; T3-one or more tumors (cm in diameter) T4-confluence of erythema 80% body surface area. References: Edge S, Byrd D. AJCC in staging manual 2010.

284. In gallbladder cancer, which tumor characteristic qualifies as T3 disease? (A) Invasion of lamina propria (B) Invasion of muscularis (C) Invasion of the liver (D) Invasion of hepatic artery

Key: C Solution: T1a: Tumor invades lamina propria. T1b: Tumor invades muscular layer.T2: Tumor invades perimuscular connective tissue; no extension beyond serosa or into liver.T3 : Tumor perforates the serosa (visceral peritoneum) and/or directly invades the liver and/or one other adjacent organ or structure, such as the stomach, duodenum, colon, pancreas, omentum, or extrahepatic bile ducts.T4: Tumor invades main portal vein or hepatic artery or invades two or more extrahepatic organs or structures. References: Edge SB, Byrd DR, Comptom CC, Fritz AG, Greene FL, Trotti A (Eds.). (2010) AJCC Cancer Staging Manual, Seventh Edition, New York, NY: Springer.

52. In ECOG 5194 (Local Excision Alone Without Irradiation for DCIS of the Breast), which features were associated with higher risk of ipsilateral breast events? (A) Lesion size and margin width (B) Nuclear grade and lesion size (C) Nuclear grade and patient age (D) Clinical mass vs. mammographic detection

Key: C Solution: The 5 year rates of ipsilateral breast events (IBE) in relation to ndividual patient, tumor and treatment characteristics were analyzed with a Cox proportional hazards model with grade as the only covariate (low/intermediate vs. high). Grade was significantly related to the risk of IBE (p=0.024).Separate models were created to examine the impact of other features on risk of IBE. Lesion size, margin width, age at last surgery, intent to use tamoxifen or not and tamoxifen use or not were analyzed. The only significant variable was age in the high-grade group, with a hazard ratio of 0.95. (95% Cl 0.91-0.99, p=0.016). References: Hughes L, et al. Local Excision Alone Without Irradiation for Ductal Carcinoma In Situ of the Breast: A Trial of the Eastern Cooperative Oncology Group. J. Clin. Oncol 27:5319-5324, 2009.

162. What, if any, adjuvant therapy should be recommended based on level 1 evidence for a lung cancer patient with 3 cm adenocarcinoma with negative margins with involvement of one level 2 and one level 4 lymph node? (A) Concurrent chemoRT using weekly carboplatin and paclitaxel (B) Cisplatin and Pemetrexate (C) Cisplatin and Vinorelbine (D) Concurrent chemoRT using weekly Cisplatin

Key: C Solution: The ANITA trial is an important trial that used Cisplatin in combination with Vinorelbine demonstrating a benefit in survival. Concurrent strategies have been overall neatie in the adjuvant setting have been shown to be safe and efficacious in phase II studies but have failed to demonstrate benefit a benefit in a phase 3 setting (Bradley, ECOG). Cisplatin Pemetrexate is commonly used in the adjuvant setting although the data is limited to the metastatic setting. References: Douillard, et al. Int J Radiat Oneal Biol Phys. 2008 Nov 1; 72(3):695-701. Impact of postoperative radiation therapy on survival in patients with complete resection and stage I, II, or IIIA non-small- cell lung cancer treated with adjuvant chemotherapy: the adjuvant Navelbine International Trialist Association (ANITA) Randomized Trial.

122. Which profile of a patient with cirrhosis and HCC is BEST suited for liver transplantation? (A) Child Pugh class A, 3 cm tumor andportal vein tumor thrombus (B) Child Pugh class A, and a single 7 cm tumor (C) Child Pugh class B, and a 4 cm tumor (D) Child Pugh class A, 3 tumors measuring 3.5 cm, 2 cm, and 3.5 cm

Key: C Solution: The Milan criteria for liver transplantation for hepatocellular carcinoma are 1) single tumor 5 cm; or 2) no more than 3 lesions without and single lesion measuring > 3 cm. The UCSF criteria are 1) single tumor <6.5 cm; 2) maximum of 3 total tumors with none >4.5 cm; and 3) cumulative tumor size <8 cm. Patients with tumor thrombus from HCC have worse outcome and are not well-suited for liver transplant. References: Mazzaferro V, Regalia E, Doci R, et al. Liver transplantation for the treatment of small hepatocellular carcinomas in patients with cirrhosis. N Engl J Med. 1996; 334: 693-699. Yao FY, Ferrell L, Bass NM, et al. Liver transplantation for hepatocellular carcinoma: expansion of the tumor size limits does not adversely impact survival. Hepatology. 2001; 33: 1394-1403.

108. Regarding the analysis of the National Cancer Database Registry for Early Stage Classical Hodgkin Lymphoma: (A) only about 20% of the patients received combined modality treatments from 2003-2011. (B) uninsured patients had the same odds of receiving combined modality treatment. (C) combined modality treatment was associated with better OS. (D) showed higher rates of second cancers for combined modality treatments.

Key: C Solution: The NCDB analysis included 20,600 patients with HL treated from 2003-2011 and found that only 49.5% were treated with CMT. Uninsured patients were less likely to received CMT. There was significant improvement in OS with combined modality treatment. The authors concluded that "widespread" abandonment of CMT beyond circumstances sanctioned by guidelines may affect survival. References: Olszewski A, et al. Journal of Clinical Oncology. 2015. 33(6):625-33.

267. Upper endoscopy performed for dysphagia visualizes a mass centered in the gastric cardia with its proximal edge involving the gastroesophageal junction. What is the most appropriate anatomic classification for this tumor? (A) Siewert Type I (B) Siewert Type II (C) Siewert Type III (D) Gastric cardia tumor

Key: C Solution: The Siewert classification for tumors of the GE junction is based on whether the epicenter of the tumor is proximal to the GEJ (Type I), directly at the GEJ (Type II), or distal to the GEJ (Type Ill). Tumors in the gastric cardia but not involving the GEJ would not fall under the Siewert classification. References: Siewert JR, Stein HJ. Adenocarcinoma of the gastroesophageal junction: classification, pathology and extent of resection. Dis Esoph 1996; 9:173-182.

281. How will reducing the dose rate of low LET radiations from 1000 cGy per minute to 0.5 cGy per minute change the - parameters in the linear quadratic survival model? (A) will increa (B) (C) (D)

Key: C Solution: The alpha-beta survival model is based on the Theory of Dual Radiation Action. This theory postulates the pairwise interaction of spatially and temporally proximate sublesions to form the lethal lesions that ultimately kill the cell. A fraction of these lethal lesions form when both sublesions result from a single energy adsorption event (e.g. single electron track). This process produces the linear (alpha) component of the dose response. Alpha type events are dose rate independent and dominate at low doses and dose rates. The quadratic portion of the dose response (Beta) results from sublesions that are formed by independent events (multiple electron tracks). Since the number of sublesions themselves is directly proportional to radiation dose, reducing the dose rate spreads out the production of sublesions over time allowing a fraction of them to be repaired before the formation of the second proximate sublesions required for lethal lesion creation occurs. By lowering the dose rate, alpha (single-hit) events remain constant while beta (multi-hit) events decrease.

84. What were the volumes and doses for PMRT in the British Columbia trial of node positive premenopausal breast cancer patients receiving mastectomy and adjuvant chemotherapy +/- PMRT? (A) Chest wall, axillary and supraclavicular nodes, and ipsilateral internal mammary lymph nodes; 37.5 Gy and 35 Gy in 16 fx (B) Chest wall, axillary and supraclavicular nodes and ipsilateral internal mammary lymph nodes; 50 Gy and 45 Gy in 25 fx (C) Chest wall, axillary and supraclavicular nodes and bilateral internal mammary lymph nodes; 37.5 Gy and 35 Gy in 16 fx (D) Chest wall, axillary and supraclavicular nodes and bilateral internal mammary lymph nodes; 50 Gy and 45 Gy in 25 fx

Key: C Solution: The chest wall was treated to 37.5 Gy in 16 fractions and the supraclavicular/axillary nodes and bilateral internal mammary nodes to 35 Gy in 16 fractions. References: Ragaz, et al. JNCI 2005; Reference: Ragaz et al, NJEM 1997.

288. Compared to a normal-incidence beam, an oblique-incidence photon beam is characterized by: (A) decreased surface dose and decreased dmax . (B) decreased surface dose and increased dmax. (C) increased surface dose and decreased dmax. (D) increased surface dose and increased dmax.

Key: C Solution: The entrance (surface) dose for a therapeutic photon beam is less than the dose at depth because only a small population of scattered electrons have been created near the beam entry point to deposit dose. The depth of maximum dose occurs roughly where these electrons are being added to and removed from the beam in equal numbers. For oblique beams, electron scatter can come from a greater area, as these points near the surface are now effectively downstream of lateral points of beam entry. This extra scatter contribution both increases the surface dose and produces the equilibrium of maximum dose at a more shallow depth. References: Khan FM. "Physics of Radiation Therapy, 4th Edition", section 13.3.

33. What is the benefit of a short course of androgen deprivation therapy at five years (D'Amico, JAMA 2004) in the definitive treatment of intermediate-risk prostate cancer patients treated with radiation therapy? (A) 5% OS (B) 5% biochemical recurrence free survival (C) 10% OS (D) 10% biochemical recurrence free survival

Key: C Solution: The five-year overall survival in the D'Amico trial was 88% with and 78% without a short course of ADT. This was statistically significant with p=0.04. References: D'Amico AV, Manola J, Loffredo M, Renshaw AA, DellaCroce A, Kantoff PW. Six-month androgen suppression plus radiation therapy vs radiation therapy alone for patients with clinically localized prostate cancer: a randomized controlled trial. JAMA 2004; 292(7): 821-7.

163. In which of the following delivery techniques is the radiation paused during gantry and leaf motion? (A) Arc Therapy (B) Helical Tomotherapy (C) Step-and-shoot intensity modulation (D) Volumetric Modulated Arc Therapy

Key: C Solution: The step-and-shoot method of IMRT delivery is done with the MLC making different shapes while the gantry is not moving. The linac is paused between the leaf motion. References: Khan FM. The Physics of Radiation Therapy, Third Ed, Lippincott Williams & Wilkins, New York, 2003, pp 481-505.

241. Which of the following chromosomal translocations is associated with Ewing Sarcoma? (A) t(11;14) (B) t(2;13) (C) t(11;22) (D) t(9;22)

Key: C Solution: The t(11;22) with EWS-FL1 fusion transcript is associated with Ewing sarcoma. Option A is seen in multiple myeloma. The t(2;13) with the PAX3-FKHR fusion transcript is associated with rhabdomyosarcoma. Option D is seen in chronic myelogenous leukemia (CML). References: Halperin, et al, Pediatric Radiation Oncology.

9. In planning craniospinal fields for a pediatric patient, the bottom of the photon field is most typically between which vertebral body levels? (A) Mid-L2 to mid-L4 (B) Mid-L5 to mid-S2 (C) Mid-S2 to mid-S4 (D) S5 to coccyx

Key: C Solution: The thecal sac typically terminates between the top and bottom of S2, and the bottom of the craniospinal field is typically set 2-3 cm below that. References: Dunbar, et al. IJROBP. 26(4): 669-673.

178. Which of following shielding calculation factors represents the ratio of time that a particular barrier is being irradiated? (A) Primary design goal (P) (B) Occupancy factor (T) (C) Use factor (U) (D) Workload (W)

Key: C Solution: The use factor is the relative amount of time a barrier is irradiated. The primary design goal is the permissible dose equivalent for the area to be protected. The occupancy factor is the fraction of time during which the area under consideration is occupied. The workload is the amount of radiation per unit time that is expected to be delivered at isocenter. References: Khan FM. The Physics of Radiation Therapy, Third Ed, Lippincott Williams & Wilkins, New York, 2003, pp 407.

146. Which presenting prognostic factor has the greatest impact on the post-RT ambulatory rate for patients with NSCLC presenting with spinal cord compression? (A) Age over 72 (B) Motor Deficits developed over 5 days (C) Paraplegic at presentation (D) Cord compression occurred 2 years following initial diagnosis

Key: C Solution: This multinational retrospective experience of over 2000 patients developed a predictive system for predicting post-radiotherapy ability to walk and 6 month overall survival. Variables found to be important in predicting post-radiation abulation was tumor type, time interval to motor deficits, time to develop motor deficits as well as the most the extent of the deficit at presentation. References: Rades, et al. IJROBP 2008.

227. What is the typical energy range used in proton therapy? (A) < 5 MeV (B) 5-20 MeV (C) 50-250 MeV (D) 300-550 MeV

Key: C Solution: To obtain a Bragg peak at clinically useful depths, the typical energy range used for proton therapy is in the tens to 250 MeV. References: Ma CM, and Lomax T, eds. Proton and carbon ion therapy. CRC Press, 2012.

219. Which chromosomal aberration is MOST likely to result in VIABLE daughter cells? (A) Ring (B) Anaphase bridge (C) Translocation (D) Dicentric

Key: C Solution: Translocations are of high incidence, and particularly when balanced viability is high. The other chromosomal aberrations most commonly do not result in viable daughter cells, though exceptions do exist. References: Matsumoto K, Ramsey MJ, Nelson DO, Tucker JD. Persistence of radiationinduced translocations in human peripheral blood determined by chromosome painting. Radiat Res. 1998 Jun. Tucker JD. Low-dose ionizing radiation and chromosome translocations: a review of the major considerations for human biological dosimetry. 149(6):602-13. Cornforth MN. Analyzing radiation-induced complex chromosome rearrangements by combinatorial painting. Radiat Res. 2001 May; 155(5):643-59.

152. Which is considered an unfavorable or high risk factor (per RTOG 9802), when evaluating a patient with a low grade glioma? (A) Pre-operative neurologic symptoms (B) Size greater than 6 cm (C) Subtotal resection (D) Age greater than 30 years

Key: C Solution: Unfavorable or high risk features for inclusion on RTOG 9802 were 1. Subtotal resection or biopsy or 2. Age 40 or more (regardless of resection).Median overall survival improved from 7.8 years with 54 Gy to 13.3 years with 54 Gy followed by PCV chemotherapy. Size and pre-operative neurologic symptoms are among the LGG poor prognostic factors per EORTC (Pignatti JCO 20, 2002. PMID 11956268). References: Initial report for RTOG 9802: Shaw JCO 30, 2012 for PCV arms. PMID 22851558. Final report of 9802 is unpublished as of July 2015, but refer to: Laack Sem Rad One 25, 2015. PMID 26050590.

252. For a 6 MV X-ray beam, photons scattered at 90 degrees with respect to the beam direction will have an approximate energy of: (A) 2.5 MeV. (B) 1 MeV. (C) 0.5 MeV. (D) 0.25 MeV.

Key: C Solution: When a therapeutic-energy photon scatters from an electron in a patient, the energy of the incident photon is shared by the scattered photon and the recoiling electron. The kinematics of Compton scattering dictate that forward scattered photons will only minimally transfer energy to the electrons and have something close to the initial photon energy. Photons scatter at 90 degrees will have an energy roughly equivalent to the mass of the electron (roughly 0.5 MeV). A photon backscattered at 180 degrees will have an energy roughly equal to the mass of the electron divided by 2, or 0.25 MeV. This is true regardless of the initial energy of the incident photon. This scattering effect has implications for how lateral shielding should be constructed, as vault walls with no direct beam only need to attenuate these lower-energy photons. References: Khan FM. "Physics of Radiation Therapy, 4th Edition."

292. What is the nodal stage for a sentinel lymph node with 0.5 mm of invasive carcinoma present? (A) pN0(mol+) (B) pN0(i+) (C) pN1mic (D) pN1a

Key: C Solution: pN1mic disease is presence of micrometastasies greater than 0.2 mm and/or more than 200 cells, but not greater than 2.0 mm. References: AJCC 7th edition.

167. Of patients on ACOSOG Z0011 for whom RT records were available, approximately what percentage received whole breast RT? (A) <60% (B) 65-75% (C) 80-90% (D) >90%

Key: C & D NOTE: This item was multi-#Keyed for scoring purposes upon post-exam statistical item analysis (Both C and D were given credit as correct responses). Solution: Rationale: In this report of RT field design on the Z11 trial, 89% of patients received whole breast RT. Despite not allowable on trial, 15% of patients received nodal radiation. Detailed RT treatment information was available for 228 patients, of whom 81% received whole breast RT using tangents fields. From these patients, specific RT fields were reviewed in 142, and high tangent fields were utilized in 50%. References: Jagsi et al. JCO 2014.

236. Phosphorylation of serine 139 on H2AX is a molecular biomarker of: (A) rejoined DNA single strand breaks. (B) broken DNA single strand breaks. (C) rejoined DNA double strand breaks. (D) broken DNA double strand breaks.

Key: D References: Burma S, Chen BP, Murphy M, Kurimasa A, Chen DJ., The Journal of Biological Chemistry, (2001) ATM phosphorylates histone H2AX in response to DNA doublestrand breaks. 2001 Nov 9; 276(45):42462-7. Pouliliou S, Koukourakis MI. Biomarkers, (2014) -H2AX) as a predictive tool in radiation oncology. 2014 May; 19(3):167-80. Jakob B, Durante M. Radiation Research, (2012) Radiation dose detection by imaging response in biological targets. 2012 Apr; 177(4):524-32.

297. Which toxicity is commonly associated with mitomycin-C in treatment of anal cancer? (A) Nausea (B) Diarrhea (C) Hand Foot Syndrome (D) Hematologic

Key: D References: NCCN.

201. For TBI used as conditioning prior to hematopoietic stem cell transplant, which fractionation scheme provides myeloablative conditioning with the lowest toxicity? (A) 2 Gy given in 1 fraction in 1 day (B) 4 Gy given in 2 fractions in 1 day (C) 10 Gy given in 1 fraction in 1 day (D) 14 Gy given in 8 fractions in 4 days

Key: D Solution: 2 Gy in 1 fraction and 4 Gy in 2 fractions are non- myeloablative regimens. 10 Gy in 1fraction and 14 Gy in 8 fractions are myeloablative schemes, but hyperfractionation reduces acute and chronic toxicities. References: Wolden, et al. American College of Radiology (ACR) and American Society for Radiation Oncology (ASTRO) Practice Guideline for the Performance of Total Body Irradiation (TBI); American Journal of Clinical Oncology 36(1) 2013.

218. What is the MOST common location of adult soft tissue sarcomas? (A) Upper extremity (B) Head and neck (C) Retroperitoneum (D) Lower extremity

Key: D Solution: 4550 adults reviewed by the American College of Surgery was as follows: Thigh - 46 percent Upper extremity - 13 percent Torso - 18 percent Retroperitoneum - 13 percent Head and neck - 9 percent References: Uptodate.com.

130. What is the total dose (Gy) of radiation used to treat a clinical stage I, 2 cm squamous cell carcinoma located in the lateral mid vagina? (A) 40-45 (B) 50-55 (C) 60-65 (D) 70-75

Key: D Solution: A total radiation dose of at least 70 to 75 Gy is generally recommended, with 40 to 50 Gy in 20-25 fractions being delivered with EBRT and the additional radiation being delivered with intracavitary or interstitial brachytherapy, depending on the thickness of the primary tumor. The EBRT should include the pelvic lymph nodes, vaginal tumor with a margin, vagina, and paravaginal tissues (and inguinal lymph nodes if the vaginal tumor is in the lower half of the vaginal canal). Brachytherapy should immediately follow the completion of external radiation. Residual vaginal tumors less than 5 mm thick can be potentially treated with vaginal cylinder while tumors thicker than 5 mm require an interstitial treatment for adequate dose. The location of the lesion often impacts the recommended treatment approach. Lesions in the mid vagina if anterior or lateral may be considered for an interstitial implant whereas those on the posterior wall are treated using EBRT for the boost to minimize risk of bowel toxicity associated with a posterior interstitial implant. References: Frank, et al: Definitive radiation therapy for squamous cell carcinoma of the vagina. Int. J. Radiation Oncology Biol. Phys., Vol 62, No.1, pp. 138-147, 2005.

160. Which patient selection criterion is consistent with ASTRO guideline for hypofractionated whole breast RT? (A) Age 40 or older at diagnosis (B) pT1N1 disease (C) Has received chemotherapy (D) Dose inhomogeneity no more than +/- 7%

Key: D Solution: ASTRO guidelines support hypofractionated regimens if patients satisfy all of the following criteria: the patient is 50 years of age or older at diagnosis, the pathologic stage is T1-2N0 and the patient is treated with breast conserving surgery, the patient has not been treated with systemic chemotherapy, and within the breast along the central axis, the minimum dose is no less than 93% and maximum dose is no greater than 107% of the prescription dose. References: Smith B, et al. Fractionation for whole breast irradiation: An American Society for Radiation Oncology (ASTRO) Evidence-Based Guideline. Int. J Radiation Oncology Biol. Phys, Vol 81, No.1, pp. 59-68, 2011.

294. Based upon NCCN guidelines, in which of the following asymptomatic patients is a bone scan MOST appropriate for a workup of his newly diagnosed prostate cancer? (A) Tlc, Gleason 7 (4+3), PSA = 4.8 ng/ml (B) T2a, Gleason 7 (3+4), PSA = 8.6 ng/ml (C) Tlc, Gleason 6 (3+3), PSA = 17.2 ng/ml (D) T2a, Gleason 6 (3+3), PSA= 11.6 ng/ml

Key: D Solution: According to current NCCN guidelines, Bone Scans are recommended in the work up of prostate cancer for any of the following criteria: T1 disease with PSA > 20ng/ml,T2 disease with PSA > 10 ng/ml, a Gleason score of < 8, T3 or T4 disease, or symptomatic. Studies have shown that there is overuse of bone scans in patients with lowand intermediate-risk prostate cancers, while underuse use in high-risk patients. References: IJOBP (2014) 89(2):243-248.

79. When treating a metastatic liver lesion with SBRT what is an appropriate dose metric for the normal liver parenchyma? (A) V15 Gy (B) Mean liver dose (C) V30 Gy (D) Dose to 700 cc

Key: D Solution: According to the proper dose constraint use for liver planning when performing SBRT is to spare 700 cc of normal liver from receiving more than 15 Gy. The other choices exceed this constraint. The 30 Gy is considered the liver tolerance dose using conventional fractionation. References: Dawson LA, Ten Haken RI, Lawrence TS. Partial irradiation of the liver. Semin Radiat Oncol. 2001 Jul; 11(3):240-6. Guha C, Kavanagh BD. Hepatic radiation toxicity: Avoidance and amelioration Semin Radiat Oncol, 21 (2011), pp. 256-263.

176. Per QUANTEC, which statement is CORRECT for the risk of radiation pneumonitis in NSCLC treated with conventional fractionation? (A) 30% of radiation pneumonitis occurs within 10 months (B) Limiting the V20 < 40% will limit the risk of radiation pneumonitis to 20% (C) Limiting the dose to the central airways to 80 Gy did not reduce stricture rates (D) Limiting the mean lung dose to 20 Gy will limit the risk of radiation pneumonitis to 20%

Key: D Solution: Approximately 80% of RP is clinically manifest within 10 months of RT. There is no clear and consistent "thresholds" for candidate metrics (i.e., the response function is often gradual), and the "acceptable" risk level varies with the clinic scenario. Despite these caveats, it is prudent to limit V20 to 30-35% and MLD to 20-23 Gy (with conventional fractionation) if one wants to limit the risk of RP to 20% in definitively treated patients with non-small-cell lung cancer. Limiting the dose to the central airways to 80 Gy may reduce the risk of bronchial stricture. References: Marks, et al. Radiation dose-volume effects in the lung. Int J Radiat Oncol Biol Phys. 2010 Mar 1; 76(3 Suppl): 570-66.

6. How is the Multi-Target/Single Hit model for cell survival superior to the linearquadratic model? (A) It provides a better fit to the data at low doses (B) It has better predictive value for hyperfractionation (C) The ratios of D0 for normal and tumor tissue help define treatment outcomes (D) It provides a better fit to the data at very large single doses

Key: D Solution: At low dose the multi-target/single hit (MTSH) model produces a zero initial slope which fits the data rather poorly. Hyperfractionation assumes dose rate effects which the MTSH model ignores. Alpha-Beta ratios are used to predict treatment outcomes. MTSH does provide a better fit to the data at high does where the survival curve is exponential.

172. Radiation treatment for basal cell carcinoma of the skin: (A) typically includes elective nodal coverage. (B) has loco-regional failure rates of about 30% when used as the primary treatment (C) is ineffective when the cancer has mutations in the Hedgehog pathway. (D) requires margins of 1 to 1.5 cm for lesions smaller than 2 cm in size.

Key: D Solution: Basal cell carcinoma rarely metastasizes to lymph nodes. Failure rates after primary radiation treatment in most retrospective reviews are around 10% or less. Hedgehog pathway mutations are common in basal cell carcinoma and Hedgehog pathway inhibitors have shown effectiveness in metastatic disease. Hedgehog pathway mutations have not been associated with radiation resistance. For lesions smaller than 2 cm in size, the margins are 1 to 1.5 in NCCN guidelines. References: Locke J, Karimpour S, Young G, Lockett MA, Perez CA. Radiotherapy for epithelial skin cancer. Int J Radiat Oncol Biol Phys. 2001 Nov 1; 51(3):748-55. NCCN guidelines Basal Cell Carcinoma V 1.2015.

41. According to the data from Princess Margaret Hospital, the use of primary radiation therapy alone for a never-smoker with locally advanced HPV-mediated oropharyngeal cancer results in local control of: (A) <50%. (B) 70%. (C) 80%. (D) >90%.

Key: D Solution: Based on data from Princess Margaret Hospital, radiation alone (using altered fractionation) resulted in high rates of cure. Among minimal smokers, both the radiationalone and chemoradiation subsets demonstrated universally exemplary cancer-related outcomes although the CRT cohort seemed to have slightly higher late toxicity: RT-alone vs. CRT: OS 86% vs. 88%, p=0.45; CSS 92% (95% Cl: 82-100) vs.93% (86-99), p=0.314; LC 95% vs. 92%, p=0.512;RC 97% vs. 93%, p=0.219; DC 92% vs. 86%, p=0.365; and late toxicity 6% (2-22) vs. 16% (9-28), p=0.081. References: O'Sullivan B, Huang SH, Perez-Ordonez M, et al. Outcomes of HPV-related oropharyngeal cancer patients treated by radiotherapy alone using altered fractionation. Radiother Oncol. 2012 Apr; 103(1):49-56. doi:10.1016/j.radonc.

60. Over which interval in months should a follow-up skeletal survey for patients treated for a solitary plasmacytoma be performed? (A) 1-3 (B) 3-6 (C) 6-9 (D) 9-12

Key: D Solution: Bone or skeletal survey should be obtained annually after treatment. References: http://www.nccn.org/professionals/physician_gls/PDF/myeloma.pdf.

255. What is a histologic criterion for a WHO grade III meningioma? (A) Brain invasion (B) 0 to 3 mitotic figures per 10 hpf (C) 4 to 19 mitotic figures per 10 hpf (D) 20 or more mitotic figures per 10 hpf

Key: D Solution: Brain invasion is a grade II.Greater than 20 mitotic figures per 10 high power fields or rhabdoid features are among the criteria for a grade Ill (anaplastic/malignant) meningioma. References: 2007 WHO Classification of Brain Tumors.

120. What is the appropriate T stage for a 2.5 cm ulcerated superficial spreading melanoma, Breslow depth 3 mm, Clark level IV with 5 mitoses per mm2 ? (A) T1b (B) T2a (C) T3a (D) T3b

Key: D Solution: Breslow depth 2.01-4.0 mm is a T3 lesion. Ulceration results in T3b staging. References: AJCC Staging Manual, 7th Edition, 2010, Springer, p353.

188. The average energy of Cobalt 60 radiation is approximately 1.25 MV. The average energy is based on the: (A) Bragg peak photon emission. (B) Bremsstrahlung spectrum. (C) two 0.511 keV annihilation gammas. (D) two discreet MeV gamma ray emission lines.

Key: D Solution: Co-60 decays by beta emission accompanied by the emission of two discrete photons with energies of 1.17 MeV and 1.34 MeV. Linear accelerators create MV photon beams via bremsstrahlung interactions generated when an electron beam strikes a target. Annihilation gammas are the result of pair production. The Bragg peak is the peak in the dose distribution for heavy charged particles and is thus irrelevant. References: Khan. Physics of Radiation Therapy second ed., Chapter 1: Structure of Matter.

32. According to GORTEC 99-02, when compared to conventional chemoRT for locally advanced head and neck cancer, aggressive accelerated RT: (A) improved the PFS. (B) had less grade 3 or 4 toxicities. (C) was more effective with chemotherapy. (D) cannot replace chemotherapy.

Key: D Solution: Concomitant chemoRT and accelerated radiotherapy independently improve outcomes for patients with locally advanced head and neck squamous-cell carcinoma (HNSCC). Chemotherapy has a substantial treatment effect given concomitantly with radiotherapy and acceleration of radiotherapy cannot compensate for the absence of chemotherapy. The most favorable outcomes are noted in conventional chemoradiotherapy, suggesting that acceleration of radiotherapy is probably not beneficial in concomitant chemoRT schedules. A total of 279 patients randomized to receive conventional chemoRT, 280 to accelerated radiotherapy-chemotherapy, and 281 to very accelerated radiotherapy. Median follow-up was 5.2 years (IQR 4.9-6.2); rates of chemotherapy and radiotherapy compliance were good in all groups. Accelerated radiotherapy-chemotherapy offered no PFS benefit compared with conventional chemoRT (HR 1.02, 95% Cl 0.84 - 1.23; p=0.88) or very accelerated radiotherapy (0.83, 0.69 - 1.01; p=0.060); conventional chemoRT improved PFS compared with very accelerated radiotherapy (0.82, 0.67 - 0.99; p=0.041). Three-year PFS was 37.6% (95% Cl 32.1 - 43.4) after conventional chemoRT, 34.1% (28.7 - 39.8) after accelerated radiotherapychemotherapy, and 32.2% (27.0 - 37.9) after very accelerated radiotherapy. More patients in the very accelerated radiotherapy group had RTOG grade 3-4 acute mucosal toxicity (226 [84%] of 268 patients) compared with accelerated radiotherapy- chemotherapy (205 [76%] of 271 patients) or conventional chemoRT (180 [69%] of 262; p= 0.0001). 158 (60%) of 265 patients in the conventional chemoRT group, 176 (64%) of 276 patients in the accelerated radiotherapy-chemotherapy group, and 190 (70%) of 272 patients in the very accelerated radiotherapy group were intubated with feeding tubes during treatment (p=0.045). References: Bourhis, et al. Concomitant chemoradiotherapy versus acceleration of radiotherapy with or without concomitant chemotherapy in locally advanced head and neck carcinoma (GORTEC 99-02): an open-label phase 3 randomised trial. Lancet Oncol. 2012 Feb; 13(2):145-53. doi: 10.1016/51470-2045(11)70346-1.

196. For whole breast RT, how does the dose distribution of a field-in-field technique (forward planned IMRT) compare to conventional delivery with wedged fields? (A) Decreases heart dose for left sided cases (B) Increases skin dose (C) Improves PTV dose coverage (D) Improves PTV dose homogeneity

Key: D Solution: Coverage will be similar with the two techniques, but using a field-in-field technique can decrease the hot spots (by modulating the field in an additional direction) and thereby improving the dose homogeneity in the target. Choice of wedge or segments will not significantly affect heart dose. Skin dose is not increased with IMRT. The coverage is similar in both cases; it is the reduction of hot spots with IMRT that improves the dose homogeneity. References: Morganti, et al. Forward planned intensity modulated radiotherapy (IMRT) for whole breast postoperative radiotherapy. Is it useful? When? JACMP, 12(2) 2011.

226. What is the appropriate management of a 1cm cT1b1N0M0 cervical cancer patient that desires preservation of fertility? (A) Cone biopsy (B) Brachytherapy (C) Simple trachelectomy (D) Radical trachelectomy with pelvic node dissection

Key: D Solution: Fertility-sparing surgery for "macroinvasive" stage of cervical cancer, just as in non-fertility sparing approach, must involve removal of medial paracervical tissues and pelvic lymph nodes. Brachytherapy would alter the endometrial environment, not address the lymph nodes, and have potential scatter to the ovaries. References: NCCN Clinical Practice Guidelines in Oncology.

11. What is the appropriate dose (Gy) to treat plasmacytomas? (A) 15 (B) 25 (C) 35 (D) 45

Key: D Solution: For osseous and extraosseous plasmacytomas treated with radiation doses should be equal or greater than 45 Gy. References: http://www.nccn.org/professionals/physician_gls/PDF/myeloma.pdf.

126. Which of the following is TRUE of biliary tract and gallbladder cancer? (A) Patterns of failure are similar for extrahepatic cholangiocarcinoma and gall bladder cancer (B) The standard of care for metastatic biliary tract cancer is gemcitabine alone (C) Adjuvant chemoRT is not associated with improved survival with locally advanced gall bladder cancer (D) Patients best suited for adjuvant therapy for biliary tract cancers are those with lymph node positive disease or positive resection margins

Key: D Solution: Gallbladder cancer tends to metastasize while extra hepatic cholangiocarcinoma high a higher local- regional recurrence risk. The ABC trial established gemcitabine and cisplatin as the standard of care for metastatic biliary tract cancers. SEER-Medicare data has shown improvement in survival for locally advanced gallbladder cancer. A recent metaanalysis showed that adjuvant therapy was associated with improved survival, particularly in patients with lymph node disease or resection margin positive disease References: Jarnagin WR, Ruo L, Little SA, Klimstra D, D'Angelica M, DeMatteo RP, Wagman R, Blumgart LH, Fong Y. Patterns of initial disease recurrence after resection of gall bladder carcinoma and hilar cholangiocarcinoma: implications for adjuvant therapeutic strategies. Cancer. 2003 Oct 15. Valle J, Wasan H, Palmer DH, Cunningham D, Anthoney A, Maraveyas A, Madhusudan S, Iveson T, Hughes S, Pereira SP, Roughton M, Bridgewater J; ABC-02 Trial Investigators. Cisplatin plus gemcitabine versus gemcitabine for biliary tract cancer. N Engl J Med. 2010 Apr 8; 362(14):1273-81). Horgan AM, Amir E, Walter T, Knox JJ. Adjuvant therapy in the treatment of biliary tract cancer: a systematic review and meta-analysis. J Clin Oncol. 2012 Jun 1; 30(16):1934- 40.98(8):1689-700. Wang SJ, Lemieux A, Kalpathy-Cramer J, Ord CB, Walker GV, Fuller CD, Kim JS, Thomas CR Jr. Nomogram for predicting the benefit of adjuvant chemoradiotherapy for resected gallbladder cancer. J Clin Oncol. 2011 Dec10; 29(35):4627-32

280. A patient has a gross total resection of a posterior fossa anaplastic ependymoma. Cranial and spinal imaging are negative. What is the most appropriate RT volume? (A) Craniospinal (B) Whole brain (C) Whole ventricle (D) Tumor bed

Key: D Solution: Grade III anaplastic histology is not an indication for craniospinal irradiation in an ependymoma. Positive CSF or spinal seeding are the indications for CSI. References: Merchant J Neurosurg 86, 1997 PMID 9171172; Taylor Ped Blood Cancer 42, 2004. PMID 15049020.

89. What is the minimum threshold dose rate defining high dose brachytherapy? (A) 2 Gy/hr. (B) 4 Gy/hr. (C) 10 Gy/hr. (D) 12 Gy/hr.

Key: D Solution: High dose rate brachytherapy is defined by ICRU as > 12 Gy/hr. Low dose rate brachy is 0.4 to 2 Gy/hr. References: ICRU 38.

216. What is the most common malignancy in children less than 18 months of age? (A) Leukemia (B) Lymphoma (C) Brain tumor (D) Neuroblastoma

Key: D Solution: In descending order of frequency, leukemia, brain tumors, lymphoma, and neuroblastoma are the most common malignancies in children. Neuroblastoma is the most common malignancy in children less than 18 months of age. References: Halperin, et al. Pediatric Radiation Oncology 2011.

85. Relative to photons, how will the therapeutic ratio of protons be altered if the RBE value for 1.1 is NOT considered in the treatment plan? (A) No effect (B) Reduced, due to low tumor dose (C) Increased, due to reduced dose to normal structures (D) Reduced, by increasing the effective dose to normal structures

Key: D Solution: In generic terms, the RBE of 1.1 assigned to proton therapy indicates that protons are approximately 10% more effective in inducing cell kill in comparison to MV photons. Failure to account for this during treatment planning would result therefore in higher effective doses and hence an increased risk of normal tissue damage although, tumor control could potentially be improved. References: Francesco Tommasino and Marco Durante Proton Radiobiology. Cancers (Basel). 2015 Mar; 7(1): 353-381. Paganetti, et al. Relative biological effectiveness (RBE) values for proton beam therapy. Int J Radiat Oncol Biol Phys. 2002 Jun 1; 53(2):407-21. Levin, et al. Proton beam therapy. Br J Cancer. 2005 Oct 17; 93(8):849-54.

277. What does Temozolomide increase when used in the treatment of glioblastoma? (A) Diarrhea (B) Methylation of the MGMT gene promoter region (C) Degradation of the protein product of the MDR1 gene (D) The incidence of pseudoprogression following chemoRT NOTE: This item was 0-weighted for scoring purposes upon post-exam statistical item analysis (did not count for or against candidate in calculation of test scores).

Key: D Solution: In the Stupp study, the addition of concurrent and adjuvant temozolomide improved overall survival. In cultured glioblastoma cells, temozolomide treatment induces expression of the MGMT gene product. Methylation of the MGMT promoter region would be associated with lower MGMT expression. Increased expression of the MDR gene product would be expected to induce chemotherapy resistance following exposure. While temozolomide is not associated with diarrhea, the incidence of pseudoprogress following combined chemoradiation is increased.

73. For a Wilms tumor of the right kidney without anaplasia resected without tumor spill and one positive para-aortic lymph node, what is the COG stage and appropriate radiation therapy? (A) Stage II; No RT (B) Stage II; Right flank and para-aortic RT to 10 Gy in 5 fractions (C) Stage III; Para-aortic RT, without covering the flank, to 10.5 Gy in 7 fractions (D) Stage III; Right flank and para-aortic RT to 10.8 Gy in 6 fractions

Key: D Solution: Lymph node positive disease without metastases is stage III by COG staging. COG treatment guidelines for stage III tumors without tumor rupture but involving lymph nodes are to treat the ipsilateral flank and cover the para-aortic lymph nodes. References: Halperin, et al. Pediatric Radiation Oncology 5th edition, Chapter 13.

16. Which of the following is a risk factor for CNS leukemia? (A) High glucose (B) Low glucose (C) Immature T-cells (D) Mature B-cells

Key: D Solution: Mature B-cell immunophenotype is associated with greater risk of CNS involvement. High LOH and T-cell immunophenotype, not immature, are also associated with a greater risk. References: NCCN ALL Guidelines 1.2015, p. 67.

104. Merkel Cell Cancer is: (A) relatively radio-resistant. (B) associated with HSV 8 virus. (C) staged like basal cell carcinoma. (D) treated primarily with surgery in the absence of metastatic disease.

Key: D Solution: Merkel cell carcinoma is associated with the Merkel cell polyomavirus. HSV 8 is associated with Kaposi's sarcoma. Merkel cell carcinoma is highly radiosensitive. Merkel cell carcinoma has unique staging separate from other skin cancers in the 7th edition AJCC staging manual. First line treatment for localized Merkel cell carcinoma is surgical resection and management of the nodal basin often followed by regional radiation. References: NCCN guidelines Merkel Cell Carcinoma V 1.2015. AJCC Cancer Staging Manual 7th edition. Feng, et al. Clonal integration of a polyomavirus in human Merkel cell carcinoma. Science. 2008 Feb 22; 319(5866):1096-100. Leonard J, et al. Radiation sensitivity of Merkel cell carcinoma cell lines. Int J Radiat Oncol Biol Phys, 32 (1995), pp. 1401-1407.

246. Conducting a meta-analysis: (A) limits statistical power. (B) is not considered an evidence-based resource. (C) limits ability to extrapolate to the general population. (D) requires advanced statistical techniques.

Key: D Solution: Meta-analysis is a method for systematically combining pertinent qualitative and quantitative study data from several selected studies to develop a single conclusion that has greater statistical power. This conclusion is statistically stronger than the analysis of any single study, due to increased numbers of subjects, greater diversity among subjects, or accumulated effects and results. Advantages: Greater statistical power. Confirmatory data analysis. Greater ability to extrapolate to general population affected. Considered an evidence-based resource. Disadvantages: Difficult and time consuming to identify appropriate studies. Not all studies provide adequate data for inclusion and analysis. Requires advanced statistical techniques. Heterogeneity of study populations. References: https://himmelfarb.gwu.edu/tutorials/studydesign101/metaanalyses.html.

62. All of the following treatment options are appropriate for clinical Stage IA1 squamous cell carcinoma of the cervix without LVSI, EXCEPT: (A) simple trachelectomy. (B) extrafascial hysterectomy. (C) cone biopsy with 3mm negative margins. (D) radical hysterectomy with lymph node dissection.

Key: D Solution: Microinvasive (FIGO stage IA1) cervical cancer demonstrating no LVSI in the biopsy specimen can be successfully managed without use of radical surgery. Various treatment options are tailored to patient's wishes and considerations on fertility. References: NCCN Clinical Practice Guidelines in Oncology.

76. Regarding neutron production in a linac: (A) neutron dose is higher in an electron beam than for a photon beam of the same nominal energy. (B) significant neutron dose is generated for linac operating at energies 6 MV or higher. (C) lead is a more effective neutron shielding material than concrete. (D) fast neutrons (100 keV) generated in the treatment head are more biologically damaging than thermal neutrons (eV.)

Key: D Solution: Most neutrons coming from therapeutic radiotherapy accelerators are produced through the (gamma,n) reaction, in which a photon is absorbed in a nucleus, causing a neutron to be emitted. The (gamma,n) reaction is two-to-three orders of magnitude more likely than the comparable (e,n) reaction in electron beams, so neutron generation is not considered a hazard in electron beams, or in x-ray beams having energies below about 10 MV. This is because the energy of the incident photon must exceed the binding energy of the nucleus in the atom (which starts at about 6.6 MeV for lead and iron - relevant components in the linac head). Neutrons lose energy primarily through elastic collisions with other nuclei, which is most efficient when the scattering nuclei have similar mass to the neutron. This makes hydrogen-rich materials like concrete more efficient at moderating neutrons than heavier materials such as lead. Once neutrons have been moderated (thermalized) down to a few eV, they are much less biologically damaging, having an RBE of approximately 5, compared to 20 for fast neutrons. References: Khan FM. "Physics of Radiation Therapy, 4th Edition", Chapter 16.

231. Compared to photon therapy, why are in homogeneities more of a concern in proton therapy? (A) Decreased dose rate (B) Decreased fluence (C) Increased beam energy (D) Increased range uncertainty

Key: D Solution: Options A and C are false. Option B is true but it is not the reason that in homogeneities are of greater importance for protons than for photons. References: The Physics and Technology of Radiation Therapy" McDermott & Orton, pg. 20-58.

203. CNS PNET tumors are MOST commonly managed with what radiation technique for patients 3 years of age or older? (A) Local RT only (B) Electron beam (C) WBRT (D) Craniospinal RT

Key: D Solution: PNET tumors have a high risk of CNS dissemination and require craniospinal axis irradiation. References: http://www.nccn.org/professionals/physicians/pdf.

done

done


Ensembles d'études connexes

FILM CH 05 DVD QUIZ: Composing the Frame

View Set

PrepU Chapter 20 --> Respiratory Disorders

View Set

physics concepts: Unit 14 Magnetic forces, magnetic fields, and Faraday's law

View Set

Writing and Research Process Review

View Set